You are on page 1of 175

Salazar, Paul, Contapay; Suyom, Theresia Katherine Rojas; Tenefrancia, Martin

Benedict Morden; Teves,Sales Case


Hartley James DigestAMADO III CAÑETE;
A.; TILLADA,
Valida, Beverly Jane, Anas;2B Compilation
Veloso, Marc Lester Margallo; Yu, Czar Clinton
Carcedo; Amparado, Lanze Leo, Berro; Bagunas, Lenel john, Giban; Basilan,
KareenMacaso; Bundoc, Eva Marie Cañas; Calades III, Don Miguel Rodrigo
Ablay; Darantinao, Kent Duquiatan; De Jesus, Jeremy Charles, Tabones;
Dimatangal, Leonardo Jr. Elongo; Igana, Jonathan Bon Martin Nariz; Inciso,
Leonard Ray Rivera; MABULAY, ANGELICA JOY, ROSETE.; Ocañada, Ralf
Vincent Dueñas; Olaya, Mary Rose G.; Oracion, Ivy Ruth L.; QUILLOTES,
BALVIN CABUGUANG; Reyes, RustummeMirelleBaduya; ROSALES,
SANDRE ROSE CABILIC; Sabarrelll, Alejandro T.; SagumTristila Marie N.;
Salazar, Paul, Contapay; Suyom, Theresia Katherine Rojas; Tenefrancia, Martin
Benedict Morden; Teves, Hartley James A.; TILLADA, AMADO III CAÑETE;
Valida, Beverly Jane, Anas; Veloso, Marc Lester Margallo; Yu, Czar Clinton
Carcedo; Amparado, Lanze Leo, Berro; Bagunas, Lenel john, Giban; Basilan,
KareenMacaso; Bundoc, Eva Marie Cañas; Calades III, Don Miguel Rodrigo
Ablay; Darantinao, Kent Duquiatan; De Jesus, Jeremy Charles, Tabones;
Dimatangal, Leonardo Jr. Elongo; Igana, Jonathan Bon Martin Nariz; Inciso,
Leonard Ray Rivera; MABULAY, ANGELICA JOY, ROSETE.; Ocañada, Ralf
Vincent Dueñas; Olaya, Mary Rose G.; Oracion, Ivy Ruth L.; QUILLOTES,
BALVIN CABUGUANG; Reyes, RustummeMirelleBaduya; ROSALES,
SANDRE ROSE CABILIC; Sabarrelll, Alejandro T.; SagumTristila Marie N.;
Salazar, Paul, Contapay; Suyom, Theresia Katherine Rojas; Tenefrancia, Martin
Benedict Morden; Teves, Hartley James A.; TILLADA, AMADO III CAÑETE;
Valida, Beverly Jane, Anas; Veloso, Marc Lester Margallo; Yu, Czar Clinton
Carcedo; Amparado, Lanze Leo, Berro; Bagunas, Lenel john, Giban; Basilan,
KareenMacaso; Bundoc, Eva Marie Cañas; Calades III, Don Miguel Rodrigo
Ablay; Darantinao, Kent Duquiatan; De Jesus, Jeremy Charles, Tabones;
Dimatangal, Leonardo Jr. Elongo; Igana, Jonathan Bon Martin Nariz; Inciso,
Leonard Ray Rivera; MABULAY, ANGELICA JOY, ROSETE.; Ocañada, Ralf
Vincent Dueñas; Olaya, Mary Rose G.; Oracion, Ivy Ruth L.; QUILLOTES,
BALVIN CABUGUANG; Reyes, RustummeMirelleBaduya; ROSALES,
SANDRE ROSE CABILIC; Sabarrelll, Alejandro T.; SagumTristila Marie N.;
Salazar, Paul, Contapay; Suyom, Theresia Katherine Rojas; Tenefrancia, Martin
Benedict Morden; Teves, Hartley James A.; TILLADA, AMADO III CAÑETE;
1. Spouses Herrera v. Caguiat GR No. 139173, February 28, 2007

Facts:

Petitioners are registered owners of a lot located in Las Piñas. On March 23,
1900, respondentoffered to buy the lot and petitioners agreed to sell it at
1,500 ₱ per square meter. Respondent thengave ₱100,000 as partial
payment. A few days after, respondent, through his counsel,
wrotepetitioners informing them of his readiness to pay the balance of the
contract price and requestingthem to prepare the Deed of Sale.

Petitioners, through counsel, informed respondent in a letter that


AmparoHerrera would be leavingfor abroad on or before April 15, 1990 and
they are canceling the transaction and that respondentmay recover the
earnest money (₱100,000) anytime. Petitioners also wrote him stating that
theyalready delivered a manager’s check to his counsel in said amount.
Respondent thus filed acomplaint for specific performance and damages with
the RTC of Makati. The trial court ruledthat there was already a perfected
contract of sale between the parties and ordered the petitionersto execute a
final deed of sale in favor of respondent. The Court of appeals affirmed said
decision.

Issue:

Whether or not there was a contract of sale.

Ruling:

The transaction was a contract to sell.

“When petitioners declared in the “Receipt for Partial Payment” that they
-“RECEIVED FROM MR. GODOFREDO CAGUIAT THE AMOUNT OF ONE
HUNDREDTHOUSAND PESOS AS PARTIAL PAYMENT OF OUR LOT SITUATED
IN LAS PIÑAS…MR. CAGUIAT PROMISED TO PAY THE BALANCE OF THE
PURCHASE PRICE ON ORBEFORE MARCH 23, 1990, AND THAT WE WILL
EXECUTE AND SIGN THE FINALDEED OF SALE ON THIS DATE.” there can be
no other interpretation than that they agreed toa conditional contract of
sale, consummation of which is subject only to the full payment of
thepurchase price.

“A contract to sell is akin to a conditional sale where the efficacy or


obligatory force of thevendor’s obligation to transfer title is subordinated to
the happening of a future and uncertainevent, so that if the suspensive
condition does not take place, the parties would stand as if theconditional
obligation had never existed. The suspensive condition is commonly full
payment ofthe purchase price.

“In this case, the “Receipt for Partial Payment” shows that the true
agreement between the partiesis a contract to sell. “First, ownership over
the property was retained by petitioners and was not topass to respondent
until full payment of the purchase price. Second, the agreement between
theparties was not embodied in a deed of sale. The absence of a formal deed
of conveyance is a strongindication that the parties did not intend immediate
transfer of ownership, but only a transfer afterfull payment of the purchase
price. Third, petitioners retained possession of the certificate of titleof the
lot.

“It is true that Article 1482 provides that whenever earnest money is given
in a contract of sale, itshall be considered as part of the price and proof of
the perfection of the contract. However, thisarticle speaks of earnest money
given in a contract of sale. In this case, the earnest money wasgiven in a
contract to sell. The earnest money forms part of the consideration only if
the sale isconsummated upon full payment of the purchase price.

“Clearly, respondent cannot compelpetitioners to transfer ownership of the


property to him.”
2. Nabus v. Pacson GR No. 161318, November 25, 2009

Facts:

The spouses Bate and Julie Nabus were the owners of parcels of land with a
total area of 1,665 square meters, situated in Pico, La Trinidad, Benguet,
duly registered in their names under TCT No. T-9697 of the Register of
Deeds of the Province of Benguet. The property was mortgaged by the
Spouses Nabus to the Philippine National Bank (PNB), La Trinidad Branch, to
secure a loan in the amount of P30,000.00.

On February 19, 1977, the Spouses Nabus executed a Deed of Conditional


Sale covering 1,000 square meters of the 1,665 square meters of land in
favor of respondents Spouses Pacson for a consideration of P170,000.00,
which was duly notarized on February 21, 1977. Pursuant to the Deed of
Conditional Sale, respondents paid PNB the amount of P12,038.86 on
February 22, 19776 and P20,744.30 on July 17, 19787 for the full payment
of the loan. On December 24, 1977, before the payment of the balance of
the mortgage amount with PNB, Bate Nabus died. On August 17, 1978, his
surviving spouse, Julie Nabus, and their minor daughter, Michelle Nabus,
executed a Deed of Extra Judicial Settlement over the registered land
covered by TCT No. 9697. On the basis of the said document, TCT No. T-
177188 was issued on February 17, 1984 in the names of Julie Nabus and
Michelle Nabus.

Meanwhile, respondents continued paying their balance, not in installments


of P2,000.00 as agreed upon, but in various, often small amounts ranging
from as low as P10.009 to as high as P15,566.00,10 spanning a period of
almost seven years, from March 9, 197711 to January 17, 1984.12. There
was a total of 364 receipts of payment. The receipts showed that the total
sum paid by respondents to the Spouses Nabus was P112,455.16,14 leaving
a balance of P57,544.84.

During the last week of January 1984, Julie Nabus, accompanied by her
second husband, approached Joaquin Pacson to ask for the full payment of
the lot. Joaquin Pacson agreed to pay, but told her to return after four days
as his daughter, Catalina Pacson, would have to go over the numerous
receipts to determine the balance to be paid. When Julie Nabus returned
after four days, Joaquin sent her and his daughter, Catalina, to Atty.
Elizabeth Rillera for the execution of the deed of absolute sale.

After a week, Catalina Pacson heard a rumor that the lot was already sold to
petitioner Betty Tolero. On March 28, 2008, respondents Joaquin and Julia
Pacson filed with the Regional Trial Court of La Trinidad, Benguet (trial court)
a Complaint for Annulment of Deeds, with damages and prayer for the
issuance of a writ of preliminary injunction.

Betty Tolero put up the defense that she was a purchaser in good faith and
for value. She testified that it was Julie Nabus who went to her house and
offered to sell the property consisting of two lots with a combined area of
1,000 square meters. She consulted Atty. Aurelio de Peralta before she
agreed to buy the property.
Issues:

1. Whether or not the Deed of Conditional Sale was converted into a contract
of lease.

2. Whether or not the Deed of Conditional Sale was a contract to sell or a


contract of sale.

Ruling:

1. The Deed of Conditional Sale entered into by the Spouses Pacson and the
Spouses Nabus was not converted into a contract of lease. The 364 receipts
issued to the Spouses Pacson contained either the phrase "as partial
payment of lot located in Km. 4" or "cash vale" or "cash value (partial
payment of lot located in Km. 4)," evidencing sale under the contract and
not the lease of the property. Further, as found by the trial court, Joaquin
Pacson’s non-signing of the second page of a carbon copy of the Deed of
Conditional Sale was through sheer inadvertence, since the original contract
and the other copies of the contract were all signed by Joaquin Pacson and
the other parties to the contract.

2. The Court holds that the contract entered into by the Spouses Nabus and
respondents was a contract to sell, not a contract of sale.

- A contract of sale is defined in Article 1458 of the Civil Code, thus:

Art. 1458. By the contract of sale, one of the contracting parties


obligates himself to transfer theownership of and to deliver a determinate
thing, and the other to pay therefor a price certain in money or its
equivalent.

A contract of sale may be absolute or conditional.

- Ramos v. Heruela differentiates a contract of absolute sale and a contract


of conditional sale as follows:

Article 1458 of the Civil Code provides that a contract of sale may be
absolute or conditional. A contract of sale is absolute when title to the
property passes to the vendee upon delivery of the thing sold. A deed of sale
is absolute when there is no stipulation in the contract that title to the
property remains with the seller until full payment of the purchase price. The
sale is also absolute if there is no stipulation giving the vendor the right to
cancel unilaterally the contract the moment the vendee fails to pay within a
fixed period. In a conditional sale, as in a contract to sell, ownership remains
with the vendor and does not pass to the vendee until full payment of the
purchase price. The full payment of the purchase price partakes of a
suspensive condition, and non-fulfillment of the condition prevents the
obligation to sell from arising.36

- Coronel v. Court of Appeals distinguished a contract to sell from a contract


of sale, thus:
Sale, by its very nature, is a consensual contract because it is perfected by
mere consent. The essential elements of a contract of sale are the following:

a. Consent or meeting of the minds, that is, consent to transfer ownership in


exchange for the price;
b. Determinate subject matter; and
c. Price certain in money or its equivalent.

Under this definition, a Contract to Sell may not be considered as a Contract


of Sale because the first essential element is lacking. In a contract to sell,
the prospective seller explicitly reserves the transfer of title to the
prospective buyer, meaning, the prospective seller does not as yet agree or
consent to transfer ownership of the property subject of the contract to sell
until the happening of an event, which for present purposes we shall take as
the full payment of the purchase price. What the seller agrees or obliges
himself to do is to fulfill his promise to sell the subject property when the
entire amount of the purchase price is delivered to him. In other words, the
full payment of the purchase price partakes of a suspensive condition, the
non-fulfillment of which prevents the obligation to sell from arising and,
thus, ownership is retained by the prospective seller without further
remedies by the prospective buyer.

Stated positively, upon the fulfillment of the suspensive condition


which is the full payment of the purchase price, the prospective seller’s
obligation to sell the subject property by entering into a contract of sale with
the prospective buyer becomes demandable as provided in Article 1479 of
the Civil Code which states:

Art. 1479. A promise to buy and sell a determinate thing for a price certain
is reciprocally demandable.

An accepted unilateral promise to buy or to sell a determinate thing for a


price certain is binding upon the promissor if the promise is supported by a
consideration distinct from the price.

It is not the title of the contract, but its express terms or stipulations that
determine the kind of contract entered into by the parties. In this case, the
contract entitled "Deed of Conditional Sale" is actually a contract to sell. The
contract stipulated that "as soon as the full consideration of the sale has
been paid by the vendee, the corresponding transfer documents shall be
executed by the vendor to the vendee for the portion sold."41 Where the
vendor promises to execute a deed of absolute sale upon the completion by
the vendee of the payment of the price, the contract is only a contract to
sell."42 The aforecited stipulation shows that the vendors reserved title to
the subject property until full payment of the purchase price.

If respondents paid the Spouses Nabus in accordance with the stipulations in


the Deed of Conditional Sale, the consideration would have been fully paid in
June 1983. Thus, during the last week of January 1984, Julie Nabus
approached Joaquin Pacson to ask for the full payment of the lot. Joaquin
Pacson agreed to pay, but told her to return after four days as his daughter,
Catalina Pacson, would have to go over the numerous receipts to determine
the balance to be paid.

Unfortunately for the Spouses Pacson, since the Deed of Conditional Sale
executed in their favor was merely a contract to sell, the obligation of the
seller to sell becomes demandable only upon the happening of the
suspensive condition. The full payment of the purchase price is the positive
suspensive condition, the failure of which is not a breach of contract, but
simply an event that prevented the obligation of the vendor to convey title
from acquiring binding force. Thus, for its non-fulfillment, there is no
contract to speak of, the obligor having failed to perform the suspensive
condition which enforces a juridical relation. With this circumstance, there
can be no rescission or fulfillment of an obligation that is still non-existent,
the suspensive condition not having occurred as yet. Emphasis should be
made that the breach contemplated in Article 1191 of the New Civil Code is
the obligor’s failure to comply with an obligation already extant, not a failure
of a condition to render binding that obligation.

Since the contract to sell was without force and effect, Julie Nabus validly
conveyed the subject property to another buyer, petitioner Betty Tolero,
through a contract of absolute sale, and on the strength thereof, new
transfer certificates of title over the subject property were duly issued to
Tolero.

The Spouses Pacson, however, have the right to the reimbursement of their
payments to the Nabuses, and are entitled to the award of nominal
damages.

WHEREFORE, the petition is GRANTED. The Decision of the Court of Appeals


in CA-G.R. CV No. 44941, dated November 28, 2003, is REVERSED and SET
ASIDE.
3. Reyes v. Tuparan GR No. 188064, June 1, 2011

Facts:

Petitioner Mila Reyes owns a three-storey commercial building in Valenzuela


City. Respondent, Victoria Tuparan leased a space on said building for a
monthly rental of P4,000. Aside from being a tenant, respondent also
invested in petitioner's financing business. On June 20, 1988, Petitioner
borrowed P2 Million from Farmers Savings and Loan Bank (FSL Bank) and
mortgaged the building and lot (subject real properties). Reyes decided to
sell the property for P6.5 Million to liquidate her loan and finance her
business. Respondent offered to conditionally buy the real properties for
P4.2 Million on installment basis without interest and to assume the bank
loan. The conditions are the following:

1. Sale will be cancelled if the petitioner can find a buyer of said properties
for the amount of P6.5 Million within the next three months. All payments
made by the respondent to the petitioner and the bank will be refunded to
Tuparan with an additional 6% monthly interest.

2. Petitioner Reyes will continue using the space occupied by her drug store
without rentals for the duration of the installment payments.

3. There will be a lease for 15 years in favor of Reyes for a monthly rental of
P8, 000 after full payment has been made by the defendant.

4. The defendant will undertake the renewal and payment of the fire
insurance policies of the 2 buildings, following the expiration of the current
policies, up to the time the respondent has fully paid the purchase price

They presented the proposal for Tuparan to assume the mortgage to FSL
Bank. The bank approved on the condition that the petitioner would remain
as co-maker of the mortgage obligation.
Petitioner's Contention:

Under their Deed of Conditional Sale, the respondent is obliged to pay a


lump sum of P1.2 Million in three fixed installments. Respondent, however
defaulted in the payment of the installments. To compensate for her delayed
payments, respondent agreed to pay petitioner monthly interest. But again,
respondent failed to fulfill this obligation. The petitioner further alleged that
despite her success in finding another buyer according to their conditional
sale agreement, respondent refused to cancel their transaction. The
respondent also neglected to renew the fire insurance policy of the buildings.

Respondent's Answer:
Respondent alleges that the deed of Conditional Sale of Real Property with
Assumption of Mortgage was actually a pure and absolute contract of sale
with a term period. It could not be considered a conditional sale because the
performance of the obligation therein did not depend upon a future and
uncertain event. She also averred that she was able to fully pay the loan and
secure the release of the mortgage. Since she also paid more than the P4.2
Million purchase price, rescission could not be resorted to since the parties
could no longer be restored to their original positions.

Issue:

Is the conditional sale at bar a contract of sale or a contract to sell? Can the
transaction or obligation be rescinded given that the conditions were not
satisfied?

Ruling:

RTC:
The deed of conditional sale was a contract to sell. It was of the opinion that
although the petitioner was entitled to a rescission of the contract, it could
not be permitted because her non-payment in full of the purchase price
“may not be considered as substantial and fundamental breach of the
contract as to defeat the object of the parties in entering into the contract.”
The RTC believed that respondent showed her sincerity and willingness to
settle her obligation. Hence, it would be more equitable to give respondent a
chance to pay the balance plus interest within a given period of time. The
court ordered the respondent to pay the petitioner the unpaid balance of the
purchase price.

CA:
The CA agreed with the RTC that the remedy of rescission could not apply
because the respondent’s failure to pay the petitioner the balance of the
purchase price in the total amount of ₱805,000.00 was not a breach of
contract, but merely an event that prevented the seller (petitioner) from
conveying title to the purchaser (respondent). Since respondent had already
paid a substantial amount of the purchase price, it was but right and just to
allow her to pay the unpaid balance of the purchase price plus interest.

SC:
The SC agrees that the subject Deed of Conditional Sale with Assumption of
Mortgage is a contract to sell and not a contract of sale. The subject
contract was correctly classified as a contract to sell based on the following
pertinent stipulations:

8. That the title and ownership of the subject real properties shall
remain with the First Party until the full payment of the Second Party
of the balance of the purchase price and liquidation of the mortgage
obligation of ₱2,000,000.00. Pending payment of the balance of the
purchase price and liquidation of the mortgage obligation that was
assumed by the Second Party, the Second Party shall not sell, transfer
and convey and otherwise encumber the subject real properties
without the written consent of the First and Third Party.

9. That upon full payment by the Second Party of the full balance of
the purchase price and the assumed mortgage obligation herein
mentioned the Third Party shall issue the corresponding Deed of
Cancellation of Mortgage and the First Party shall execute the
corresponding Deed of Absolute Sale in favor of the Second Party.

The title and ownership of the subject properties remains with the petitioner
until the respondent fully pays the balance of the purchase price and the
assumed mortgage obligation. Without respondent’s full payment, there can
be no breach of contract to speak of because petitioner has no obligation yet
to turn over the title. The court agrees that a substantial amount of the
purchase price has already been paid. It is only right and just to allow
Tuparan to pay the said unpaid balance of the purchase price to Reyes.
Granting that a rescission can be permitted under Article 1191, the Court
still cannot allow it for the reason that, considering the circumstances, there
was only a slight or casual breach in the fulfillment of the obligation. The
court considered fulfillment of 20% of the purchase price is NOT a
substantial breach. Unless the parties stipulated it, rescission is allowed only
when the breach of the contract is substantial and fundamental to the
fulfillment of the obligation. Whether the breach is slight or substantial is
largely determined by the attendant circumstance. As for the 6% interest,
petitioner failed to substantiate her claim that the respondent committed to
pay it. Petition is denied.
4. Salazar v. CA GR No. 118203, July 5, 1996

Facts:

That defendant Dr. Salazar is the owner of the two (2) parcels of land with
improvements thereon located at 2914 Finlandia Street, Makati, Metro
Manila and covered by Transfer Certificate of Title Nos. 31038 and 31039 of
the Registry of Deeds of Makati; that Dr. Salazar offered to sell his
properties to JonetteBorres for One Million pesos (P1,000,000.00). The initial
proposal took place at the Dimsum Restaurant, Makati, whereby it was
proposed that the payment of the consideration was to be made within six
(6) months but was objected to by Dr. Salazar and he reduced it to a three
(3) months period that sometime on [May] 28, 1989, JonetteBorres
together with a certain Emilio T. Salazar went to see Dr. Salazar at the
latter's residence in Bataan bearing a copy of a Deed of Absolute Sale and
Deed of Warranty but Dr. Salazar refused to sign because JonetteBorres did
not have the money ready then. In said occasion Dr. Salazar further reduced
the period within which plaintiff may purchase the lots, to one (1) month or
up to June 30, 1989.

JonetteBorres then met again Dr. Salazar on June 2, 1989 at the Ninoy
International Airport who was about to leave for the United States of
America where he is a resident. JonetteBorres had with her the Deed of
Absolute Sale and asked Dr. Salazar to sign said document. Dr. Salazar
reluctantly agreed to sign the document provided that JonetteBorres pays
one half (1/2) of the consideration or P500,000.00 in "cash" by June 15,
1989 and the balance was payable on June 30, 1989. It was during this
occasion that Dr. Salazar again emphasized to JonetteBorres that he needed
the money because he was then buying a property in the United States.

Plaintiff agreed to the above conditions and Dr. Salazar constituted co-
defendant Teresa Dizon as custodian at the Deed of Absolute Sale together
with the Titles of the Land in question with the instruction to Teresa Dizon
not to surrender said documents to JonetteBorres until upon payment of the
full price in "cash".

On June 14, 1989 JonetteBorres informed defendant Dizon that she will be
able to pay the full amount of P1,000,000.00 on June 15, 1989 and on the
next day, she then went to the house of Teresa Dizon to see and get the
documents entrusted to her by Dr. Salazar. The documents not being in
Dizon's possession, they agreed to meet at Metro Bank West Avenue Branch
to get the documents and then to proceed to Makati to meet the plaintiff's
business partner a certain Balao who allegedly gave plaintiff a Far East Bank
and Trust Company check for the amount of P1,500,000.00 with which to
buy the property. For some reason or another JonetteBorres and defendant
Dizon failed to proceed to Makati.

In the meantime or on June 16, 1992, Dr. Salazar made an overseas call to
co-defendant Dizon to inquire if JonetteBorres had already paid the down
payment of P500,000.00 and Teresa Dizon replied to Dr. Salazar that
JonetteBorres had not paid the down payment. Dr. Salazar then ordered
Dizon to stop the sale.

Issue:

Whether or not the so-called Deed of Absolute Sale executed by petitioner


Emilio A. Salazar in favor of private respondent JonetteBorres is a perfected
contract of sale or a mere contract to sell.

Ruling:

It is a contract to sell not contract of sale. The withholding by Salazar


through Dizon of the Deed of Absolute Sale, the certificates of title, and all
other documents relative to the lots is an additional indubitable proof that
Salazar did not transfer to Borres either by actual or constructive delivery
the ownership of the two lots. While generally the execution of a deed of
absolute sale constitutes constructive delivery of ownership, the withholding
by the vendor of that deed under explicit agreement that it be delivered
together with the certificates of titles to the vendee only upon the latter's full
payment of the consideration amounts to a suspension of the effectivity of
the deed of sale as a binding contract.

Undoubtedly, Salazar and Borres mutually agreed that despite the Deed of
Absolute Sale title to the two lots in question was not to pass to the latter
until full payment of the consideration of P1 million. The form of the
instrument cannot prevail over the true intent of the parties as established
by the evidence.

Accordingly, since Borres was unable to pay the consideration, which was a
suspensive condition, Salazar cannot be compelled to deliver to her the deed
of sale, certificates of title, and other documents concerning the two lots. In
other words, no right in her favor and no corresponding obligation on the
part of Salazar were created.
5. Castillo v. Reyes, 539 SCRA 193
Facts:
On November 7, 1997, EmmalizaBohler and respondents negotiated for the
sale of the former’s house and lot located at Poblacion, New Washington,
Aklan, to the latter for the consideration of ₱165,000.00. 2 On the following
day, November 8, they signed an Agreement.
Upon the signing of the said contract, respondents handed to Bohler
₱20,000.00 cash and allegedly a ₱110,000.00-check. Bohler nonetheless
insisted that the entire partial payment should be in cash as she needed it to
redeem the subject property from the bank on the following Monday. She
hence demanded for its payment up to midnight on that day otherwise she
would cancel the sale.
On February 21, 2003, the RTC rendered its Decision6 declaring the
November 8, 1997 Agreement a contract to sell.
Petitioners, thus, filed the instant petition for review on certiorari imputing
the following errors to the CA.
Issue:
Whether the transaction between Bohler and the respondents is a perfected
contract of sale or a mere contract to sell
RULING:
Sale is a consensual contract and is perfected by mere consent, which is
manifested by a meeting of the minds as to the offer and acceptance thereof
on the subject matter, price and terms of payment of the price.
In the instant case, the November 8, 1997 Agreement clearly indicates that
Bohler and the Spouses Reyes had a meeting of the minds on the subject
matter of the contract, the house and lot; on the price, ₱165,000.00; and on
the terms of payment, an initial payment of ₱130,000.00 on the date of
execution of the agreement and the remaining balance on or before
December 15, 1997. At that precise moment when the consent of both
parties was given, the contract of sale was perfected.
The said agreement cannot be considered a contract to sell.
In a contract of sale, the vendor loses ownership over the property and
cannot recover it until and unless the contract is resolved or rescinded;
whereas, in a contract to sell, title is retained by the vendor until full
payment of the price.
The November 8, 1997 Agreement herein cannot be characterized as a
contract to sell because the seller made no express reservation of ownership
or title to the subject house and lot.12 Instead, the Agreement contains all
the requisites of a contract of sale.
WHEREFORE, premises considered, the petition for review on certiorari is
DENIED DUE COURSE.
6. United Muslim and Christian Urban Poor Association, Inc. v. BRYC-
V Development, GR#179653
Facts:
Sea Foods Corporation is the registered owner of Lot No. 300petitioner
United Muslim and Christian Urban Poor Association, Inc.
An organization of squatters occupying Lot No. 300 initiated negotiations
with SFC for the purchase thereof.
UMCUPAI expressed its intention to buy the subject property using the
proceeds of its pending loan application with National Home Mortgage
Finance Corporation
Thereafter, the parties executed a Letter of Intent to selland Letter of Intent
to Purchase.
However, the intended sale was derailed due to UMCUPAI's inability to
secure the loan from NHMF as not all its members occupying Lot No. 300
were willing to join the undertaking.
Lot No. 300 was subdivided into three (3) parts
UMCUPAI purchased Lot No. 300-A
Lot No. 300-B was constituted as road right of way and donated by SFC to
the local government.
UMCUPAI failed to acquire Lot No. 300-C for lack of funds.
UMCUPAI negotiated anew with SFC and was given by the latter another
three months to purchase Lot No. 300-C. However, despite the extension,
the three-month period lapsed with the sale not consummated... because
UMCUPAI still failed to obtain a loan from NHMF.
SFC sold Lot No. 300-C to respondent BRYC-V Development Corporation
UMCUPAI filed with the RTC a complaint against respondents SFC and BRYC
seeking to annul the sale of Lot No. 300-C,UMCUPAI alleged that the sale
between the respondents violated its valid and subsisting agreement with
SFC embodied in the Letter of Intent.
According to UMCUPAI, the Letter of Intent granted it a prior, better, and
preferred right over BRYC in the purchase of Lot No. 300-C.
BRYC said that UMCUPAI's complaint did not state a cause of action since
UMCUPAI had unequivocally recognized its ownership of Lot No. 300-C when
UMCUPAI likewise sent BRYC a Letter of Intent
SFC countered that the Letter of Intent is not, and cannot be considered, a
valid and subsisting contract of sale.
SFC maintained that the Letter of Intent subject to a condition i.e., payment
of the acquisition price, which UMCUPAI failed to do when it did not obtain
the loan from NHMF.
RTC dismissed UMCUPAI's complaint.
Letter of Intent was simply SFC's declaration of intention to sell, and not a
promise to sell, the subject lot.
CA, on appeal, affirmed in toto the RTC's ruling.
Issues:
Is the letter of intent to sell and letter of intent to buy a bilateral reciprocal
contract within the meaning or contemplation of article 1479, first
paragraph, civil code of the Philippines?
Ruling:
UMCUPAI claims that the Letter of Intent did not merely grant the parties
the option to respectively sell or buy the subject property.
Although not stated plainly, UMCUPAI claims that the Letter of Intent is
equivalent to a conditional contract of sale subject only to the suspensive
condition of payment of the purchase price.
The first paragraph of Article 1479 contemplates the bilateral relationship of
a contract to sell as distinguished from a contract of sale which may be
absolute or conditional under Article 1458
Art. 1479. A promise to buy and sell a determinate thing for a price certain
is reciprocally demandable.
An accepted unilateral promise to buy or to sell a determinate thing for a
price certain is binding upon the promisor if the promise is supported by a
consideration distinct from the price.
The case of Coronel v. Court of Appeals
A contract to sell may thus be defined as a bilateral contract whereby the
prospective seller, while expressly reserving the ownership of the subject
property despite delivery thereof to the prospective buyer, binds himself to
sell the said property exclusively to the prospective buyer upon fulfillment of
the condition agreed upon, that is, full payment of the purchase price.
A contract to sell may not even be considered as a conditional contract of
sale where the seller may likewise reserve title to the property subject of the
sale until the fulfillment of a suspensive condition, because in a conditional
contract of sale, the first element of consent is present, although it is
conditioned upon the happening of a contingent event which may or may not
occur.
The prospective seller still has to convey title to the prospective buyer by
entering into a contract of absolute sale.
In a contract to sell, there being no previous sale of the property, a third
person buying such property despite the fulfillment of the suspensive
condition such as the full payment of the purchase price, for instance,
cannot be deemed a buyer in bad faith and the prospective buyer cannot
seek the relief of reconveyance of the property.
In a conditional contract of sale, however, upon the fulfillment of the
suspensive condition, the sale becomes absolute and this will definitely affect
the seller's title thereto.
Applying Article 1544 of the Civil Code, such second buyer of the property
who may have had actual or constructive knowledge of... such defect in the
seller's title, or at least was charged with the obligation to discover such
defect, cannot be a registrant in good faith.
Such second buyer cannot defeat the first buyer's title.
In the instant case, however, the parties executed a Letter of Intent, which
is neither a contract to sell nor a conditional contract of sale.
Nowhere in the Letter of Intent does it state that SFC relinquishes its title
over the subject property, subject only to the condition of complete payment
of the purchase price; nor, at the least, that SFC, although expressly
retaining ownership thereof, binds itself to sell the... property exclusively to
UMCUPAI.
The Letter of Intent to Buy and Sell is just that - a manifestation of SFC's
intention to sell the property and UMCUPAI's intention to acquire the same.
It is a written statement expressing the intention of the parties to enter into
a formal agreement especially a business arrangement or transaction.
SFC expressly declared its "intention" to sell and UMCUPAI expressly
declared its "intention" to buy subject property.
A mere "intention" cannot give rise to an obligation to give, to do or not to
do
A Letter of Intent is not a contract between the parties thereto... because it
does not bind one party, with respect to the other, to give something, or to
render some service
Letter of Intent to Sell fell short of an "offer" contemplated in Article 1319 of
the Civil Code because it... is not a certain and definite proposal to make a
contract but merely a declaration of SFC's intention to enter into a contract.
UMCUPAI's declaration of intention to buy is also not certain and definite as
it is subject to the condition that UMCUPAI shall endeavor to raise funds... to
acquire subject land.
The Letter of Intent/Agreement is not an "option contract" because aside
from the fact that it is merely a declaration of intention to sell and to buy
subject to the condition that UMCUPAI shall raise the necessary funds to pay
the price of the land, and does not contain a... binding promise to sell and
buy, it is not supported by a distinct consideration distinct from the price of
the land intended to be sold and to be bought x xx No option was granted to
UMCUPAI under the Letter of Intent/Agreement to buy subject land to the
exclusion of all others within a fixed period nor was SFC bound under said
Agreement to Sell exclusively to UMCUPAI only the said land within the fixed
period.
Neither can the Letter of Intent/Agreement be considered a bilateral
reciprocal contract to sell and to buy contemplated under Article 1479 of the
Civil Code which is reciprocally demandable.
The Letter of Intent/Agreement does not contain a PROMISE to sell and to
buy subject property.
While UMCUPAI succeeded in raising funds to acquire a portion of Lot No.
300-A, it failed to raise funds to pay for Lot No. 300-C.
Petition is hereby DENIED.
7. People’s Homesite& Housing Corp. v. CA, 133 SCRA 777 (1984)
FACTS:
Plaintiff People’s Homesite& Housing Corporation (PHHC) appealed from the
ruling of the Court Appeals reversing the decision of the trial court affirming
the withdrawal of the award of the plaintiff to respondent spouses of the
subject lot.
The facts show that PHHC passed a resolution awarding Lot 4 of the
consolidation-subdivision plan, containing 4,182.2 square meters at a price
of twenty-one pesos per square meter, however, it is subject to the approval
of the Quezon City Council and to the approval of the OEC (PHHC) Valuation
Committee and higher authorities.
The city council disapproved the proposed consolidated subdivision plan and
another plan was prepared and submitted to the city council for approval.
This time, the council approved the plan with Lot 4 having a reduced area of
2,608.7 square meters. Subsequently, the PHHC board of directors recalled
all the wards of lots to persons who failed to pay the deposit or down
payment. The Mendozas never paid the price of the lot nor paid the initial
deposit.
The Mendoza spouses asked for reconsideration of the withdrawal of the
previous award to them of Lot 4 and for the cancellation of the re-award of
said lot to third parties.
ISSUE:
Was there a perfected contract of sale by the plaintiff with respondent
spouses which bounds the former to sell subject Lot 4?
HELD:
NO, there was no perfected sale of Lot 4. It was conditionally or contingently
awarded to the Mendozas subject to the approval by the city council of the
proposed consolidation-subdivision plan and the approval of the award by
the valuation committee and higher authorities.
When the plan with the area of Lot 4 reduced to 2,608.7 square meters was
approved, the Mendozas should have manifested in writing their acceptance
of the award for the purchase of Lot 4 just to show that they were still
interested in its purchase although the area was reduced and to obviate any
doubt on the matter.
Therefore, the PHCC board of directors acted within its rights in withdrawing
the tentative award.
8. Lim v. CA182 SCRA 566 (1990)

Facts:
The subject of this controversy is a parcel of land consisting of 1,101 square
meters and located in Diliman, Quezon City.
It was originally owned by Felix, Manuel and Maria Concepcion Orlino, who
mortgaged it to the Progressive Commercial Bank as security for a
P100,000.00 loan on July 1, 1965. The loan not having been paid, the
mortgage was foreclosed and the bank acquired the property as the highest
bidder at the auction sale on March 28, 1969. The mortgagee thereafter
transferred all its assets, including the said land, to the Pacific Banking
Corporation (PBC).
On May 22, 1975, the Orlinos, and their respective spouses (hereinafter
referred to as the private respondents), who had remained in possession of
the land, made a written offer to PBC to repurchase the property.
The bank, through its Assistant Vice-President, sent a letter, dated
November 9, 1977, to the private respondents' counsel.
One year later, on November 2, 1978, PBC advised the private respondents
that if the transaction was not finalized within 30 days.
On April 8, 1980, or two years later, PBC reminded the private respondents
of its letter of November 2, 1978
On September 30, 1980, the private respondents filed a complaint in the
Regional Trial Court of Quezon City against the petitioners and PBC for the
annulment of the deed of sale on the ground that the subject land had been
earlier sold to them. The petitioners claim they are purchasers in good faith.
Issue:
Whether the petitioner acted in good faith but whether there was in fact a
prior sale of the same property to the private respondents?
Was the transaction between private respondents and PBC, as embodied in
the letter of November 9, 1977, a contract to sell or a contract of sale?
Ruling:
It is not enough to say that the contract of sale being consensual, it became
effective between the bank and the private respondents as of November 9,
1977. There is no question about that; but such agreement is like putting
the cart before the horse. Precisely, our purpose is to ascertain to what
particular undertakings the parties have given their mutual consent so we
can determine the nature of their agreement.
In the case at bar, the private respondents obligated themselves to deliver
to the bank the sum of P160,000.00 and their share of 2,901.15 square
meters on a property situated in Caloocan City. In the letter of PBC dated
November 9, 1977, they were requested to "expedite the loan (they were
negotiating for this purpose) so we can consummate the transaction as soon
as possible". That was in 1977. In 1978, they were reminded of their
obligation and asked to comply within thirty days. They did not. On April 8,
1980, they were reminded of that letter of November 2, 1978, and again
asked to comply; but again they did not. Surely, the bank could not be
required to wait for them forever, especially so since they remained in
possession of the property and there is no record that they were paying
rentals. Under the circumstances, PBC had the right to consider the contract
to sell between them terminated for non-payment of the stipulated
consideration. We hereby confirm that rescission.
Having arrived at these conclusions, the Court no longer finds it necessary to
determine if the petitioners acted in bad faith when they purchased the
subject property. The private respondents lost all legal interest in the land
when their contract to sell was rescinded by PBC for their non-compliance
with its provisions. As that contract was rito longer effective when the land
was sold by PBC to the petitioners, the private respondents had no legal
standing to assail that subsequent transaction. The deed of sale between
PBC and the petitioners must therefore be sustained.
WHEREFORE, the petition is GRANTED and the challenged decision of the
Court of Appeals is REVERSED. TCT No. 268623 in favor of the petitioners is
recognized as valid and the complaint for the annulment of the deed of sale
dated May 14, 1980, is hereby dismissed. Costs against the private
respondents.
SO ORDERED.
9. SPOUSES PINGOL v. HON. COURT OF APPEALS
G.R. No. 102909 September 6, 1993,
A deed of sale is absolute in nature although denominated as a "Deed
of Conditional Sale" where there is no stipulation in the deed that title to the
property sold is reserved in the seller until the full payment of the price, nor
is there a stipulation giving the vendor the right to unilaterally resolve the
contract the moment the buyer fails to pay within a fixed period.Additionally,
in a contract of sale, the title passes to the vendee upon the delivery of the
thing sold, whereas in a contract to sell, by agreement, ownership is
reserved in the vendor and is not to pass until the full payment of the price.
FACTS:
Vicente Pingol was the owner of the subject lot. He executed in favor
of Francisco Donasco a deed of absolute sale under which the purchase price
will be paid in installments. Donasco immediately took possession of the
subject lot and constructed a house thereon. He started paying the monthly
installments but was able to pay only for up to two years. When he died, the
subject house and lot remained in the possession of his heirs. Now, the heirs
filed an action for Specific Performance against spouses Pingol, and prayed
that the defendants be ordered to accept the payment of the balance for the
agreed price on the lot. In their answer, spouses Pingol argued that the deed
of sale embodied a conditional contract of sale as the consideration is to be
paid on installment basis, and considering the breach by Francisco of his
contractual obligation, the sale was deemed to have been cancelled.
ISSUE:
Whether the parties entered into a contract to sell.
RULING:
NO. The plain and clear tenor of the deed of sale and Pingol’s failure to
reserve his title lead to the conclusion that the deed embodies a contract of
sale. A deed of sale is absolute in nature although denominated as a "Deed
of Conditional Sale" where there is no stipulation in the deed that title to the
property sold is reserved in the seller until the full payment of the price, nor
is there a stipulation giving the vendor the right to unilaterally resolve the
contract the moment the buyer fails to pay within a fixed period. The deed of
sale in this case contains neither stipulation. In addition, the contract here
being one of absolute sale, the ownership of the subject lot was transferred
to the buyer upon the actual and constructive delivery thereof. The
constructive delivery of the subject lot was made upon the execution of the
deed of sale while the actual delivery was effected when the heirs took
possession of and constructed a house.
10. Medina v. Collector of Internal Revenue
G.R. No. L-15113 January 28, 1961

FACTS:

On 20 May 1944, Antonio Medina married Antonia Rodriguez. Before


1946, the spouses had neither property nor business of their own. Later,
however, Antonio acquired forest concessions in the municipalities of San
Mariano and Palanan, Isabela. From 1946 to 1948, the logs cut and removed
by the Antonio from his concessions were sold to different persons in Manila
through his agent, Mariano Osorio. In 1949, Antonia started to engage in
business as a lumber dealer, and up to around 1952, Antonio sold to her
almost all the logs produced in his San Mariano concession. Antonia, in turn,
sold in Manila the logs bought from her husband through the same agent,
Mariano Osorio. The proceeds were either received by Osorio for Antonio or
deposited by said agent in Antonio’s current account with the PNB.

On the thesis that the sales made by Antonio to his wife were null and
void pursuant to the provisions of Article 1490 of the Civil Code of the
Philippines, the Collector considered the sales made by Antonia as Antonio’s
original sales taxable under Section 186 of the National Internal Revenue
Code and, therefore, imposed a tax assessment on Antonio. On 30
November 1963, Antonio protested the assessment; however, the Collector
insisted on his demand. On 9 July 1954, Antonio filed a petition for
reconsideration, revealing for the first time the existence of an alleged
premarital agreement of complete separation of properties between him and
his wife, and contending that the assessment for the years 1946 to 1952 had
already prescribed. After one hearing, the Conference Staff of the Bureau of
Internal Revenue eliminated the 50% fraud penalty and held that the taxes
assessed against him before 1948 had already prescribed. Based on these
findings, the Collector issued a modified assessment, demanding the
payment of only P3,325.68. Antonio again requested for reconsideration, but
the Collector, in his letter of 4 April 1955, denied the same.

ISSUE:

Whether or not the sales made by the petitioner to his wife could be
considered as his original taxable sales.

RULING:

Article 1490 of the Civil Code Article 7 and 10 of the Code of


Commerce merely state, under certain conditions, a presumption that the
wife is authorized to engage in business and for the incidents that flow
therefrom when she so engages therein. The transactions permitted therein
however are those entered into with strangers, and do not constitute
exceptions to the prohibitory provisions of Article 1490 against sales
between spouses.
The government is always an interested party to all matters involving
taxable transactions and qualified to question their validity or legitimacy
whenever necessary to block tax evasion. It cannot be contended thus that
the Collector cannot assail the questioned sales, he being a stranger to said
transactions.
Contracts violative of the provisions of Article 1490 of the Civil Code
are null and void (Uy Sui Pin vs. Cantollas, 70 Phil. 55; UyCoque vs. Sioca,
45 Phil. 43). In the present case, being void transactions, the sales made by
the petitioner to his wife were correctly disregarded by the Collector in his
tax assessments that considered as the taxable sales those made by the wife
through the spouses’ common agent, Mariano Osorio.
11. Calimlim-Canullas v. Fortun
G.R. No. L-57499 June 22, 1984

FACTS:

Petitoner Mercedes Calimlim-Canullas and Fernando Canullas were married


in 1962, with 5 children and were living on a house situated on a land
inherited by the latter. In 1978, Fernando abandoned his family and lived
with Corazon Daguines. In 1980, Fernando sold the house and lot to
Daguines, who initiated a complaint for quieting of title. Mercedes resisted,
claiming that the house and lot were conjugal properties, and the sale was
null and void for she had not consented thereto.

ISSUES:
1. Whether or not the construction of a conjugal house on the exclusive
property of the husband ipso facto gave the land a character of
conjugal property?

2. Whether or not the sale of the lot together with the house and
improvements thereon was valid under the circumstances surrounding
the transaction.

RULING:

1. Bothe the land and the building belong to the conjugal partnership but
the conjugal partnership is indebted to the husband for the value of
the land. The spouse owning the lot becomes a creditor of the conjugal
partnership for the value of the lot, which value would be reimbursed
at the liquidation of the conjugal partnership. Fernando could not have
alienated the house and lot to Daguines since Mercedes had not given
her consent to said sale.

2. The contract of sale was null and void for being contrary to morals and
public policy. The sale was made by a husband in favor of a concubine
after he abandoned his family and left the conjugal home where his
wife and children lived and from whence they derived their support.
That sale was subversive of the stability of the family, a basic social
institution which public policy cherishes and protects. The law
emphatically prohibits the spouses from selling property to each other
subject to certain exceptions. Similarly, donations between spouses
during married are prohibited. And this is so because if transfers or
conveyances between spouses were allowed during marriage, that
would destroy the system of conjugal partnership, a basic policy in
Civil Law. It was also designed to prevent the exercise of undue
influence by one spouse over the other, as well as to protect the
institution of marriage, which is the corner stone of family law. The
prohibitions apply to a couple living as husband and wife without
benefit of marriage, otherwise the condition of those who incurred guilt
would turn out to be better than those in legal union. Those provisions
are dictated by public interest and their criterion ust be imposed upon
the wig of the parties.
12. Philippine Trust Co. v. Roldan
G.R. No. L-8477 May 31, 1956

FACTS:

Mariano Bernardo, a minor, inherited 17 parcels of land from his deceased


father. Respondent Maraiano’s step-mother, was appointed as his guardian.
As guardian, she sold the 17 parcels to Dr. Ramos, her brother-in-law for
P14,700.00. After a week, Dr. Ramos sold the lands to her for P15,000.00.
Subsequently, she sold 4 out of 17 parcels to Emilio Cruz. Petitioner replaced
Roldan as guardian, and two months thereafter, this litigation sought to
declare as null and void the sale to Dr. Ramos and the sale to Emilio Cruz.

ISSUE:

Whether the sale of the land by the guardian is null and void for being
violative of the prohibition for a guardian to purchase either in person or
through the mediation of another property of her ward.

RULING:

Remembering the general doctrine that guardianship is a trust of the highest


order, and the trustee cannot be allowed to have any inducement to neglect
his ward’s interest, and in line with the Cort’s suspicion whenever the
guardian acquires ward’s property we have no hesitation to declare that in
this case, in the eyes of the law, Socorro Roldan took by purchase her
ward’s parcels thru Dr. Ramos, and that Article 1459 of the Civil Code
applies.
13. DOMINGO D. RUBIAS vs. ISAIAS BATILLER, G.R. No. L-35702,
May 29, 1973

Facts:

Before the war with Japan, Francisco Militante filed an application for
registration of the parcel of land in question. After the war, the petition was
heard and denied. Pending appeal, Militante sold the land to petitioner, his
son-in-law. Plaintiff filed an action for forcible entry against respondent.
Defendant claims the complaint of the plaintiff does not state a cause of
action, the truth of the matter being that he and his predecessors-in-interest
have always been in actual, open and continuous possession since time
immemorial under claim of ownership of the portions of the lot in question.

Issue:

Whether or not the contract of sale between appellant and his father-in-law
was void because it was made when plaintiff was counsel of his father-in-law
in a land registration case involving the property in dispute.

Ruling:

The stipulated facts and exhibits of record indisputably established plaintiff's


lack of cause of action and justified the outright dismissal of the complaint.
Plaintiff's claim of ownership to the land in question was predicated on the
sale thereof made by his father-in- law in his favor, at a time when
Militante's application for registration thereof had already been dismissed by
the Iloilo land registration court and was pending appeal in the Court of
Appeals.

Article 1491 of our Civil Code (like Article 1459 of the Spanish Civil Code)
prohibits in its six paragraphs certain persons, by reason of the relation of
trust or their peculiar control over the property, from acquiring such
property in their trust or control either directly or indirectly and "even at a
public or judicial auction," as follows: (1) guardians; (2) agents; (3)
administrators; (4) public officers and employees; judicial officers and
employees, prosecuting attorneys, and lawyers; and (6) others especially
disqualified by law.

Fundamental consideration of public policy render void and inexistent such


expressly prohibited purchase (e.g. by public officers and employees of
government property intrusted to them and by justices, judges, fiscals and
lawyers of property and rights in litigation and submitted to or handled by
them, under Article 1491, paragraphs (4) and (5) of our Civil Code) has
been adopted in a new article of our Civil Code, viz, Article 1409 declaring
such prohibited contracts as "inexistent and void from the beginning."

Indeed, the nullity of such prohibited contracts is definite and permanent


and cannot be cured by ratification. The public interest and public policy
remain paramount and do not permit of compromise or ratification. In his
aspect, the permanent disqualification of public and judicial officers and
lawyers grounded on public policy differs from the first three cases of
guardians, agents and administrators (Article 1491, Civil Code), as to whose
transactions it had been opined that they may be "ratified" by means of and
in "the form of a new contact, in which cases its validity shall be determined
only by the circumstances at the time the execution of such new contract.
The causes of nullity which have ceased to exist cannot impair the validity of
the new contract. Thus, the object which was illegal at the time of the first
contract, may have already become lawful at the time of the ratification or
second contract; or the service which was impossible may have become
possible; or the intention which could not be ascertained may have been
clarified by the parties. The ratification or second contract would then be
valid from its execution; however, it does not retroact to the date of the first
contract."
14. BERNARDITA R. MACARIOLA vs. HONORABLE ELIAS B.
ASUNCION, A.M. No. 133-J, May 31, 1982

Facts:

Reyes siblings filed a complaint for partition against Macariola, concerning


the properties left by their common father, Francisco Reyes. Asuncion was
the judge who rendered the decision, which became final for lack of an
appeal. A project of partition was submitted to Judge Asuncion after the
finality of the decision. This project of partition was only signed by the
counsel of the parties, who assured the judge that they were given
authorization to do so.

One of the properties in the project of partition was Lot 1184, which was
subdivided into 5 lots. One of these lots (Lot 1184-D) was sold to Anota, a
stenographer of the court, while another (Lot 1184-E) was sold to Dr.
Galapon, who later on sold a portion of the same lot to Judge Asuncion and
his wife. A year after, spouses Asuncion and Dr. Galapon sold their
respective shares over the lot to Traders Manufacturing and Fishing
Industries. At the time of the sale, Judge Asuncion and his wife were both
stockholders, with Judge Asuncion as President and his wife as secretary of
said company.

A year after the company’s registration with the SEC, Macariola filed a
complaint against Judge Asuncion alleging: • that he violated Art. 1491 (5)
of the Civil Code in acquiring a portion of the lot, which was one of those
properties involved in the partition case; and • that he violated Art 14 (1 and
5) of the Code of Commerce, Sec 3 (H) of RA 3019, Sec 12, Rule XVIII of
the Civil Service Rules, and Canon 25 of the Canons of Judicial Ethics by
associating himself with a private company while he was a judge of the CFI
of Leyte. This case was referred to Justice Palma of the CA for investigation,
report and recommendation. After hearing, the said Investigating Justice
recommended that Judge Asuncion should be reprimanded or warned in
connection with the complaints filed against him.

Issue/s:

1. Whether or not Judge Asuncion violated Art 1491 (5) of the Civil Code in
acquiring by purchase a portion of Lot 1184-E, which was among those
properties involved in the partition case.

2. Whether or not Judge Asuncion violated Art 14 (1 and 5) of the Code of


Commerce, Sec 3 (H) of RA 3019, Sec 12, Rule XVIII of the Civil Service
Rules and Canon 25 of the Canons of Judicial Ethics when he associated
himself with Traders Manufacturing and Fishing Industries, Inc., as
stockholder and a ranking officer

Ruling:

1. NO. Although Art 1491 (5) of the Civil Code prohibits justices, judges
among others from acquiring by purchase the property and rights in
litigation or levied upon an execution before the court, the SC has ruled,
however, that for the prohibition to operate, the sale or assignment of the
property must take place during the pendency of the litigation involving the
property. In this case, when Judge Asuncion purchased a portion of Lot
1184-E, the decision in the partition case was already final because none of
the parties filed an appeal within the reglementary period. Thus, the lot in
question was no longer subject of the litigation. Moreover, Judge Asuncion
did NOT buy the lot directly from the plaintiffs in the partition case but from
Dr. Galapon, who earlier purchased the lot from the plaintiffs. The
subsequent sale from Dr. Galapon to Judge Asuncion is NOT a scheme to
conceal the illegal and unethical transfer of said lot as a consideration for the
approval of the project of partition. As pointed out by the Investigating
Justice, there is no evidence in the record showing that Dr. Galapon acted as
a mere dummy of Judge Asuncion. In fact, Dr. Galapon appeared to be a
respectable citizen, credible and sincere, having bought the subject lot in
good faith and for valuable consideration, without any intervention of Judge
Asuncion.

Although Judge Asuncion did NOT violate Art 1491 (5) of the Civil Code, it
was IMPROPER for him to have acquired the lot in question. Canon 3 of the
Canons of Judicial Ethics requires that judges’ official conduct should be free
from the appearance of impropriety. It was unwise and indiscreet on the part
of Judge Asuncion to have purchased the property that was or had been in
litigation in his court and caused it to be transferred to a corporation of
which he and his wife were ranking officers at the time of such transfer. His
actuations must not cause doubt and mistrust in the uprightness of his
administration of justice.

2. NO. Art 14 (1 and 5) of the Code of Commerce prohibits justices of the


SC, judges and officials of the department of public prosecution in active
service from engaging in commerce, either in person or proxy or from
holding any office or have an direct, administrative or financial intervention
in commercial or industrial companies within the limits of the territory in
which they discharge their duties. However, this Code is the Spanish Code of
Commerce of 1885, which was extended to the Philippines by a Royal
Decree. Upon the transfer of sovereignty from Spain to the US to the
Philippines, Art 14 of the Code of Commerce must be deemed to have been
abrogated because where there is change of sovereignty, the political laws of
the former sovereign are automatically abrogated, unless they are expressly
re-enacted by affirmative act of the new sovereign. There appears to be no
affirmative act that continued the effectivity of said provision.

Sec 3 (H) of RA 3019 provides for instances when public officers are
considered to have committed corrupt practices, which include having
financial or pecuniary interest in any business, contract or transaction in
connection with which he intervenes or takes part in his official capacity or in
which he is prohibited by the Constitution or by any law from having any
interest. Judge Asuncion cannot be held liable under said provision because
there is no showing that he participated or intervened in his official capacity
in the business or transactions of Traders Manufacturing. In this case, the
business of the corporation in which he participated has obviously no relation
to his judicial office.

Sec 12, Rule XVIII of the Civil Service Rules does NOT apply to members of
the Judiciary, who are covered under RA 296 (Judiciary Act of 1948) and Art
X (7) of the 1973 Constitution. Under Sec 67 of RA 296, the power to
remove or dismiss judges is vested in the President of the Philippines, not in
the CSC, and only on 2 grounds—serious misconduct and inefficiency. Under
the 1973 Constitution, only the SC can discipline judges of the inferior courts
as well as other personnel of the Judiciary. Judges cannot be considered as
subordinate civil service officers or employees because the Commissioner of
the CSC is not the head of the Judiciary department. Moreover, only
permanent officers in the classified service are subject to the jurisdiction of
the CSC. Judges, however, are not within this classification, as they are
considered to be non-competitive or unclassified service of the government
as a Presidential appointee.
Canon 25 of the Canons of Judicial Ethics reminds judges to abstain from
making personal investments in enterprises, which are apt to be involved in
litigation in his court. Judge Asuncion and his wife, however, had withdrawn
from the corporation and sold their shares to third parties only 22 days after
its incorporation, which indicates that Judge Asuncion realized that their
interest in the corporation contravenes said Canon. The Court even
commended the spouses for such act.
15. THE DIRECTOR OF LANDS vs. SILVERETRA ABABA, G.R. No. L-
26096, February 27, 1979

Facts:

The adverse claimant Atty. Fernandez was retained as counsel by petitioner


(Abarquez) in a civil case for the annulment of a contract of sale with the
right of repurchase and for the recovery of the land which was the subject
matter thereof. Unable to compensate his lawyer whom he also retained for
his appeal, the petitioner executed a document whereby he obliged himself
to give to his lawyer ½ of whatever he might recover from lots 5600 and
5602 should the appeal prosper.

The real property sought to be recovered was actually the share of petitioner
in lots 5600 and 5602 which were part of the estate of his deceased parents
and which were partitioned among the heirs, which included the petitioner
and his sister.

The case having been resolved and title having been issued to the petitioner,
adverse claimant waiter for the petitioner to compy with his obligation under
the document executed by him by delivering the ½ portion of the said
parcels of land. Petitioner refused to comply with his obligation and instead
offered to sell the whole parcels of land to spouses larrazabal. Then, adverse
claimant immediately took steps to protect his interest by filing a motion to
annotate his attorney’s lien and by notifying the prospective buyer of his
claim over the ½ portion of the land.

The motion was granted. The annotation of adverse claim appeared on the
new transfer certificate of title. This adverse claim became the subject of
cancellation proceedings filed by petitioner-spouses. The trial court resolved
the case in favor of the adverse claimant. On appeal, petitioners contended
that a contract for a contingent fee violates Art. 1491 because it involves an
assignment of property subject of litigation.

Issue:

Whether or not the contract for a contingent fee as basis of interest of Atty.
Fernandez is prohibited under Art. 1491 of the Civil Code.

Ruling:

NO. The Contention is without merit. Art. 1491 prohibits the sale or
assignment between the lawyer and his client of property which is the
subject of litigation. For the prohibition to operate, the sale or assignment of
property must take place during the pendency of the litigation involving the
property.

Likewise, under American Law, the prohibition does not apply to “cases
where after completion of litigation the lawyer accepts on account of his fees
and interest in the assets realized by the litigation. There is clear distinction
between such cases and one in which the lawyer speculates on the outcome
of the matter in which he is employed.

Further, a contract for a contingent fee is not covered by Art. 1491 because
the transfer or assignment of the property in litigation takes effect only after
the finality of a favourable judgment. In the instant case, the attorney’s fees
of Atty. Fernandez, consisting of ½ of whatever the petitioner might recover
from his share in the lots in question is contingent upon the success of the
appeal. Hence, the payment of the attorney’s fees, that is the transfer or
assignment of ½ of the property in litigation will take place only if the appeal
prospers. Therefore, the transfer actually takes effect after the finality of a
favourable judgment rendered on appeal and not during the pendency of
litigation involving the property in question. Consequently, the contract for a
contingent fee is not covered by Art. 1491 of the Civil Code.
16. MAHARLIKA PUBLISHING vs. TAGLE, G.R. No. L-65594. July 9,
1986

Facts:

GSIS owned a parcel of land with a building and printing equipment in Paco,
Manila. It was sold to Maharlika in a Conditional Contract of Sale with the
stipulation that if Maharlika failed to pay monthly installments in 90 days,
the GSIS would automatically cancel the contract. Because Maharlika failed
to pay several monthly installments, GSIS demanded that Maharlika vacate
the premises. Even though Maharlika refused to do so, the GSIS published
an advertisement inviting the public to bid in a public auction. A day before
the scheduled bidding, Adolfo Calica, the President of Maharlika, gave the
GSIS head office 2 checks worth 11,000 and a proposal for a compromise
agreement. The GSIS General Manager Roman Cruz gave a not to Maharlika
saying “Hold Bidding. Discuss with me.” However, the public bidding took
place as scheduled and the property was subsequently awarded to Luz
Tagle, the wife of the GSIS Retirement Division Chief. Maharlika demanded
that the sale be considered null and void, as Mrs. Tagle should have been
disqualified from bidding for the GSIS property. RTC and CA both ruled that
the Tagles were entitled to the property and Maharlika should vacate the
premises.

Issue:

Whether or not Tagle are entitled to the property.

Ruling:

NO. The sale to them was against public policy. First of all, the GSIS head
office was stopped from claiming that they did not give the impression to
Maharlika that they were accepting the proposal for a compromise
agreement. The act of the general manager is binding on GSIS. Second,
Article 1491 (4) of the CC provides that public officers and employees are
prohibited from purchasing the property of the state or any GOCC or
institution, the administration of which has been entrusted to them cannot
purchase, even at public or judicial auction, either in person or through the
mediation of another. The SC held that as an employee of the GSIS,
Edilberto Tagle and his wife are disqualified from bidding on the property
belonging to the GSIS because it gives the impression that there was politics
involved in the sale. It is not necessary that actual fraud be shown, for a
contract which tends to injure the public service is void although the parties
entered into it honestly and proceeded under it in good faith.
17. Mangayao v. de Guzman, 55 SCRA 540 (1974)

Facts:
Petitioners are non-Christian Filipinos of the Subano tribe who filed on March
21, 1960 an action for the recovery of property and declaration of nullity of
contract against respondents, SantayLasud and Guintana Cia Lasud, in the
Court of First Instance of Zamboanga del Sur where they obtained a decision
in their favor, affirmed on appeal by the Supreme Court on May 29, 1964.
By virtue of such decision which became final and executory on October 5,
1964, the petitioners, on April 1, 1965, placed in possession of the property
in question and reimbursed private respondents the sum of five thousand
pesos. There was in the meanwhile an action by private respondents for the
annulment of such judgment, notwithstanding its having been affirmed by
the Supreme Court and becoming final and executory. Respondent Judge
surprisingly issued an order restraining petitioners, TumipusMangayao, and
GuimandaBubungan, to desist from executing or causing the execution of
the decision in Civil Case No. 575.

The Supreme Court immediately issued a resolution requiring respondents to


answer and granting the writ of preliminary injunction prayed for. As was to
be expected, no valid issue could be raised as to the legal question involved
on the above facts. The situation presented is that of a judgment final and
executory, from this Court no less, being sought to be thwarted by private
respondents. What was indeed surprising was the receptivity of respondent
Judge to such an unwarranted move.

Issue:

Whether or not the decision affirmed by the SC in Mangayao v. Lasud may


be annulled. WON the sale is valid.

Ruling:

In Mangayao v. Lasud, a decision was rendered declaring the deed of sale


null and void ab initio, declaring plaintiffs the owners of disputed land. The
SC cited their 1964 decision in Mangayao v. Lasud, which is the law of this
case. Justice J.B.L. Reyes as ponente, after noting the plain and explicit
provision in the Administrative Code 14 and the Public Land Act 15 requiring
the approval of the authorities concerned to deeds of sale by illiterate non-
Christians, as petitioners in this case, continued in this wise: "The plain text
of both law clearly imports that non-approved conveyances and
encumbrances of realty by illiterate non-Christians (which appellees are
admitted to be) are not valid, i.e., not binding or obligatory; they are ab
initio void, as correctly held by the appealed decision. The approval of the
executive authority is not in the nature of a ratification of a defective
conveyance; such approval is an essential requisite for its validity, and
without it the proposed contract is absolutely void or inexistent. To hold the
contract as merely voidable, i.e., as operative and binding if not
disapproved, would not only do violence to the text of the statutes that
requires executive approval, and not disapproval, but would nullify the
obvious intent of the statute to guard the patrimony of illiterate non-
Christians from those who are inclined to prey upon their ignorance or
ductility (Porkan vs. Yatco, 70 Phil. 161; Porkan vs. Navarro, 73 Phil. 698;
Madale vs. Sa Raya, 49 Off. Gaz. 536), since it is not to be expected that the
illiterate non-Christian who signs away his real property for lack of
instructions and discrimination, would thereafter be sharp enough to ask the
executive authority to refuse approval of his contract; nor would the literate
buyer be at all likely to do so. The net result of appellants' 'voidable
conveyance' theory, therefore, would be that the illiterate non-Christian
could be stripped of his immovable just as if the protective statutes
heretofore quoted had not been enacted at all."

Respondent Judge thus did manifest a failure to abide not only by a final
decision of this Court, but by the clear policy of the law given expression in
such vigorous and forthright language by Justice J.B.L. Reyes. When it is
further considered that the unrest in that region of the Philippines is partly
attributed to the exploitation of the poor and the oppressed perpetrated by
those with means, irrespective of the faith that they profess, it becomes
even more manifest why respondent Judge must be taken to task

The writ of certiorari is granted and the order of the lower court of April 29,
1965 for the issuance of a preliminary injunction is nullified and set aside.
Respondent Judge, or whoever is acting in his place, is directed to dismiss
Civil Case No. 798 of the Court of First Instance of Zamboanga del Sur
entitled, "SantayLasud and Guintana Cia Lasud v. Mangayao, Bubungan, The
Development Bank of the Philippines and the Province of Zamboanga del
Sur" for annulment of judgment with preliminary injunction filed by private
respondents SantayLasud and Guintana Cia Lasud. The writ of preliminary
injunction issued by this Court by virtue of its resolution of September 10,
1965 is made permanent. With costs against respondents.
18. RESTITUTO DE LEON vs. COURT OF APPEALS, JUANITA RAMOS
and MAXIMO PEREZ, G.R. No. 88788, September 4, 1992

Facts:
The petitioner is challenging the purported sale to the private respondents of
two parcels of land which he claims is his own by right of inheritance. The
said properties were part of the Buenavista Estate purchased by the Republic
of the Philippines for distribution among landless tenants and farmers. On
April 1, 1955, Lot No. S-117 was sold to Manuel de Leon by the Department
of Agriculture on behalf of the Republic. On August 5, 1969, Lot 43 thereof
was also sold by the Republic, through the Land Authority, to the heirs of
Manuel de Leon, represented by Restituto, his grandson.

The first Deed of Sale carries the limitation that it shall not be sold,
assigned, encumbered, mortgaged or transferred, within the period of five
(5) years from the date hereof without first obtaining the written consent of
the Secretary of Agriculture and Natural Resources and that except by
hereditary succession, it shall not be conveyed, transferred or assigned in
favor of any person who is not landless and disqualified to acquire or own
land in the Philippines. The second sale was subject to a similar condition
provided that it shall not be subdivided, sold or in any manner transferred or
encumbered, within the period of fifteen years from the date of execution,
without first obtaining the written consent of Governor of the Land Authority
and only to persons who are qualified to purchased said land under CA 539,
RA 1162, as amended, RA 1400 as amended and/or RA 3844 or to
government banking institutions or agencies or to any private banking
institutions.

On July 24, 1969, the herein private respondents filed a complaint against
the petitioner for partition of the lands and accounting, alleging that they
had bought 1/2 of the lands from Maria de los Santos, the widow of Manuel
de Leon, by virtue of a "TuluyangBilihan" dated March 18, 1959.However, de
los Santos failed to deliver possession to them until her death on February 5,
1960. The petitioner, who had succeeded her in the lands, had resisted their
demands for accounting of the income and averred that the subject
properties belonged to him as the sole heir of Manuel de Leon. He alleged
that "TuluyangBilihan" was a nullity because Maria de los Santos had no
authority to convey the properties during the prohibited period without the
written consent of the appropriate authorities.

After trial, Judge Benigno M. Puno sustained the private respondents in a


decision dated April 15, 1977. This was affirmed on appeal by the
respondent court on February 16, 1989, and reconsideration was denied on
June 15, 1989.

The Court of Appeals agreed that the "TuluyangBilihan" was genuine and
valid and that the alleged lack of the stipulated written consent could be
invoked only by the Republic of the Philippines and not by the petitioner. He
was not a party to the "TuluyangBilihan." Besides, the said stipulations were
not applicable to cases of hereditary succession, and De los Santos, who sold
the lands, was the heir of her husband.
The petitioner has seek for relief.

Issue:
Whether or not the sale under "TuluyangBilihan" violated the conditions and
therefore void.

Ruling:
The sale under "TuluyangBilihan" dated March 18, 1959, is null and void ab
initio for violation of the conditions stipulated in the Deed of Sale dated April
1, 1955, and the Deed of Sale dated August 5, 1969. The SC requires the
petitioner to refund to the private respondents the amount of P2,300.00, the
consideration of the annulled "TuluyangBilihan," with legal interest thereon
from March 18, 1959, until it is fully paid.

The stipulations in the first deed of sale in favor of Manuel de Leon were
binding on his heirs, who were also bound directly this time, by the similar
stipulations in the second deed of sale. The purpose of these stipulations
was to keep within the family the property which the government had sold to
the tenant or farmer for a minimum cost to enable him to acquire his own
land. Hence, it was necessary for Maria de los Santos, before selling the
subject properties to the private respondents, to first secure the written
consent to such sale of the Secretary of Agriculture and Natural Resources
(in the case of the first lot) and of the Governor of the Land Authority (in the
case of the second lot).

The only exception to the said conditions is when the land is acquired by or
transferred to another person by hereditary succession. Thus, when the
lands were inherited by Maria de los Santos as the surviving spouse of
Manuel de Leon, there was no need for such written consent but such
consent was still necessary when, as transferee of the properties, she later
sold them to the private respondents. The lands were transferred to the
private respondents by virtue of the "TuluyangBilihan" and not by hereditary
succession. To be valid, therefore, the sale needed the written consent of
the above-named officials.

In any event, as the "TuluyangBilihan" was null and void ab initio, ownership
of the disputed lands was not transferred to the private respondents but
remained with Maria de los Santos. The Republic of the Philippines, if not the
petitioner, may still ask for the reversion of the properties to the State for
violation of the conditions in the deeds of sale. Meanwhile the petitioner
would have preferential rights of possession thereovervis-a-vis the private
respondents, who rely only on the void "TuluyangBilihan."
19. DONATO REYES YAP and MELITONA MARAVILLAS vs. HON.
EZEKIEL S. GRAGEDA, as Judge of the Court of First Instance of
Albay and JOSE A. RICO, GR. No. L-31606, March 28, 1983

Facts:
On April 12, 1939, Maximino Rico, for and in his own behalf and that of the
minors Maria Rico, Filomeno Rico, Prisco Rico, and Lourdes' Rico, executed a
Deed of Absolute Sale over Lot 339 and a portion of Lot 327 in favor of the
petitioner Donato Reyes Yap who was then a Chinese national.

Subsequently, the petitioner as vendee caused the registration of the


instrument of sale and the cancellation of OCT Nos. 29332 and 29410 and
the consequent issuance in his favor of TCT No. T-2433 covering the two lots
subject matter of the Contract of Sale.

After the lapse of nearly fifteen years from and after the execution of the
deed of absolute sale, Donato Reyes Yap was admitted as a Filipino citizen
and allowed to take his oath of allegiance to the Republic of the Philippines.
He was, thereafter, issued Certificate of Naturalization No. 7, File No. 19 of
the Court of First Instance of Albay.

On December 1, 1967, the petitioner ceded the major portion of Lot No. 327
consisting of 1,078 square meters which he acquired by purchase under the
deed of sale in favor of his engineer son, Felix Yap, who was also a Filipino
citizen because of the Filipino citizenship of his mother and the naturalization
of his father Donato Reyes Yap.

Subsequently, Lourdes Rico, aunt and co-heir of respondent Jose A. Rico


sold the remaining portion of Lot 327 to the petitioner who had his rights
thereon duly registered under Act 496. Petitioner, Donato Reyes Yap, has
been in possession of the lots in question since 1939, openly, publicly,
continuously, and adversely in the concept of owner until the present time.

Issue:
Whether or not the sale of land is valid to a naturalized Filipino.

Ruling:
There should be no question that the sale of the land in question in 1939
was inexistent and void from the beginning because it was a contract
executed against the mandatory provision of the 1935 Constitution, which is
an expression of public policy to conserve lands for the Filipinos. But the
factual set-up had changed. The litigated property is now in the hands of a
naturalized Filipino. It is no longer owned by a disqualified vendee. Petitioner
as a naturalized citizen, was constitutionally qualified to own the subject
property. There would be no more public policy to be served in allowing
respondent Rico to recover the land as it is already in the hands of a
qualified person.
20. VILLONCO REALTY COMPANY and EDITH PEREZ DE TAGLE vs.
BORMAHECO, INC., FRANCISCO N. CERVANTES and ROSARIO N.
CERVANTES, 65 SCRA 352, G.R. No. L-26872, July 25, 1975

Facts:
Francisco Cervantes of Bormaheco Inc. agrees to sell to Villonco Realty a
parcel of land and its improvements located in Buendia, Makati. Bormaheco
made the terms and condition for the sale and Villonco returned it with some
modifications.

The sale is for P400 per square meter but it is only to be consummated after
respondent shall have also consummated purchase of a property in Sta. Ana,
Manila. Bormaheco won the bidding for the Sta. Ana land and subsequently
bought the property.

Villonco issued a check to Bormaheco amounting to P100,000 as earnest


money. After 26 days from signing the contract of sale, Bormaheco returned
the P100,000 to Villonco with 10% interest for the reason that they are not
sure yet if they will acquire the Sta. Ana property. Villonco rejected the
return of the check and demanded for specific performance.

Issue:
Whether or not Bormaheco is bound to perform the contract with Villonco.

Ruling:
The contract is already consummated when Bormaheco accepted the offer
by Villonco. The acceptance can be proven when Bormaheco accepted the
check from Villonco and then returned it with 10% interest as stipulated in
the terms made by Villonco.
On the other hand, the fact that Villonco did not object when
Bormahecoencashed the check is a proof that it accepted the offer of
Bormaheco.
Whenever earnest money is given in a contract of sale, it shall be considered
as part of the price and as proof of the perfection of the contract (Art. 1482,
Civil Code).
21. Zayco vs Zerra, 44 Phil 326 (1923)
Facts:
Zayco and Serra executed a contract for an option to buy Palma
Central for 1M but no stipulation was made as to how much the first
payment would be and when it should be paid. Zayco wrote to Serra
accepting the contract tendering P100,000 as his first payment before the
option period expired. Serra wrote to Zayco stating that the option contract
of November 7, 1918, was cancelled and annulled. On the same day, Zayco
brought suit against Serra to compel him to execute the deed of sale and
conveyance of the Palma Central and Estate and to pay, in addition,
P500,000 as damages. Serra demurred on the ground that the contract does
not specify the part of the price that was to be paid in cash and the part that
was to be paid within a period not exceeding three years. Zayco later
learned that Serra had already sold the property to Whitaker and Concepcion
for 1.5M.
Issue:
Whether or not there was a perfected contract of sale.
Ruling:
The contract was not valid as an option because there was no
consideration, but it was at least an offer to sell. When plaintiff tendered
P100,000. as part payment, acceptance involved a proposal, not contained
in the offer. This proposal, in turn, required the acceptance by Serra. The
failure of Serra to accept (for Serra cancelled the offer) prevented the
perfection of the contract.
Consent is manifested by the meeting of the offer and the acceptance
upon the thing and the cause which are to constitute the contract.
22. Cronico vs JM Tuason, GR L-35272 (1977)
Facts:
JM Tuason was the registered owner of Lot 22. Florencio Cronico
offered to buy the lot from JM Tuason with the help of Mary Venturanza.
Cronico was required to present proofs of her rights to the lot, and indeed
presented certain documents showing her priority rights to buy the lot.
Claudio Ramirez also learned that said lot was being sold. Both Cronico and
Ramirez then sent individual letters to JM Tuason expressing their desire to
purchase the land and requested information concerning the area, the price,
and other terms and conditions of the contract to sell. JM Tuason sent
separate reply letters to the prospective buyers. Cronico was able to obtain
the letter the next day and thus presented the letter to the Head of the Real
Estate Department of JM Tuason; and requested Venturanza to issue a check
as down payment, but the same was refused. Ramirez, on the other hand,
received the letter two days after it was sent stating that the lot was
available for sale under the conditions set forth and that said lot was being
offered for sale on a first come first serve basis. He then immediately
verbally accepted such, followed by a letter to JM Tuason confirming the
verbal acceptance, the next day. Counsel of Ramirez then wrote JM Tuason
for the early execution of the Contract to Sell with a check as down payment
(Mar 31). Counsel of Cronico, however, also wrote JM Tuason requesting
that the lot be sold to him (Mar 27). Subsequently, JM Tuason and Ramirez
executed a Contract to Sell, which resulted an instant suit.
Arguments:
Cronico: That  the promise to sell is supported by a consideration as to her
because she had established her link as successor of Gregorio Venturanza
who bought the lot from Juan Ramos who in turn acquired said lot from
Pedro Deudor.
JM Tuason: As ruled by the CA, the records do not show that JM Tuason’s
letter-offer or unilateral promise to sell was supported by a consideration
other than the selling price.
Issue:
Whether or not JM Tuason’s promise to sell the lot to Cronico has a
consideration separate from the selling price of said lot and thus binding
upon the promissory to comply with such promise.
Ruling:
No, the promise of the respondent company to sell the lot in question
to the petitioner, Florencia Cronico has no consideration separate from the
selling price of said lot. It appears that the Compromise Agreement upon
which Cronico predicates her right to buy the lot in question has been
rescinded and set aside.
(1) In order that a unilateral promise may be binding upon the promisor,
Article 1479, Civil Code of the Philippines, requires the concurrence of the
condition that the promise be “supported by a consideration distinct from the
price. Accordingly, the promiseecan not compel the promisor to comply with
the promise, unless the former establishes the existence of said distinct
consideration. The promisee has the burden of proving such consideration.
(Sanchez vs. Rigos, 45 SCRA 368, 372-373) The petitioner, Florencia
Cronies, has not established the existence of a consideration distinct from
the price of the lot in question.
(2) The petitioner cannot claim that she had accepted the promise before it
was withdrawn because she had violated the condition of "first, come, first
served” basis.
(3) It was only on March 27, 1962 that the respondent company received a
letter from counsel of the petitioner requesting that the lot subject of this
litigation be sold to her. The respondent, Claudio R. Ramirez, had on March
23, 1962, confirmed in writing his verbal acceptance of the terms and
conditions of the sale of the lot in question.
23. Carceller vs CA, 302 SCRA 718 (1999)
FACTS:
Respondent State Investment Houses Inc. has a parcel of land
in Cebu City leased to petitioner Jose Ramon Caceller with an option to
purchase valid until the expiration of the lease contract.
3weeks before the expiration of the contract, petitioner made a request to
the respondent for the extension of the lease contact so he can have an
ample time to raise enough funds to avail of the option of sale.
Respondent denied the request and a month after the expiration of the
contract, petitioner made known his intention to buy the property.
Respondent reiterated the provisions in the contract and asked the petitioner
to leave the property, which will now be offered to the general public for a
higher price.
Issue:
Whether or not can still exercise his option of sale even after the time
to do such has already lapsed.
Ruling:
The appealed decision of respondent court, insofar as it affirms the
judgment of the trial court in granting petitioner the opportunity to exercise
the option to purchase the subject property, is hereby AFFIRMED. However
the purchase price should be based on the fair market value of real property
in Bulacao, Cebu City, as of February 1986, when the contract would have
been consummated.
The contract must be interpreted together with the intention of the
parties. The letter of the plaintiff to the respondent requesting for an
extension is sufficient proof of his intent to avail of the option of sale.
In contractual relations, the law allows the parties reasonable leeway
on the terms of their agreement, which is the law between them. When
petitioner made his intention to buy known to the buyer one month after the
expiration of contract is within a reasonable time- frame.
Petitioner may buy the property but not anymore to the price stated in
the contract. As such, respondent may increase the price of the land but
only to a reasonable and fair market value.
An option is a preparatory contract in which one party grants to the
other, for a fixed period and under specified conditions, the power to decide,
whether or not to enter into a principal contract. It binds the party who has
given the option, not to enter into the principal contract with any other
person during the period designated, and, within that period, to enter into
such contract with the one to whom the option was granted, if the latter
should decide to use the option. It is a separate agreement distinct from the
contract which the parties may enter into upon the consummation of the
option.
24. Villamor vs CA, 202 SCRA 607 (1991)
Facts:
MacariaLabing-isa Reyes was the owner of a 600-square meter lot.
Macaria sold a portion of 300 square meters of the lot to the Spouses Julio
and Marina Villamor for the total amount of P21,000.00.
Earlier, Macaria borrowed P2,000.00 from the spouses which amount
was deducted from the total purchase price of the 300 square... meter lot
sold.
The portion sold to the Villamor spouses is now covered by TCT No.
39935 while the remaining portion which is still in the name of
MacariaLabing-isa is covered by TCT No. 39934 (pars. 5 and 7, Complaint).
Macaria executed a"Deed of Option" in favor of Villamor in which the
remaining 300 square meter portion (TCT No. 39934) of the lot would be
sold to Villamor under the conditions stated therein.  The document reads:
"This Deed of Option, entered into in the City of Manila, Philippines, this 11th
day of November, 1971, by and between MacariaLabing-isa, of age, married
to Roberto Reyes, "That, I MacariaLabingisa, am the owner in fee simple of a
parcel of land with an area of 600 square meters, more or less, more
particularly described in TCT No. (18431) 18938 of the Office of the Register
of Deeds
"That I, with the conformity of my husband, Roberto Reyes, have sold one-
half thereof to the aforesaid spouses Julio Villamor and Marina V. Villamor at
the price of P70.00 per sq. meter, which was greatly higher than the actual
reasonable prevailing value of lands in that place... at the time, which
portion, after segregation, is now covered by TCT No. 39935 of the Register
of Deeds for the City of Caloocan, issued on August 17, 1971 in the name of
the aforementioned spouses vendees;
"That the only reason why the Spouses-vendees Julio Villamor and Marina V.
Villamor, agreed to buy the said one-half portion at the above-stated price of
about P70.00 per square meter, is because I, and my husband Roberto
Reyes, have agreed to sell and convey to them the... remaining one-half
portion still owned by me and now covered by TCT No. 39935 of the Register
of Deeds for the City of Caloocan, whenever the need of such sale arises
"That I am willing to have this contract to sell inscribed on my aforesaid title
as an encumbrance upon the property covered thereby, upon payment of
the corresponding fees; and
According to Macaria, when her husband, Roberto Reyes, retired in 1984,
they offered to repurchase the lot sold by them to the Villamor spouses but
Marina Villamor refused and reminded them instead that the Deed of Option
in fact gave them the option to purchase the remaining... portion of the lot.
The Villamors, on the other hand, claimed that they had expressed their
desire to purchase the remaining 300 square meter portion of the lot but the
Reyeses had been ignoring them.
Thus, on July 13, 1987, after conciliation proceedings... they... filed a
complaint for specific performance against the Reyeses.
judgment was rendered by the trial court in favor of the Villamor spouses,...
Not satisfied with the decision of the trial court, the Reyes spouses appealed
to the Court of Appeals
On February 12, 1991, the Court of Appeals rendered a decision reversing
the decision of the trial court and dismissing the complaint.  The reversal of
the trial court's decision was premised on the finding of respondent court
that the Deed of Option is void for lack of... consideration.
The Villamor spouses brought the instant petition for review
The court a quo, ruled that the Deed of Option was a valid written
agreement between the parties and made the following conclusions:
The respondent appellate court, however, ruled that the said deed of option
is void for lack of consideration
"Plaintiff-appellees say they agreed to pay P70.00 per square meter for the
portion purchased by them although the prevailing price at that time was
only P25.00 in consideration of the option to buy the remainder of the land. 
This does not seem to be the... case.  In the first place, the deed of sale was
never produced by them to prove their claim.
In the second place, if this was really the... condition of the prior sale, we
see no reason why it should be reiterated in the Deed of Option. On the
contrary, the alleged overprice paid by the plaintiff-appellees is given in the
Deed as reason for the desire of the Villamors to acquire the land rather
than as aconsideration for the option given to them
"At all events, the consideration needed to support a unilateral promise to
sell is a distinct one, not something that is as uncertain as P70.00 per
square meter which is allegedly 'greatly higher than the actual prevailing
value of lands.'  A sale must be for a pricecertain (Art. 1458).  For how much
the portion conveyed to the plaintiff-appellees was sold so that the balance
could be considered the consideration for the promise to sell has not been
shown, beyond a mere allegation that it was very much below P70.00 per
squaremeter.
Issues:
Whether or not theCourt of Appeals erred infinding that thedeed
ofoption is voidfor lack ofconsideration;
The pivotal issue to be resolved in this case is the validity of the Deed of
Option whereby the private respondents agreed to sell their lot to petitioners
"whenever the need of such sale arises, either on our part (private
respondents) or on the part of Julio Villamor andMarina Villamor
(petitioners)."
Ruling:
As expressed in Gonzales v. Trinidad, 67 Phil. 682,...
consideration is "the why of the contracts, the essential reason which
moves the contracting parties to enter into the contract."  The cause
or the impelling reason on the part of private respondents in
executing... the deed of option as appearing in the deed itself is the
petitioners' having agreed to buy the 300 square meter portion of
private respondents' land at P70.00 per square meter "which was
greatly higher than the actual reasonable prevailing price."
This cause or... consideration is clear from the deed which stated:
"That the only reason why the spouses-vendees Julio Villamor and Marina V.
Villamor agreed to buy the said one-half portion at the above stated price of
about P70.00 per square meter, is because I, and my husband Roberto
Reyes, have agreed to sell and convey to them... the remaining one-half
portion still owned by me
The respondent appellate court failed to give due consideration to
petitioners' evidence which shows that in 1969 the Villamor spouses bought
an adjacent lot from the brother of MacariaLabing-isa for only P18.00 per
square meter which the private respondents did not... rebut.  Thus,
expressed in terms of money, the consideration for the deed of option is the
difference between the purchase price of the 300 square meter portion of
the lot in 1971... and the prevailing reasonable price of the same lot in
1971.
Ordinarily, an optional contract is a privilege existing in one person, for
which he had paid a consideration and which gives him the right to buy, for
example, certain merchandise or... certain specified property, from another
person, if he chooses, at any time within the agreed period at a fixed price
If We look closely at the "deed of option" signed by the parties, We will
notice that the first part... covered the statement on the sale of the 300
square meter portion of the lot to Spouses Villamor at the price of P 70.00
per square meter "which was higher than the actual reasonable prevailing
value of the lands in that place at that time (of sale)." The second part
stated that... the only reason why the Villamor spouses agreed to buy said
lot at a much higher price is because the vendor (Reyeses) also agreed to
sell to the Villamors the other half-portion of 300 square meters of the land
Had the deed stopped there, there would be no dispute that... the deed is
really an ordinary deed of option granting the Villamors the option to buy the
remaining 300 square meter-half portion of the lot in consideration for their
having agreed to buy the other half of the land for a much higher price.  But,
the "deed of option" went... on and stated that the sale of the other half
would be made "whenever the need of such sale arises, either on our
(Reyeses) part or on the part of the Spouses Julio Villamor and Marina V.
Villamor.
It appears that while the option to buy was granted to the Villamors, the
Reyeses were likewise granted an option to sell. In other words, it was not
only the Villamors who were granted an option to buy for which they paid a
consideration.  The Reyeses as well were granted an option to sell should the
need for such sale on their part arise.
In the instant case, the option offered by private respondents had been
accepted by the petitioner, the promisee, in the same document.  The
acceptance of an offer to sell for a price certain created a bilateral contract
to sell and buy and upon acceptance, the offeree,... ipso facto assumes
obligations of a vendee
However, the Deed of Option did not provide for the period within which the
parties may demand the performance of their respective undertakings in the
instrument.  The parties could not have contemplated that the delivery of
the property and the payment thereof could be... made indefinitely and
render uncertain the status of the land.  The failure of either parties to
demand performance of the obligation of the other for an unreasonable
length of time renders the contract ineffective.
Under Article 1144 (1) of the Civil Code, actions upon a written contract
must be brought within ten (10) years.  The Deed of Option was executed on
November 11, 1971.  The acceptance, as already mentioned, was also
accepted in the same instrument.  The complaint... in this case was filed by
the petitioners on July 13, 1987, seventeen (17) years from the time of the
execution of the contract. Hence, the right of action had prescribed.
There were allegations by the petitioners that they demanded from the
private respondents as early as
1984 the enforcement of their rights under the contract. Still, it was beyond
the ten (10) year period prescribed by the Civil Code.
It is of judicial notice that the price of real estate in Metro Manila is
continuously on the rise.  To allow the petitioner to demand the delivery of
the property subject of this case thirteen (13) years or seventeen (17) years
after the execution of the deed at the price... of only P70.00 per square
meter is inequitous.
25. Tanay Recreation v. Fausto, 455 SCRA 436 (2005)
Facts:
Petitioner Tanay Recreation Center and Development Corp. (TRCDC) is the
lessee of a 3,090-square meter property located in SitioGayas, Tanay, Rizal,
owned by Catalina MatienzoFausto, under a Contract of Lease executed on
August 1, 1971.  On this property stands the Tanay Coliseum Cockpit
operated by petitioner.  The lease contract provided for a 20-year term,
subject to renewal within sixty days prior to its expiration.  The contract also
provided that should Fausto decide to sell the property, petitioner shall have
the “priority right” to purchase the same.
On June 17, 1991, petitioner wrote Fausto informing her of its intention to
renew the lease. However, it was Fausto’s daughter, respondent Anunciacion
F. Pacunayen, who replied, asking that petitioner remove the improvements
built thereon, as she is now the absolute owner of the property.[4] It
appears that Fausto had earlier sold the property to Pacunayen on August 8,
1990, for the sum of P10,000.00 under a
“KasulatanngBilihanPatuluyanngLupa,” and title has already been transferred
in her name under Transfer Certificate of Title (TCT) No. M-35468.
Despite efforts, the matter was not resolved.  Hence, on September 4, 1991,
petitioner filed an Amended Complaint for Annulment of Deed of Sale,
Specific Performance with Damages, and Injunction, docketed as Civil Case
No. 372-M.
In her Answer, respondent claimed that petitioner is estopped from assailing
the validity of the deed of sale as the latter acknowledged her ownership
when it merely asked for a renewal of the lease.  According to respondent,
when they met to discuss the matter, petitioner did not demand for the
exercise of its option to purchase the property, and it even asked for grace
period to vacate the premises.
After trial on the merits, the Regional Trial Court of Morong, Rizal (Branch
78), rendered judgment extending the period of the lease for another seven
years from August 1, 1991 at a monthly rental of P10,000.00, and dismissed
petitioner’s claim for damages.
On appeal, docketed as CA-G.R. CV No. 43770, the Court of Appeals (CA)
affirmed with modifications the trial court’s judgment per its Decision dated
June 14, 1999.
The CA also resolved that TRCDC VACATE the leased premises immediately:
To GRANT the motion of Pacunayen to allow her to withdraw the
amount of P320,000.00, deposited according to records, with this
court.
To order TRCDC to MAKE THE NECESSARY ACCOUNTING regarding the
amounts it had already deposited (for unpaid rentals for the extended
period of seven [7] years of the contract of lease).  In case it had not
yet completed its deposit, to immediately pay the remaining balance to
Pacunayen.
To order TRCDC to PAY the amount of P10,000.00 as monthly rental,
with regard to its continued stay in the leased premises even after the
expiration of the extended period of seven (7) years, computed from
August 1, 1998, until it finally vacates therefrom.
Petitioner filed a motion for reconsideration but it was denied per Resolution
dated September 14, 1999.
Dissatisfied, petitioner elevated the case to this Court on petition for review
on certiorari.
Issue:
Whether or not the right to purchase expressly provided under a contract of
lease only applies if the lessor decides to sell his property to strangers.
Ruling:
ART. 1311.  Contracts take effect only between the parties, their assigns
and heirs, except in case where the rights and obligations arising from the
contract are not transmissible by their nature, or by stipulation or by
provision of law.  The heir is not liable beyond the value of the property he
received from the decedent.
The general rule is that heirs are bound by contracts entered into by their
predecessors-in-interest.
Exception:When the rights and obligations arising therefrom are not
transmissible by –
(1) their nature,
(2) stipulation or
(3) provision of law.
The lease contract between petitioner and Fausto is a property right, which
is a right that passed on to respondent and the other heirs, if any, upon the
death of Fausto.
WHEREFORE, the instant Petition for Review is PARTIALLY GRANTED. The
Court of Appeals’ Decision dated June 14, 1999 in CA-G.R. CV No. 43770 is
MODIFIED as follows:
(1) The “KasulatanngBilihanPatuluyanngLupa” dated August 8, 1990
between Catalina MatienzoFausto and respondent
AnunciacionFaustoPacunayen is hereby deemed rescinded;
(2) The Heirs of the deceased Catalina MatienzoFausto who are hereby
deemed substituted as respondents, represented by respondent
AnunciacionFaustoPacunayen, are ORDERED to recognize the obligation of
Catalina MatienzoFausto under the Contract of Lease with respect to the
priority right of petitioner Tanay Recreation Center and Development Corp.
to purchase the subject property under reasonable terms and conditions;
(3) Transfer Certificate of Title No. M-35468 shall remain in the name of
respondent AnunciacionFaustoPacunayen, which shall be cancelled in the
event petitioner successfully purchases the subject property;
(4) Respondent is ORDERED to pay petitioner Tanay Recreation Center and
Development Corporation the amount of Twenty Thousand Pesos
(P20,000.00) as actual damages, plus interest thereon at the legal rate of
six percent (6%) per annum from the filing of the Complaint until the finality
of this Decision.  After this Decision becomes final and executory, the
applicable rate shall be twelve percent (12%) per annum until its
satisfaction; and,
(5) Respondent is ORDERED to pay petitioner the amount of Ten Thousand
Pesos (P10,000.00) as attorney’s fees, and to pay the costs of suit.
(6) Let the case be remanded to the Regional Trial Court, Morong, Rizal
(Branch 78) for further proceedings on the determination of the “reasonable
terms and conditions” of the offer to sell by respondents to petitioner,
without prejudice to possible mediation between the parties.
The rest of the unaffected dispositive portion of the Court of Appeals’
Decision is AFFIRMED.
26. San Miguel Properties v. Huang, 336 SCRA 737 (2000)
Facts:
This is a petition for review of the decision, dated April 8, 1997, of the Court
of Appeals which reversed the decision of the Regional Trial Court, Branch
153, Pasig City dismissing the complaint brought by respondents against
petitioner for enforcement of a contract of sale.
Petitioner San Miguel Properties Philippines, Inc. engaged in the purchase
and sale of real properties. Parts of its inventory are two parcels of land
totalling 1, 738 square meters.On February 21, 1994, the properties were
offered for sale for P52,140,000.00 in cash made to Atty. Helena M. Dauz,
acting for respondent spouses as undisclosed principals. In a letter dated
March 24, 1994, Atty. Dauz signified her clients’ interest in purchasing the
properties for the said amount under the following terms: the sum of
P500,000.00 would be given as earnest money and the balance would be
paid in eight equal monthly installments from May to December, 1994.
However, petitioner refused the counter-offer.
On March 29, 1994, Atty. Dauz wrote another letter the following terms for
the purchase of the properties:
This is to express our interest to buy your-above-mentioned property
with an area of 1, 738 sq. meters. For this purpose, we are enclosing
herewith the sum of P1,000,000.00 representing earnest-deposit
money, subject to the following conditions.
1. We will be given the exclusive option to purchase the property within
the 30 days from date of your acceptance of this offer.
2. During said period, we will negotiate on the terms and conditions of
the purchase; SMPPI will secure the necessary Management and Board
approvals; and we initiate the documentation if there is mutual
agreement between us.
3. In the event that we do not come to an agreement on this
transaction, the said amount of P1,000,000.00 shall be refundable to us
in full upon demand. . . .
Isidro A. Sobrecarey, petitioner’s vice-president, indicated his conformity to
the offer and accepted the "earnest-deposit" of P1 million, upon request of
respondent spouses.
Atty. Dauz and Sobrecarey then commenced negotiations. Sobrecarey
informed Atty. Dauz that petitioner was willing to sell the subject properties
on a 90-day term. Atty. Dauz countered with an offer of six months within
which to pay. In another meeting the parties again met during which
Sobrecarey informed Atty. Dauz that petitioner had not yet acted on her
counter-offer. This prompted Atty. Dauz to propose a four-month period of
amortization.
Atty. Dauz asked for an extension of 45 days from April 29, 1994 to June 13,
1994, within which to exercise her option to purchase the property.On July
7, 1994, petitioner, wrote Atty. Dauz informing her that because the parties
failed to agree on the terms and conditions of the sale despite the extension
granted by petitioner, the latter was returning the amount of P1 million
given as "earnest-deposit."
On July 20, 1994, respondent spouses, wrote petitioner demanding the
execution within five days of a deed of sale covering the properties.
Respondents attempted to return the "earnest-deposit" but petitioner
refused on the ground that respondents’ option to purchase had already
expired.
Issue:
Whether or not the amount given constituted a perfected contract of sale
Ruling:
No. The amount given was not a part of the purchase price and proof of the
perfection of the contract of sale but only as a guarantee that respondents
would not back out of the sale. With regard to the alleged payment and
acceptance of earnest money, the Court holds that respondents did not give
the P1 million as “earnest money” as provided by Article 1482 of the Civil
Code. They presented the amount merely as a deposit of what would
eventually become the earnest money or down payment should a contract of
sale be made by them. The amount was thus given not as a part of the
purchase price and as proof of the perfection of the contract of sale but only
as a guarantee that respondents would not back out of the sale.
Respondents in fact described the amount as an “earnest-deposit.”
Option giving respondents the exclusive right to buy the properties within
the period agreed upon is separate and distinct from the contract of sale
which the parties may enter. The first condition for an option period of 30
days sufficiently shows that a sale was never perfected. As petitioner
correctly points out, acceptance of this condition did not give rise to a
perfected sale but merely to an option or an accepted unilateral promise on
the part of respondents to buy the subject properties within 30 days from
the date of acceptance of the offer. Such option giving respondents the
exclusive right to buy the properties within the period agreed upon is
separate and distinct from the contract of sale which the parties may enter.
All that respondents had was just the option to buy the properties which
privilege was not, however, exercised by them because there was a failure
to agree on the terms of payment. No contract of sale may thus be enforced
by respondents.
Option secured by respondents from petitioner was fatally defective;
Consideration in an option contract may be anything of value, unlike in sale
where it must be the price certain in money or its equivalent.—Even the
option secured by respondents from petitioner was fatally defective. Under
the second paragraph of Art. 1479, an accepted unilateral promise to buy or
sell a determinate thing for a price certain is binding upon the promisor only
if the promise is supported by a distinct consideration. Consideration in an
option contract may be anything of value, unlike in sale where it must be the
price certain in money or its equivalent. There is no showing here of any
consideration for the option. Lacking any proof of such consideration, the
option is unenforceable.
The manner of payment of the purchase price is an essential element before
a valid and binding contract of sale can exist.—The appellate court opined
that the failure to agree on the terms of payment was no bar to the
perfection of the sale because Art. 1475 only requires agreement by the
parties as to the price of the object. This is error. In Navarro v. Sugar
Producers Cooperative Marketing Association, Inc., we laid down the rule
that the manner of payment of the purchase price is an essential element
before a valid and binding contract of sale can exist. Although the Civil Code
does not expressly state that the minds of the parties must also meet on the
terms or manner of payment of the price, the same is needed, otherwise
there is no sale. As held in Toyota Shaw, Inc. v. Court of Appeals,
agreement on the manner of payment goes into the price such that a
disagreement on the manner of payment is tantamount to a failure to agree
on the price.
It is not the giving of earnest money, but the proof of the concurrence of all
the essential elements of the contract of sale which establishes the existence
of a perfected sale. It is not the giving of earnest money, but the proof of
the concurrence of all the essential elements of the contract of sale which
establishes the existence of a perfected sale.
27. Secuya v. vda de Selma, 326 SCRA 244 (2000)
Facts:
Before the grant of her application for private sale of Lot 5679, a friar land,
Maxima Caballero executed a document entitled "Agreement of Partition,"
wherein she stipulated to transfer one-third (1/3) of the lot to and accepted
by Paciencia Sabellano, her aunt. When the application was approved,
Maxima failed to transfer the agreed portion to Paciencia who took
possession thereof. Paciencia thereafter sold the same to Dalmacio Secuya.
The document was, however, lost. When Paciencia died, her only heir,
Ramon Sabellano, executed a private document, entitled "Deed of
Confirmation of Sale," confirming the sale between Paciencia and Dalmacio.
Meanwhile, Maxima sold the entire lot to Silverio Aro, husband of Cesaria
Caballero. Upon Silverio's death, the lot was transferred to Cesaria from
whom respondent bought the lot. Respondent was assured that petitioners
who were occupying a portion of the land were tenants. A clean title to the
whole lot was transferred to respondent. Petitioners, heirs of Dalmacio
Secuya, filed an action for quieting of title on the ground that respondent's
title is a cloud on their title as owners and possessors of the property subject
of litigation. They claimed that they had been occupying the property for
forty-seven years though they did not pay the land taxes. The trial court
rendered judgment against respondent. It was affirmed, on appeal, by the
Court of Appeals.
Issue:

1. Whether or not there was a valid transfer or conveyance of one-third (1/3)


portion of Lot 5679 by Maxima Caballero in favor of Paciencia Sabellona, by
virtue of [the] Agreement of Partition

2. Whether Gerarda M. vda. de Selma was a buyer in bad faith with respect to
the land, which is a portion of Lot 5679

Ruling:

The agreement is that of An Express Trust, Not a Partition

Notwithstanding its purported nomenclature, this Agreement is not one of


partition, because there was no property to partition and the parties were
not co-owners. Rather, it is in the nature of a trust agreement.

The present Agreement of Partition involves an express trust. Under Article


1444 of the Civil Code, "[n]o particular words are required for the creation of
an express trust, it being sufficient that a trust is clearly intended." That
Maxima Caballero bound herself to give one third of Lot No. 5629 to
Paciencia Sabellona upon the approval of the former's application is clear
from the terms of the Agreement. Likewise, it is evident that Paciencia
agreed to the covenant and is thus bound to fulfill her obligation therein.

As a result of the Agreement, Maxima Caballero held the portion specified


therein as belonging to Paciencia Sabellona when the application was
eventually approved and a sale certificate was issued in her name. 15 Thus,
she should have transferred the same to the latter, but she never did so
during her lifetime. Instead, her heirs sold the entire Lot No. 5679 to
Silvestre Aro in 1955..

Second Issue:
The Validity of Private Respondent's Title
No she is not a buyer in bad faith.

Petitioners debunk Private Respondent Selma's title to the disputed property,


alleging that she was aware of their possession of the disputed properties.
Thus, they insist that she could not be regarded as a purchaser in good faith
who is entitled to the protection of the Torrens system.

Indeed, a party who has actual knowledge of facts and circumstances that
would move a reasonably cautious man to make an inquiry will not be
protected by the Torrens system.

In Sandoval v. Court of Appeals,22 we held:

It is settled doctrine that one who deals with property registered under
the Torrens system need not go beyond the same, but only has to rely
on the title. He is charged with notice only of such burdens and claims
as are annotated on the title.

The aforesaid principle admits of an unchallenged exception: that a


person dealing with registered land has a right to rely on the Torrens
certificate of title and to dispense without the need of inquiring further
except when the party has actual knowledge of facts and
circumstances that would impel a reasonably cautious man to make
such inquiry, or when the purchaser has knowledge of a defect or the
lack of title in his vendor or of sufficient facts to induce a reasonably
prudent man to inquire into the status of title of the property in
litigation. The presence of anything which excites or arouses suspicion
should then prompt the vendee to look beyond the certificate and
investigate the title of the vendor appearing on the face of the
certificate. One who falls within the exception can neither be
denominated an innocent purchaser for value purchaser in good faith;
and hence does not merit the protection of the law.

Granting arguendo that private respondent knew that petitioners, through


Superales and his family, were actually occupying the disputed lot, we must
stress that the vendor, Cesaria Caballero, assured her that petitioners were
just tenants on the said lot. Private respondent cannot be faulted for
believing this representation, considering that petitioners' claim was not
noted in the certificate of the title covering Lot No. 5679.

Moreover, the lot, including the disputed portion, had been the subject of
several sales transactions. The title thereto had been transferred several
times, without any protestation or complaint from the petitioners. In any
case, private respondent's title is amply supported by clear evidence, while
petitioners' claim is barren of proof.

Clearly, petitioners do not have the requisite title to pursue an action for quieting of
title

WHEREFORE, the Petition is hereby DENIED and the assailed Decision


AFFIRMED. Costs against petitioners.
28. Claudel v. CA, 199 SCRA 113 (1991)
Facts:
As early as December 28, 1922, Basilio also known as "Cecilio" Claudel,
acquired from the Bureau of Lands, Lot No. 1230 of the Muntinlupa Estate
Subdivision, located in the poblacion of Muntinlupa, Rizal, with an area of
10,107 square meters, he secured Transfer Certificate of Title (TCT) No.
7471 issued by the Registry of Deeds for the Province of Rizal in 1923, he
also declared the lot in his name, the latest Tax Declaration being No. 5795.
He dutifully paid the real estate taxes thereon until his death in 1937.
Thereafter, his widow "Basilia" and later, her son Jose, one of the herein
petitioners, paid the taxes. The same piece of land purchased by Cecilio
would, however, become the subject of protracted litigation thirty-nine years
after his death. Two branches of Cecilio's family contested the ownership
over the land-on one hand the children of Cecilio, now the herein petitioners
(hereinafter referred to as the HEIRS OF CECILIO) and the other, the
brother and sisters of Cecilio and their children and descendants, now the
herein private respondents (hereinafter referred to as SIBLINGS OF
CECILIO). In 1972, the HEIRS OF CECILIO partitioned this lot among them
and obtained the corresponding Transfer Certificates of Title on their shares.
Four years later, on December 7, 1976, private respondents SIBLINGS OF
CECILIO, filed a complaint for Cancellation of Titles and Reconveyance with
Damages," alleging that 46 years earlier, or sometime in 1930, their parents
had purchased from the late Cecilio Claudel several portions of Lot No. 1230
for the sum of P30.00. They admitted that the transaction was verbal.
However, as proof of the sale, the SIBLINGS OF CECILIO presented a
subdivision plan of the said land, dated March 25, 1930, indicating the
portions allegedly sold to the SIBLINGS OF CECILIO.
Issues:
1. Whether or not a contract of sale of land may be proven orally.
2. Whether or not the prescriptive period for filing an action for cancellation
of titles and reconveyance with damages (the action filed by the SIBLINGS
OF CECILIO) should be counted from the alleged sale upon which they claim
their ownership (1930) or from the date of the issuance of the titles sought
to be cancelled in favor of the HEIRS OF CECILIO (1976).

Ruling:
The rule of thumb is that a sale of land, once consummated, is valid
regardless of the form it may have been entered into. For nowhere does law
or jurisprudence prescribe that the contract of sale be put in writing before
such contract can validly cede or transmit rights over a certain real property
between the parties themselves.
However, in the event that a third party, as in this case, disputes the
ownership of the property, the person against whom the claim is brought
cannot present any proof of such sale and hence has no means to enforce
the contract. Thus the Statute of Frauds was precisely devised to protect the
parties in a contract of sale of real property so that no such contract is
enforceable unless certain requisites, for purposes of proof, are met.
The provisions of the Statute of Frauds pertinent to the present controversy,
state:
Art. 1403 (Civil Code). The following contracts are unenforceable,
unless they are ratified:
2) Those that do not comply with the Statute of Frauds as set forth in
this number. In the following cases, an agreement hereafter made
shall be unenforceable by action unless the same, or some note or
memorandum thereof, be in writing, and subscribed by the party
charged, or by his agent, evidence, therefore, of the agreement
cannot be received without the writing, or a secondary evidence of its
contents.
e) An agreement for the leasing for a longer period than one
year, or for the sale of real property or of an interest therein.
The purpose of the Statute of Frauds is to prevent fraud and
perjury in the enforcement of obligations depending for their
evidence upon the unassisted memory of witnesses by requiring
certain enumerated contracts and transactions to be evidenced
in Writing.
Therefore, except under the conditions provided by the Statute of Frauds,
the existence of the contract of sale made by Cecilio with his siblings cannot
be proved.
On the second issue, the belated claim of the SIBLINGS OF CECILIO who
filed a complaint in court only in 1976 to enforce a light acquired allegedly as
early as 1930, is difficult to comprehend. The Civil Code states that the
following actions must be commenced within six years. If the parties
SIBLINGS OF CECILIO had allegedly derived their right of action from the
oral purchase made by their parents in 1930, then the action filed in 1976
would have clearly prescribed. More than six years had lapsed.
29. Pichel vs Alonzo, GR No. L-36902, January 30, 1982

Facts:
This case originated in the lower Court as an action for the annulment of a
"Deed of Sale" dated August 14, 1968 and executed by Prudencio Alonzo, as
vendor, in favor of Luis Pichel, as vendee, involving property awarded to the
former by the Philippine Government under Republic Act No. 477 (REPUBLIC
ACT NO. 477 AN ACT TO PROVIDE FOR THE ADMINISTRATION AND
DISPOSITION OF PROPERTIES, INCLUDING THE PROCEEDS AND INCOME
THEREOF TRANSFERRED TO THE REPUBLIC OF THE PHILIPPINES, UNDER
THE PHILIPPINE PROPERTY ACT OF 1946 AND OF REPUBLIC ACT NO. EIGHT,
AND OF THE PUBLIC LANDS AND IMPROVEMENTS THEREON TRANSFERRED
TO THE NATIONAL ABACA AND OTHER FIBERS CORPORATION UNDER THE
PROVISIONS OF EXECUTIVE ORDER NO. 29 DATED OCTOBER 25, 1946 AND
OF EXECUTIVE ORDER NO. 99 DATED OCTOBER 22, 1947).
Plaintiff Prudencio Alonzo was awarded by the Government that parcel of
land designated as Lot No. 21 of Subdivision Plan Psd 32465 of Balactasan,
Lamitan, Basilan City in accordance with Republic Act No. 477. The award
was cancelled by the Board of Liquidators on January 27, 1965 on the
ground that, previous thereto, plaintiff was proved to have alienated the
land to another, in violation of law. In 197 2, plaintiff's rights to the land
were reinstated.
On August 14, 1968, plaintiff and his wife sold to defendant an the fruits of
the coconut trees which may be harvested in the land in question for the
period, September 15, 1968 to January 1, 1976, in consideration of
P4,200.00. Even as of the date of sale, however, the land was still under
lease to one, Ramon Sua, and it was the agreement that part of the
consideration of the sale, in the sum of P3,650.00, was to be paid by
defendant directly to Ramon Sua so as to release the land from the clutches
of the latter. Pending said payment plaintiff refused to snow the defendant
to make any harvest.
In July 1972, defendant for the first time since the execution of the deed of
sale in his favor, caused the harvest of the fruit of the coconut trees in the
land.
Lower court: Holding that although the agreement in question is
denominated by the parties as a deed of sale of fruits of the coconut trees
found in the vendor's land, it actually is, for all legal intents and purposes, a
contract of lease of the land itself Deed of sale is hereby declared as null and
void.

ISSUES:
1. WON the document, Deed of Sale of herein parties one of sale of
coconut fruits of Lot 21 and not a lease of land as found by the Lower Court?
2. WON the Deed of Sale, as such contract of sale, is valid under the
provisions of Sec. 8, RA No. 477?

RULING:
1. Yes. It is a sale of coconut fruits.
In clear and express terms, the document defines the object of the contract
thus: "the herein sale of the coconut fruits are for an the fruits on the
aforementioned parcel of land during the years ...(from) SEPTEMBER 15,
1968; up to JANUARY 1, 1976." Moreover, as petitioner correctly asserts,
the document in question expresses a valid contract of sale. It has the
essential elements of a contract of sale as defined under Article 1485 of the
New Civil Code which provides thus:
Art. 1458. By the contract of sale one of the contracting parties obligates
himself to transfer the ownership of and to deliver a determinate thing, and
the other to pay therefor a price certain in money or its equivalent.
A contract of sale may be absolute or conditional.
The subject matter of the contract of sale in question are the fruits of the
coconut trees on the land during the years from September 15, 1968 up to
January 1, 1976, which subject matter is a determinate thing. Under Article
1461 of the New Civil Code, things having a potential existence may be the
object of the contract of sale.
We do not agree with the trial court that the contract executed by and
between the parties is "actually a contract of lease of the land and the
coconut trees there." (CFI Decision, p. 62, Records). The Court's holding that
the contract in question fits the definition of a lease of things wherein one of
the parties binds himself to give to another the enjoyment or use of a thing
for a price certain and for a period which may be definite or indefinite (Art.
1643, Civil Code of the Philippines) is erroneous. The essential difference
between a contract of sale and a lease of things is that the delivery of the
thing sold transfers ownership, while in lease no such transfer of ownership
results as the rights of the lessee are limited to the use and enjoyment of
the thing leased.
The contract was clearly a "sale of the coconut fruits." The vendor sold,
transferred and conveyed "by way of absolute sale, all the coconut fruits of
his land," thereby divesting himself of all ownership or dominion over the
fruits during the seven-year period. The possession and enjoyment of the
coconut trees cannot be said to be the possession and enjoyment of the land
itself because these rights are distinct and separate from each other, the
first pertaining to the accessory or improvements (coconut trees) while the
second, to the principal (the land). A transfer of the accessory or
improvement is not a transfer of the principal. It is the other way around,
the accessory follows the principal. Hence, the sale of the nuts cannot be
interpreted nor construed to be a lease of the trees, much less extended
further to include the lease of the land itself.

2. Yes.
We find after a close and careful examination of the terms of the first
paragraph of Section 8 hereinabove quoted, that the grantee of a parcel of
land under R.A. No. 477 is not prohibited from alienating or disposing of the
natural and/or industrial fruits of the land awarded to him. What the law
expressly disallows is the encumbrance or alienation of the land itself or any
of the permanent improvements thereon. Permanent improvements on a
parcel of land are things incorporated or attached to the property in a fixed
manner, naturally or artificially. They include whatever is built, planted or
sown on the land which is characterized by fixity, immutability or
immovability. Houses, buildings, machinery, animal houses, trees and plants
would fall under the category of permanent improvements, the alienation or
encumbrance of which is prohibited by R.A. No. 477. While coconut trees are
permanent improvements of a land, their nuts are natural or industrial fruits
which are meant to be gathered or severed from the trees, to be used,
enjoyed, sold or otherwise disposed of by the owner of the land. Herein
respondents, as the grantee of Lot No. 21 from the Government, had the
right and prerogative to sell the coconut fruits of the trees growing on the
property.
The purpose of the law is not violated when a grantee sells the produce or
fruits of his land. On the contrary, the aim of the law is thereby achieved, for
the grantee is encouraged and induced to be more industrious and
productive, thus making it possible for him and his family to be economically
self-sufficient and to lead a respectable life. At the same time, the
Government is assured of payment on the annual installments on the land.
We agree with herein petitioner that it could not have been the intention of
the legislature to prohibit the grantee from selling the natural and industrial
fruits of his land, for otherwise, it would lead to an absurd situation wherein
the grantee would not be able to receive and enjoy the fruits of the property
in the real and complete sense.
30. Melliza v. City of Iloilo, G.R. No. L-24732, April 30, 1968

FACTS:

Juliana Melliza owned 3 parcels of residential land in Iloilo City (OCT 3462).
Said parcels of land were known as Lots Nos. 2, 5 and 1214. The total area
of Lot 1214 was 29,073 sq. m. She donated to the then Municipality of Iloilo
to serve as site for the municipal hall. The donation was revoked by the
parties for the reason that the area donated was not enough to development
plan of the municipality, the so- called “Arellano Plan.”

Certeza Surveying Co., Inc. divided subsequently Lot 1214 into four parts
named; Lot 1214A, Lot 1214B, Lot 1214C, and Lot1214D. Juliana Melliza
executed an instrument without any caption providing for the absolute sale
of all of lot 5, in favor of the Municipal Government of Iloilo for the sum of
P6,422Melliza later sold her remaining interest in Lot 1214 to Remedios Sian
Villanueva which later transferred to Pio Sian Melliza. Annotated at the back
of Pio Sian Melliza’s title certificate was the following ‘that a portion of
10,788 sq. m. of Lot 1214 now designated as Lots 1412-B-2 and 1214-B-3
of the subdivision plan belongs to the Municipality of Iloilo as per instrument
dated November 15 1932’, On 24 August 1949 the City of Iloilo, which
succeeded to the Arellano Plan, donated the city hall site together with the
building thereon, to the University of the Philippines (Iloilo branch).

In 1952, the University of the Philippines enclosed the site donated with a
wire fence. Pio Sian Melliza thereupon made representations, through his
lawyer, with the city authorities for payment of the value of the lot (Lot
1214-B). No recovery was obtained, because the City allegedly did not have
funds. The University of the Philippines, meanwhile, obtained Transfer
Certificate of Title No. 7152 covering the three lots, Nos. 1214-B, 1214-C
and 1214-D.

Pio Sian Melizza filed an action in the Court of First Instance of Iloilo against
Iloilo City and the University of the Philippines for recovery of Lot 1214-B or
of its value. After stipulation of facts and trial, the Court of First Instance
decided on dismissing the complaint. Court ruled that the title certificate
Juliana Melliza presented was in favor to the respondent, included in the
conveyance Lot 1214-B, for that Iloilo City had the right to donate Lot 1214-
B to UP. Pio Sian Melliza appealed to the Court of Appeals.

The Court of Appeals affirmed the interpretation of the Court of First


Instance that the portion of Lot 1214 sold by Juliana Melliza was not limited
to the 10,788 square meters specifically mentioned but included whatever
was needed for the construction of avenues, parks and the city hall site.
Nonetheless, it ordered the remand of the case for reception of evidence to
determine the area actually taken by Iloilo City for the construction of
avenues, parks and for city hall site.
The Court affirmed the decision appealed from insofar as it affirms that of
the Court of First Instance, and dismissed the complaint.

ISSUE:

Whether or not the conveyance by Juliana Melliza to Iloilo municipality


included that portion of Lot 1214 known as Lot 1214-B.

RULING:

First of all, there is no question that the paramount intention of the parties
was to provide Iloilo municipality with lots sufficient or adequate in area for
the construction of the Iloilo City hall site, with its avenues and parks.

Secondly, reading the public instrument in toto, with special reference to the
paragraphs describing the lots included in the sale, shows that said
instrument describes four parcels of land by their lot numbers and area; and
then it goes on to further describe, not only those lots already mentioned,
but the lots object of the sale, by stating that said lots are the ones needed
for the construction of the city hall site, avenues and parks according to the
Arellano plan. If the parties intended merely to cover the specified lots —
Lots 2, 5, 1214-C and 1214-D, there would scarcely have been any need for
the next paragraph, since these lots are already plainly and very clearly
described by their respective lot number and area. Said next paragraph does
not really add to the clear description that was already given to them in the
previous one.

The requirement of the law that a sale must have for its object a
determinate thing, is fulfilled as long as, at the time the contract is entered
into, the object of the sale is capable of being made determinate without the
necessity of a new or further agreement between the parties. The specific
mention of some of the lots plus the statement that the lots object of the
sale are the ones needed for city hall site, avenues and parks, according to
the Arellano plan, sufficiently provides a basis, as of the time of the
execution of the contract, for rendering determinate said lots without the
need of a new and further agreement of the parties.

Appellant however fails to consider that the area needed under that plan for
city hall site was then already known; that the specific mention of some of
the lots covered by the sale in effect fixed the corresponding location of the
city hall site under the plan; that, therefore, considering the said lots
specifically mentioned in the public instrument Exhibit “D”, and the projected
city hall site, with its area, as then shown in the Arellano plan (Exhibit 2), it
could be determined which, and how much of the portions of land contiguous
to those specifically named, were needed for the construction of the city hall
site.
And, moreover, there is no question either that Lot 1214-B is contiguous to
Lots 1214-C and 1214-D, admittedly covered by the public instrument. It is
stipulated that, after execution of the contract , the Municipality of Iloilo
possessed it together with the other lots sold. It sits practically in the heart
of the city hall site.

Furthermore, Pio Sian Melliza, from the stipulation of facts, was the notary
public of the public instrument. Said instrument was also registered with the
Register of Deeds and such registration was annotated at the back of the
corresponding title certificate of Juliana Melliza. From these stipulated facts,
it can be inferred that Pio Sian Melliza knew of the aforesaid terms of the
instrument or is chargeable with knowledge of them; that knowing so, he
should have examined the Arellano plan in relation to the public instrument
Exhibit “D”; that, furthermore, he should have taken notice of the
possession first by the Municipality of Iloilo, then by the City of Iloilo and
later by the University of the Philippines of Lot 1214-B as part of the city hall
site conveyed under that public instrument, and raised proper objections
thereto if it was his position that the same was not included in the same.
The fact remains that, instead, for twenty long years, Pio Sian Melliza and
his predecessors-in-interest, did not object to said possession, nor exercise
any act of possession over Lot 1214-B. Applying, therefore, principles of civil
law, as well as laches, estoppel, and equity, said lot must necessarily be
deemed included in the conveyance in favor of Iloilo municipality, now Iloilo
City.

WHEREFORE, the decision appealed from is affirmed insofar as it affirms that


of the Court of First Instance, and the complaint in this case is dismissed.
31. Yturralde vs. CA, G.R. No. L-31586, February 28, 1972
FACTS:
Spouses Francisco Yturralde and Margarita de los Reyes, owned a parcel of
agricultural land located in Guilinan, Tungawan, Zamboanga del Sur,
containing an area of 14.1079 hectares, more or less, and registered in their
names under Original Certificate of Title No. 2356 of the Office of the
Register of Deeds of Zamboanga del Sur. Sometime in the year 1944,
Francisco Yturralde died intestate, survived by his wife, Margarita de los
Reyes, and their children who are the petitioners herein, Ernesto, Fortunata,
Montano, Zosimo, Ramon, Guadalupe, Luis, Josefina and Rosalia, all
surnamed Yturralde. In 1950, Margarita de los Reyes contracted a second
marriage with her brother-in-law and uncle of the petitioners herein,
DamasoYturralde .
On May 30, 1952, DamasoYturralde and Margarita de los Reyes executed a
deed of sale with right of repurchase in favor of the respondent herein,
IsabeloRebollos, covering the above-mentioned property in consideration of
the sum of P1,715.00. The vendors a retro failed to exercise the right to
repurchase the property within the three-year period agreed upon, which
expired on May 30, 1955. In 1961, Margarita de los Reyes died.
On May 3, 1965, the respondent, IsabeloRebollos, filed a petition for
consolidation of ownership with the Court of First Instance of Zamboanga del
Sur, docketed as Civil Case No. 436 therein, naming as respondents in the
case the petitioners herein and DamasoYturralde (Annex A, Petition).
Summons was then issued, and received on June 17, 1965 by the
respondent therein, Damaso, Ernesto, Fortunata, Montano, Guadalupe, Luis
and Rosalia, all surnamed Yturralde (Annexes C and F, Petition). However,
summons could not be served on three of the respondents therein, Josefina,
Zosima and Ramon Yturralde, as they were no longer residing at their last
known addresses (Annexes B, C and F, Petition). The Judge then presiding
the Court of First Instance of Zamboanga del Sur, Hon. DimalanesBuissan, in
his order dated October 7, 1965, directed that summons be served upon the
said three respondents therein (Annex C, Petition). On November 20, 1965,
the Court rendered a decision consolidating the ownership of the subject
property in favor of Rebollos, and ordering the Register of Deeds of
Zamboanga del Sur to cancel Original Certificate of Title No. 2356 covering
said property and, in lieu thereof, to issue a transfer certificate of title in the
name of Rebollos (Annex H, Petition).

Issue:
Whether the requirements for consolidation of ownership by vendee a
retro had been complied with?
Held:
Unlike the old Civil Code, Article 1607 of the new Civil Code of 1950 provides
that consolidation of ownership in the vendee a retro of real property by
virtue of the failure of the vendor a retro "to comply with the provisions of
Article 1616 shall not be recorded in the Registry of Property without a
judicial order, after the vendor has been duly heard." In the case of Teodoro
vs. Arcenas, 1 this Court, through Mr. Justice Jose B. L. Reyes, ruled that
under the aforesaid Article 1607 of the new Civil Code, such consolidation
shall be effected through an ordinary civil action, not by a mere motion, and
that the vendor a retro should be made a party defendant, who should be
served with summons in accordance with Rule 14 of the Revised Rules of
Court; and that the failure on the part of the court to cause the service of
summons as prescribed in Rule 14, is sufficient cause for attacking the
validity of the judgment and subsequent orders on jurisdictional grounds. 2
The Court in said case stressed that the reason behind the requirement of a
judicial order for consolidation as directed by Article 1067 of the new Civil
Code is because "experience has demonstrated too often that many sales
with right of re-purchase have been devised to circumvent or ignore our
usury laws and for this reason, the law looks upon them with disfavor
(Report of the Code Commission, pp. 63-64). When, therefore, Article 1607
speaks of a judicial order after the vendor shall have been duly heard, it
contemplates none other than a regular court proceeding under the
governing Rules of Court, wherein the parties are given full opportunity to
lay bare before the court the real covenant. Furthermore, the obvious intent
of our Civil Code, in requiring a judicial confirmation of the consolidation in
the vendee a retro of the ownership over the property sold, is not only to
have all doubts over the true nature of the transaction speedily ascertained,
and decided, but also to prevent the interposition of buyers in good faith
while such determination is being made. Under the former method of
consolidation by a mere extrajudicial affidavit of the buyer a retro, the latter
could easily cut off any claims of the seller by disposing of the property,
after such consolidation, to strangers in good faith and without notice. The
chances of the seller a retro to recover his property would thus be nullified,
even if the transaction were really proved to be a mortgage and not a sale."
32. Gaite v. Fonacier, 2 SCRA 831 (1961), July 31, 1961

FACTS:
Defendant decided to revoke the authority granted by him to Plaintiff to
exploit and develop the mining claims, and plaintiff assented thereto subject
to certain conditions. As a result, a document was executed wherein Plaintiff
transferred to Defendant, for the consideration of P20,000.00, plus 10% of
the royalties that Defendant would receive from the mining claims, all his
rights and interests on all the roads, improvements, and facilities in or
outside said claims, the right to use the business name “Larap Iron Mines”
and its goodwill, and all the records and documents relative to the mines. In
the same document, plaintiff transferred to Defendant all his rights and
interests over the "24,000 tons of iron ore, more or less" that the former
had already extracted from the mineral claims, in consideration of the sum
of P75,000.00, P10,000.00 of which was paid upon the signing of the
agreement, and the balance of SIXTY-FIVE THOUSAND PESOS (P65,000.00)
will be paid from and out of the first letter of credit covering the first
shipment of iron ores and of the first amount derived from the local sale of
iron ore. To secure the payment of the said balance of P65,000.00,
Defendant promised to execute in favor of Plaintiff a surety bond, and
pursuant to the promise. In the subsequent year, Defendant failed to renew
the bond to sureties. Plaintiff, the latter filed the present complaint against
them in the Court of First Instance of Manila (Civil Case No. 29310) for the
payment of the P65,000.00 balance of the price of the ore, consequential
damages, and attorney’s fees.

ISSUE:
Whether or not there were insufficient tons of ores.

HELD:
No. The sale between the parties is a sale of a specific mass or iron ore
because no provision was made in their contract for the measuring or
weighing of the ore sold in order to complete or perfect the sale, nor was the
price of P75,000,00 agreed upon by the parties based upon any such
measurement.(see Art. 1480, second par., New Civil Code). The subject
matter of the sale is, therefore, a determinate object, the mass, and not the
actual number of units or tons contained therein, so that all that was
required of the seller Gaite was to deliver in good faith to his buyer all of the
ore found in the mass, notwithstanding that the quantity delivered is less
than the amount estimated by them (Mobile Machinery & Supply Co., Inc.
vs. York Oilfield Salvage Co., Inc. 171 So. 872, applying art. 2459 of the
Louisiana Civil Code). There is no charge in this case that Gaite did not
deliver to appellants all the ore found in the stockpiles in the mining claims
in questions; Gaite had, therefore, complied with his promise to deliver, and
appellants in turn are bound to pay the lump price.
33. Laroza v. Guia, GR# L-45252, January 31, 1985

FACTS:
1966 CFI Laguna’s Decision in Civil Case SP-488: DonaldoGuia argued
he is the co-owner of the land involved in the present case; Francisco Guia
(the one who sold the lands to Petitioner Laroza) argued that he is the sole
owner of the land. Francisco lost; Donaldo was adjudged as the owner of the
lands.
Years after, Petitioner Laroza insists that he has been in continuous
possession of the land since their purchase of the same in good faith and for
valuable considerations from Francisco. However, Donaldo intruded upon the
peaceful possession by attempting to survey the land and partition the
same, pursuant to Civil Case SP-488.
Donaldo, through counsel, filed a motion to dismiss the complaint
alleging, among others, "that the land subject matter of the complaint has
already been the subject of a final and executory judgment in Civil Case No.
SP-488”.
Laroza filed an action to quiet title against Donaldo regarding such
parcel of land in San Pablo at CFI Laguna. Donaldo filed a motion to dismiss
stating that Laroza has no cause of action. He argues that the land in
question had already been subject of a final and executory judgement in
Civil Case SP-488. CFI Laguna dismissed the complaint of Laroza based on
res judicata.
Laroza appealed to CA. CA forwarded the case to the SC because “no
factual issue is involved” and the “issues raised are purely legal questions
beyond its jurisdiction”
Hence, this petition.

ISSUE:
Whether or not the land is a valid subject matter in the contract of sale
entered into by Laroza and FrancisoGuia.

HELD:
NO. The land is not a valid subject matter in the contract of sale
entered into by Laroza and Francisco Guia.Records show that long before
appellants had acquired subject property, a notice of lispendens (Civil Case
No. SP-488) had already been registered with the Office of the Register of
Deeds of San Pablo City affecting the property. Lis pendens is a notice of
pending litigation; a warning to the whole world that one who buys the
property so annotated does so at his own risk (Rehabilitation Finance
Corporation vs. Morales, 101 Phil. 175). Notwithstanding, appellants bought
the land from Francisco Guia, defendant in Civil Case No. SP-488. Having
purchased the property with notice of lispendens, appellants took the risk of
losing it in case the decision in the said civil case, as what actually
happened, is adverse to their predecessor-in-interest, Francisco Guia. Time
and again, We have decreed that the filing of a notice of lispendens charges
all strangers with a notice of the particular litigation referred to therein and,
therefore, any right they may thereafter acquired on the property is subject
to the eventuality of the suit. The doctrine of lispendens is founded upon
reason of public policy and necessity, the purpose of which is to keep the
subject matter of the litigation within the power of the Court until the
judgment or decree shall have been entered; otherwise, by successive
alienations pending the litigation, its judgment or decree shall be rendered
abortive and impossible of execution. On this score alone, appellants case
would necessarily fall.
SC finds that in his motion to dismiss, appellee had thoroughly discussed the
issue of res judicata and, coupled by the fact that it was the same court
which heard and decided Civil Case No. SP-488, the trial court can rightfully
rule on said issue.
Petition is dismissed.
34. Bagnas v. CA, 176 SCRA 159 (1989), August 10, 1989

FACTS:
HilarioMateum died on March 11, 1964, single, without ascendants or
descendants, and survived only by petitioners who are his collateral
relatives. He left no will, no debts, and an estate consisting of 29 parcels of
land in Kawit and Imus, 10 of which are involved in this controversy. On
April 3, 1964, respondents who are also collateral relatives of the deceased,
but more remote, registered 2 deeds of sale purportedly executed by
Mateum in their favor. The considerations were P1.00 and “services
rendered, being rendered, and to be rendered for my benefit”. On the
strength of the deeds, respondents were able to secure title over the 10
parcels of land. On May 22, 1964, petitioners commenced a suit against
respondents, seeking annulment of the deeds of sale a fictitious, fraudulent
or falsified or, alternatively, as donations void for want of acceptance in
public instrument. Respondents contend that the sales were made for
valuable considerations, and attacked the legal standing of the petitioners as
being mere collateral heirs.

ISSUES:
Whether or not the consideration that is stipulated in the two deeds of
sale is adequate to render the sale valid.

HELD:
No. The consideration that is stipulated in the two deeds of sale is
inadequate to render the sale valid. Upon the consideration alone that the
apparent gross, not to say enormous, disproportion between the stipulated
price (in each deed) of P1.00 plus unspecified and unquantified services and
the undisputably valuable real estate allegedly sold worth at least
P10,500.00 going only by assessments for tax purposes which, it is well-
known, are notoriously low indicators of actual value plainly and
unquestionably demonstrates that they state a false and fictitious
consideration, and no other true and lawful cause having been shown, the
Court finds both said deeds, insofar as they purport to be sales, not merely
voidable, but void ab initio. Neither can the validity of said conveyances be
defended on the theory that their true causa is the liberality of the transferor
and they may be considered in reality donations because the law also
prescribes that donations of immovable property, to be valid, must be made
and accepted in a public instrument, and it is not denied by the respondents
that there has been no such acceptance which they claim is not required.
The transfers in question being void, it follows as a necessary consequence
and conformably to the concurring opinion in Armentia, with which the Court
fully agrees, that the properties purportedly conveyed remained part of the
estate of HilarioMateum, said transfers notwithstanding, recoverable by his
intestate heirs, the petitioners herein, whose status as such is not
challenged.
The questioned transfers are declared void and of no force or effect.
Such certificates of title as the private respondents may have obtained over
the properties subject of said transfers are hereby annulled, and said
respondents are ordered to return to the petitioners’ possession of an the
properties involved in tills action, to account to the petitioners for the fruits
thereof during the period of their possession, and to pay the costs. No
damages, attorney's fees or litigation expenses are awarded, there being no
evidence thereof before the Court.
SO ORDERED.

35. Vda. De Gordon v. CA, 109 SCRA 88 (1981), November 23, 1981
FACTS:
Petitioner 80-year-old RestitutaVda. De Gordon owned 2 parcels of
land in Quezon City with an assessed value of P16,800, while the residential
house thereon is assessed at 45,580.
From 1953 to 1963, taxes against said parcels of land remained
unpaid. Thus QC Treasurer held a public sale to satisfy the taxes.
On December 3, 1964, the properties were sold to Respondent Duazo
for P10,500 representing the tax, penalty and costs.
When Gordon failed to redeem said parcels of land within the 1-year
period prescribed by law, QC Treasurer executed on January 4, 1966 a final
deed of sale of said lands and the improvements thereon in favor of Duazo.
Gordon now opposed Duazo’s petition for consolidation of ownership,
which CA dismissed.
Hence this petition.

ISSUE:
Whether or not the price is so grossly inadequate as to justify the
setting aside of the public sale.

HELD:
No. The price is not grossly inadequate as to justify the setting aside of
the public sale.
While the price of P10,500.00 is less than the total assessed value of
the land and the improvement thereon, said price cannot be considered so
grossly inadequate as to be shocking to the conscience of the court.
In the case at bar, the price of P10,500.00 is about one sixth of the
total assessed value of the two parcels of land in question and the residential
house thereon. The finding of the lower court that the house and land in
question have a fair market value of not less than P200,000.00 has no
factual basis. It cannot be said, therefore, that the price of P10,500.00 is so
inadequate as to be shocking to the conscience of the court.
Mere inadequacy of the price alone is not sufficient ground to annul
the public sale. (Barrozo vs. Macaraeg, 83 Phil. 378).
As the Court has held in Velasquez vs. Coronet alleged gross
inadequacy of price is not material "when the law gives the owner the right
to redeem as when a sale is made at public auction, upon the theory that
the lesser the price the easier it is for the owner to effect the redemption."
As the Court further stressed in the recent case of Tajonera vs. Court of
Appeals, the law governing tax sales for delinquent taxes may be "harsh and
drastic, but it is a necessary means of insuring the prompt collection of taxes
so essential to the life of the Government."
ACCORDINGLY, the appellate court's decision under review is hereby
affirmed. Without costs.
36. IMELDA ONG, ET AL. vs. ALFREDO ONG, ET AL., 139 SCRA 133,
G.R. No. L-67888, October 8, 1985

Facts:

On February 25, 1976 Imelda Ong, for and in consideration of One (P1.00)
Peso and other valuable considerations, executed in favor of private
respondent Sandra Maruzzo, then a minor, a Quitclaim Deed whereby she
transferred, released, assigned and forever quit-claimed to Sandra Maruzzo,
her heirs and assigns, all her rights, title, interest and participation in the ½
undivided portion of the parcel of land. But Imelda Ong revoked the
aforesaid Deed of Quitclaim and, thereafter, on January 20, 1982 donated
the whole property described above to her son, Rex Ong-Jimenez.

On June 20, 1983, Sandra Maruzzo, through her guardian (ad litem) Alfredo
Ong, filed with the Regional Trial Court of Makati, Metro Manila an action
against petitioners, for the recovery of ownership/possession and
nullification of the Deed of Donation over the portion belonging to her and
for Accounting.

In their responsive pleading, petitioners claimed that the Quitclaim Deed is


null and void inasmuch as it is equivalent to a Deed of Donation, acceptance
of which by the donee is necessary to give it validity. Further, it is averred
that the donee, Sandra Maruzzo, being a minor, had no legal personality and
therefore incapable of accepting the donation.

The trial court rendered judgment in favor of respondent Maruzzo and held
that the Quitclaim Deed is equivalent to a Deed of Sale and, hence, there
was a valid conveyance in favor of the latter. The Intermediate Appellate
Court affirmed the appealed judgment and held that the Quitclaim Deed is a
conveyance of property with a valid cause or consideration; that the
consideration is the One (P1.00) Peso which is clearly stated in the deed
itself; that the apparent inadequacy is of no moment since it is the usual
practice in deeds of conveyance to place a nominal amount although there is
a more valuable consideration given.

Issue:

Whether or not the quitclaim is equivalent to a deed of sale or to a deed of


donation

Ruling:

The Quitclaim is equivalent to a Deed of Sale.


A careful perusal of the subject deed reveals that the conveyance of the one-
half (½) undivided portion of the above-described property was for and in
consideration of the One (P 1.00) Peso and the other valuable considerations
(emphasis supplied) paid by private respondent Sandra Maruzzo through her
representative, Alfredo Ong, to petitioner Imelda Ong. Stated differently, the
cause or consideration is not the One (P1.00) Peso alone but also the other
valuable considerations. As aptly stated by the Appellate Court-

... although the cause is not stated in the contract it is presumed that
it is existing unless the debtor proves the contrary (Article 1354 of the
Civil Code). One of the disputable presumptions is that there is a
sufficient cause of the contract (Section 5, (r), Rule 131, Rules of
Court). It is a legal presumption of sufficient cause or consideration
supporting a contract even if such cause is not stated therein (Article
1354, New Civil Code of the Philippines.) This presumption cannot be
overcome by a simple assertion of lack of consideration especially
when the contract itself states that consideration was given, and the
same has been reduced into a public instrument with all due
formalities and solemnities. To overcome the presumption of
consideration the alleged lack of consideration must be shown by
preponderance of evidence in a proper action. (Samanilla vs, Cajucom,
et al., 107 Phil. 432).

The execution of a deed purporting to convey ownership of a realty is in


itself prima facie evidence of the existence of a valuable consideration, the
party alleging lack of consideration has the burden of proving such
allegation. (Caballero, et al. vs. Caballero, et al., (CA), 45 O.G. 2536).

WHEREFORE, the appealed decision of the Intermediate Appellate Court


should be, as it is hereby AFFIRMED, with costs against herein petitioners.
37. SPOUSES SALVACION SERRANO LADANGA and AGUSTIN S.
LADANGA vs. COURT OF APPEALS and BERNARDO S. ASENETA, as
Guardian of the Incompetent CLEMENCIA A. ASENETA, 131 SCRA
361, G.R. No. L-55999, August 24, 1984

Facts:

Clemencia, a spinster who retired as division superintendent of public


schools at 65 in 1961, had a nephew named Bernardo S. Aseneta, the child
of her sister Gloria, and a niece named Salvacion, the daughter of her sister
Flora. She legally adopted Bernardo in 1961. When Clemencia was about 78
years old, she signed nine deeds of sale in favor of Salvacion for various real
properties. One deed of sale concerned the said Paco property (administered
by the Ladanga spouses) which purportedly was sold to Salvacion for
P26,000. The total price involved in the nine deeds of sale and in the tenth
sale executed on November 8, 1974 was P92,200.

On the witness stand, Clemencia denied having "received even one centavo"
of the price of P26,000 much less the P92,000. This testimony was
corroborated by Soledad L. Maninang, 69, a dentist with whom Clemencia
had lived for more than thirty years in Kamuning, Quezon City. The notary
testified that the deed of sale for the Paco property was signed in the office
of the Quezon City registry of deeds. He did not see Salvacion giving any
money to Clemencia.

In May, 1975, Bernardo as guardian of Clemencia, filed an action for


reconveyance of the Paco property, accounting of the rentals and damages.
Clemencia was not mentally incompetent but she was placed under
guardianship because she was an easy prey for exploitation and deceit.

The trial court and the Appellate Court declared void the sale of the Paco
property. The Ladanga spouses argue that the Appellate Court erred in not
considering that inadequacy of price may indicate a donation or some other
contract; in disregarding the presumption that the sale was fair and regular
and for a sufficient consideration; in overlooking important facts and in not
holding that Bernardo had no right to file a complaint to annul the sale.

Issue:

Whether or not the sale is void for lack of consideration.

Ruling:

A contract of sale is void and produces no effect whatsoever where the price,
which appears therein as paid, has in fact never been paid by the purchaser
to the vendor (MenesesVda. de Catindig vs. Heirs of Catalina Roque, L-
25777, November 26, 1976, 74 SCRA 83, 88; Mapalo vs. Mapalo, 123 Phil.
979, 987; Syllabus, Ocejo, Perez & Co. vs. Flores and Bas, 40 Phil. 921).
Such a sale is inexistent and cannot be considered consummated (Borromeo
vs. Borromeo, 98 Phil. 432; Cruzado vs. Bustos and Escaler, 34 Phil. 17;
Garanciang vs. Garanciang, L-22351, May 21, 1969, 28 SCRA 229).

It was not shown that Clemencia intended to donate the Paco property to
the Ladangas. Her testimony and the notary's testimony destroyed any
presumption that the sale was fair and regular and for a true consideration.
Judge Colayco concluded that the Ladangas abused Clemencia's confidence
and defrauded her of properties with a market value of P393,559.25 when
she was already 78 years old. The contention that Bernardo had no right to
institute the instant action because he was not a compulsory heir of
Clemenciacannot be sustained. Bernardo was Clemencia's adopted son.
Moreover, Clemencia, by testifying in this case, tacitly approved the action
brought in her behalf.

Moral damages awarded by the trial court is not sanctioned by Art. 2217-
2220 of the Civil Code. Clemencia's own signature in the deed brought about
the mess within which she was entangled.

WHEREFORE, the judgment of the Appellate Court is affirmed with the


modification that the adjudication for moral and exemplary damages is
discarded. No costs.
38. REPUBLIC OF THE PHILIPPINES vs. PHILIPPINE RESOURCES
DEVELOPMENT CORPORATION and the COURT OF APPEALS, 102 Phil.
960, G.R. No. L-10141, January 31, 1958

Facts:

The Bureau of Prisons instituted a complaint against Macario Apostol for the
latter’s failure to pay the unpaid balance for logs purchased. Apostol, who
was then the president of the respondent corporation, delivered goods
belonging to the corporation and without the knowledge or consent of the
stockholders thereof, to the Bureau of Prisons in an attempt to settle his
personal debts with the latter entity. The corporation demanded the Bureau
of Prisons for the return of the goods. Upon the refusal of the Bureau, the
corporation filed a motion to intervene.

Issue:

Is the “price” limited only to be paid in money?

Ruling:

No. Article 1458 of the New Civil Code provides that the purchaser may pay
“a price certain in money or its equivalent,” which means that they meant of
the price need not be in money. In this case, the materials have been
assessed and evaluated and their price equivalent in terms of money have
been determined and that said materials for whatever price they have been
assigned were considered as tokens of payment.
39. TOYOTA SHAW, INC. vs. COURT OF APPEALS and LUNA L. SOSA,
24 SCRA 320, G.R. No. L-116650, May 23, 1995

Facts:

Private respondent Luna L. Sosa wanted to purchase a Toyota Lite Ace. With
his his son, Gilbert, he went to the Toyota office at Shaw Boulevard, Pasig
and met Popong Bernardo, a sales representative of Toyota. Sosa
emphasized to Bernardo that he needed the Lite Ace not later than 17 June
1989. Bernardo assured him that a unit would be ready for pick up at 10:00
a.m. on that date. They contracted an agreement on the delivery of the unit
and that the balance of the purchase price would be paid by credit financing
through B.A. Finance. The next day, Sosa and Gilbert delivered the
downpayment and met Bernardo who then accomplished a printed Vehicle
Sales Proposal (VSP) in which the amount was filled-up but the spaces
provided for “Delivery Terms” were not filled-up. However, on 17 June 1989,
at 9:30 am, Bernardo called Gilbert to inform him that the car could not be
delivered because “nasulotang unit ng ibangmalakas.”

Toyota contends, on the other hand, that the Lite Ace was not delivered to
Sosa because of the disapproval by B.A. Finance of the credit financing
application of Sosa. Toyota then gave Sosa the option to purchase the unit
by paying the full purchase price in cash but Sosa refused. Sosa asked that
his down payment be refunded. Toyota did so on the very same day by
issuing a Far East Bank check for the full amount, which Sosa signed with
the reservation, “without prejudice to our future claims for damages.”
Thereafter, Sosa sent two letters to Toyota. In the first letter, he demanded
the refund of the down payment plus interest from the time he paid it. The
second, he demanded one million pesos representing interest and damages,
both with a warning that legal action would be taken if payment not paid.
Toyota’s refused to accede to the demands of Sosa. The latter filed with RTC
a complaint against Toyota for damages under Articles 19 and 21 of the Civil
Code. In its answer to the complaint, Toyota alleged that no sale was
entered into between it and Sosa, that Bernardo had no authority to sign for
and in its behalf. It alleged that the VSP did not state the date of delivery.

Issue:

Was there a perfected contract of sale?

Ruling:

There was no perfected contract of sale. What is clear from the agreement
signed by Sosa and Gilbert is not a contract of sale. No obligation on the part
of Toyota to transfer ownership of a determinate thing to Sosa and no
correlative obligation on the part of the latter to pay therefore a price certain
appears therein. The provision on the down payment of PIOO,OOO.OO made
no specific reference to a sale of a vehicle. If it was intended for a contract
of sale, it could only refer to a sale on installment basis, as the VSP executed
the following day con finned. Nothing was mentioned about the full purchase
price and the manner the installments were to be paid. A definite agreement
on the manner of payment of the price is an essential element in the
formation of a binding and enforceable contract of sale. This is so because
the agreement as to the manner of payment goes, into the price such that a
disagreement on the manner of payment is tantamount to a failure to agree
on the price. Definiteness as to the price is an essential element of a binding
agreement to sell personal property.
40. CEBU WINDLAND DEV. CORP. V. ONG SIAO HUA, 588 SCRA 120
(2009)

FACTS:
Hua bought 2 condominium units from Cebu Winland Development
Corporation. The area per condominium unit as indicated in the price list is
155 square meters and the price per square meter is P22,378.95. On
October 10, 1996, possession of the subject properties was turned over to
Hua.
After the purchase price was fully paid on January 31, 1997, Cebu Winland
sent to Hua Deedsof Absolute Sale for the two condominium units for
signature. Upon examination of the deed of absolute sale, Hua was
distressed to find that the stated floor area is only 127 square meters
contrary to the area indicated in the price list which was 155 square meters.
Hua caused a verification survey of the said condominium units and
discovered that the actual area is only 110 square meters per unit. Hua
demanded from Cebu Winland to refund the amount of P2,014,105.50
representing excess payments for the difference in the area,computedas
follows: 155 sq.m.-110 = 45 x 2 units = 90 sq.m. x P22,378.95 =
P2,014,105.50.

Cebu Winland refused to refund the said amount. Consequently, Hua filed a
Complaint on August 7, 1998 in the Regional Office of the Housing and Land
Use Regulatory Board (HLURB) in Cebu City. The Arbiterruled that Hua's
action had already prescribed pursuant to Article 1543, in relation to Articles
1539 and 1542, of the Civil Code. Hua appealed.
Cebu Winland argues that it delivered possession of the subject properties to
Hua on October 10, 1996, hence, Hua's action filed on August 7, 1998 has
already prescribed.
Hua, on the one hand, contends that his action has not prescribed because
the prescriptive period has not begun to run as the same must be reckoned
from the execution of the deeds of sale which has not yet been done.

ISSUE/S:

1. Whether Hua's action has prescribed.


2. Whether the sale in the case is one made with a statement of its area or
at the rate of a certain price for a unit of measure and not for a lump sum.
HELD:
1. The action has not yet prescribed.
The pertinent provisions of the Civil Code on the obligation of the vendor to
deliver theobject of the sale provide: Art. 1495. The vendor is bound to
transfer the ownership of and deliver, as well as warrant the thing which is
the object of the sale. Art. 1496. The ownership of the thing sold is acquired
by the vendee from the moment it is delivered to him in any of the ways
specified in Articles 1497 to 1501, or in any other manner signifying an
agreement that the possession is transferred from the vendor to the vendee.
Art. 1497. The thing sold shall be understood as delivered, when it is placed
in the control and possession of the vendee.
Delivery is an act by which one party parts with the title to and the
possession of the property, and the other acquires the right to and the
possession of the same. In its natural sense, delivery means something in
addition to the delivery of property or title; it means transfer of possession.
“Delivery” as used in the Law on Sales refers to the concurrent transfer of
two things: (1) possession and

(2) ownership. This is the rationale behind the jurisprudential doctrine that
presumptive delivery via execution of a public instrument is negated by the
reality that the vendee actually failed to obtain material possession of the
land subject of the sale. In the same vein, if the vendee is placed in actual
possession of the property, but by agreement of the parties ownership of the
same is retained by the vendor until the vendee has fully paid the price, the
mere transfer of the possession of the property subject of the sale is not the
“delivery” contemplated in the Law on Sales or as used in Article 1543 of the
Civil Code.

In the case at bar, it appears that Cebu Windland was already placed in
possession of the subject properties. However, it iscrystal clear that the
deeds of absolute sale were still to be executed by the parties upon payment
of the last installment. This fact shows that ownership of the said properties
was withheld by petitioner. Following case law, it is evident that the parties
did not intend to immediately transfer ownership of the subject properties
until full payment and the execution of the deeds of absolute sale.
Consequently, there is no “delivery” to speak of in this case since what was
transferred was possession only and not ownership of the subject properties.

The transfer of possession of the subject properties on October 10, 1996 to


Hua cannot be considered as “delivery” within the purview of Article 1543 of
the Civil Code. It follows that since there has been no transfer of ownership
of the subject properties since the deeds of absolute sale have not yet been
executed by the parties, the action filed by Hua has not prescribed.

2. Article 1539 provides that “If the sale of real estate should be made with
a statement of its area, at the rate of a certain price for a unit of measure or
number, the vendor shall be obliged to deliver to the vendee…all that may
have been stated in the contract; but, should this be not possible, the
vendee may choose between a proportional reduction of the price and the
rescission of the contract….” Article 1542, on the one hand, provides that
“In the sale of real estate, made for a lump sum and not at the rate of a
certain sum for a unit of measure or number, there shall be no increase or
decrease of the price, although there be a greater or lesser area or number
than that stated in the contract."

Article 1539 governs a sale of immovable by the unit, that is, at a stated
rate per unit area. In a unit price contract, the statement of area of
immovable is not conclusive and the price may be reduced or increased
depending on the area actually delivered. If the vendor delivers less than the
area agreed upon, the vendee may oblige the vendor to deliver all that may
be stated in the contract or demand for the proportionate reduction of the
purchase price if delivery is not possible. If the vendor delivers more than
the area stated in the contract, the vendee has the option to accept only the
amount agreed upon or to accept the whole area, provided he pays for the
additional area at the contract rate.

In some instances, a sale of an immovable may be made for a lump sum


and not at a rate per unit. The parties agree on a stated purchase price for
an immovable the area of which may be declared based on an estimate or
where both the area and boundaries are stated.

In the case where the area of the immovable is stated in the contract based
on an estimate, the actual area delivered may not measure up exactly with
the area stated in the contract. According to Article 1542 of the Civil Code, in
the sale of real estate, made for a lump sum and not at the rate of a certain
sum for a unit of measure or number, there shall be no increase or decrease
of the price although there be a greater or lesser area or number than that
stated in the contract. However, the discrepancy must not be substantial. A
vendee of land, when sold in gross or with the description "more or less"
with reference to its area, does not thereby ipso facto take all risk of
quantity in the land. The use of "more or less" or similar words in
designating quantity covers only a reasonable excess or deficiency.

Where both the area and the boundaries of the immovable are declared, the
area covered within the boundaries of the immovable prevails over the
stated area. In cases of conflict between areas and boundaries, it is the
latter which should prevail. What really defines a piece of ground is not the
area, calculated with more or less certainty, mentioned in its description, but
the boundaries thereinlaid down, as enclosing the landand indicatingits
limits. In a contract of sale of land in a mass, it is well established that the
specific boundaries statedin the contract must control over any statement
with respect to the area contained within its boundaries. It is not of vital
consequence that a deed or contract of sale of land should disclosethe area
with mathematical accuracy. It is sufficient if its extent is objectively
indicatedwith sufficient precision to enable one to identifyit. An error as to
the superficial area is immaterial. Thus, the obligation of the vendor is to
deliver everything within the boundaries, inasmuch asit is the entirety
thereof that distinguishes the determinate object.

In the case at bar, it is undisputed by the parties that the purchase price of
the subject properties was computed based on the price list prepared by
petitioner, or P22,378.95 per square meter. Clearly, the parties agreed on a
sale at a rate of a certain price per unit of measure and not one for a lump
sum. Hence, it is Article 1539 and not Article 1542 which is the applicable
law. Accordingly, respondent is entitled to the relief afforded to him under
Article 1539, that is, either a proportional reduction of the price or the
rescission of the contract, at his option. Respondent chose the former
remedy since he prayed in his Complaint for the refund of the amount of
P2,014,105.50 representing the proportional reduction of the price paid to
petitioner.
41. KUENZLE V. MACKE, 14 PHIL 610

FACTS:
The plaintiff alleges that it was the owner of the Oregon Saloon consisting of
bar, furniture, furnishings, and fixtures in which the Jose Desiderio, as
sheriff, levied upon by virtue of an execution issued upon a judgment
secured by the defendant Macke & Chandler, against Stanley & Krippendorf.
Said plaintiff notified the sheriff that it was the owner of said goods and
forbade the sale thereof under said execution. The sheriff sold said goods
under said execution and the firm of Macke & Chandler was the purchaser of
said goods. Bachrach, Elser, and Gale, were the sureties upon the bond
given to the sheriff by Macke & Chandler before said goods were sold. The
defendants in this case allege that the property described by the plaintiff and
sold at the execution sale referred to was not the property of the plaintiff at
the time of said levy and sale, but was the property of Stanley &
Krippendorf, who were in possession of the same at the time of such levy.
They further allege that Stanley & Krippendorf, being indebted in a
considerable sum to the plaintiff in this case, attempted to sell to the said
plaintiff by an instrument in writing the property in question which was
never recorded and was a private document. The said property was not
delivered to the plaintiff but that property remained from the time of sale
forward in the exclusive possession and control of said Stanley &
Krippendorf, and that they conducted the business.

ISSUE/S:
Whether or not there is an effect in the said instrument of sale in
transferring the property in question from Stanley & Krippendorf to the
plaintiff.

HELD:

No. The ownership of personal propertycan not be transferred to the


prejudice of third persons except by delivery of the property itself; and that
a sale without delivery gives the would-be purchaser no rights in said
property except those of a creditor. The bill of sale in the case at bar could
have no effect against a person dealing with the property upon the faith of
appearances. It is evident that the bill of sale was in no sense a conditional
sale of property. Possession of the property in suit was not taken at any time
by the plaintiff. The defendant Macke &Chandlre, having purchased the
property at an execution sale, property conducted, obtained a good title to
the property in question as against the plaintiff in this case.
42. PRISCA NAVAL, ET AL. vs. FRANCISCO ENRIQUEZ, ET AL.G.R. No.
1318 April 12, 1904

FACTS:
Don Jorge Enriquez, as heir of his deceased parents, Antonio Enriquez and
Doña Ciriaca Villanueva, whose estates were at that time still undistributed,
by public instrument sold to Don Victoriano Reyes his interest in both
estates, equivalent to a tenth part thereof. By another instrument executed,
Don Enrique Barrera, Don Victoriano Reyes sold to Doña Carmen de la
Cavada his interest in the estate of Don Antonio Enriquez and Doña Ciriaca
Villanueva, which he had acquired from Don Jorge Enriquez. The purchaser,
Doña Carmen, was the wife of Don Francisco Enriquez, who was the
executor and administrator of the testamentary estate of Don Antonio
Enriquez at the dates of the execution of the two mentioned. The plaintiffs
demand that these deeds be declared null and void, as well as the contracts
evidenced thereby, apparently solelyso far as they refer to the estate of Don
Antonio Enriquez, no mention being made of the estate of Doña Ciriaca
Villanueva in the complaint. This relief is prayed for upon the grounds that
the deeds in question were consummated and were executed for the
purpose of deceiving and defrauding Don Jorge Enriquez and his family, and
such executor Don Francisco Enriquez was unable to acquire by his own act
or that of any intermediary the said hereditary portion of Don Jorge Enriquez
under the provisions of paragraph 3 of article 1459 of the Civil Code.
ISSUE/S:
Whether or not the deeds executed are null and void.
HELD:

No. The date of those contracts down to the death of Jorge Enriquez, which
occurred July 6, 1891, more than five year had passed and more than fifteen
before the filing of the complaint on January 9, 1902, nothing having been
done in the meantime on the part of the plaintiffs or the person under whom
they claim to interrupt the running of the statute. The action of nullity only
lasts four years, counted from the date of the consummation of the contract,
when the action is based, as in this case, upon the absence of consideration.
(Art. 1301 of the Civil Code.)The contract of sale is consummated by the
delivery of the purchase money and of the thing sold. (Art. 1462, par. 2,
Civil Code.)

Article 1464 provides that "With respect to incorporeal property, the


provisions of par. 2 of Art. 1462 shall govern." In the deeds of sale executed
by Victoriano Reyes in favor of Doña Carmen de la Cavada, in consequence
he (the vendor) by virtue of this title cedes and conveys all rights which he
has or may have to the part of the inheritance which is the object of this sale
may exercise all the acts of ownership corresponding to her right, to which
end by means of the delivery of this instrument and of his other title deeds
he makes the transfer necessary to consummate the contract, which upon
his part he declares to be perfect and consummated from this date.
43. OBANA VS CA 135 SCRA 557 (1985)
FACTS:
Sandoval is the owner and manager of Sandovaland Sons Rice Mill and is
engaged in buying and sellingpalay.Sandoval was approached by a Chan Lin
who offered to purchase 170 cavans of wagwag rice at P37.25per cavan, to
be delivered the following day at Obana’sstore with payment to be made
upon delivery.As agreed, 170 cavans of rice were transported to Obana’s
store the following day; Chan Lin accompanied the shipment. When the
driver tried to collect payment, ChanLin was nowhere to be found. The driver
tried to collect from Obana but he refused stating that he had purchased the
goods from Chan Lin at P33. Sandoval filed a suit for replevin against Obana.
Municipal Trial Court ordered Obana to pay half of the cost of the
rice(P2,805). Obana appealed, CFI held in favor of Obana and dismissed the
complaint.On appeal with CA, CA ruled in favor of Sandoval ordering Obana
to pay full price of the rice at P37.25.

ISSUE/S:
Whether or not Obana is liable to pay Sandoval at 37.25 per cavan.

HELD:

We agree with petitioner-defendant that there was a perfected sale. Article


1475 of the Civil Code lays down the general rule that there is perfection
when there is consent upon the subject matter and price, even if neither is
delivered. Ownership of the rice, too, was transferred to the vendee, Chan
Lin, upon its delivery to him at San Fernando, La Union, the place
stipulated and pursuant to Articles 1477 and 1496 of the same Code.

At the very least, Chan Lin had a rescissible title to the goods for the non-
payment of the purchase price, but which had not been rescinded at the
time of the sale to petitioner.

Having been repaid the purchases price by Chan Lin , the sale, as between
them, had been voluntarily rescinded, and petitioner-defendant was thereby
divested of any claim to the rice. Technically, therefore, he should return the
rice to Chan Lin, but since even the latter, again from petitioner-defendant's
own testimony above-quoted, was ready to return the rice to SANDOVAL,
and the latter's driver denies that the rice had been returned by petitioner-
defendant cannot be allowed to unjustly enrich himself at the expense of
another by holding on to property no longer belonging to him. 7 In law and in
equity, therefore, SANDOVAL is entitled to recover the rice, or the value
theref since hewas not paid the price therefor.
44. Vallarta v CA, G.R. No. L-40195 May 29, 1987

Facts:
Rosalinda Cruz, the private offended party, and accused Victoria Vallarta are
long time friends and business acquaintances
Cruz entrusted to Victoria Vallarta seven pieces of jewelry
Vallarta decided to buy some items, exchanged one item with another, and
issued a post-dated check... the check was dishonored and Cruz was...
informed that Vallarta's account had been closed.
both the trial court and the Court of Appeals found Vallarta guilty beyond
reasonable doubt of the crime of estafa.
petitioner stresses that the transaction between her and Cruz was a "sale or
return," perfected and consummated on November 20, 1968 when the seven
pieces of jewelry were delivered
Thus, even if it was dishonored, petitioner claims that she can only be held
civilly liable, but not criminally liable... the jewelries were entrusted on
November 20, 1968, but the sale was perfected in December 1968, and the
finding that there was deceit in the issuance of the postdated check.
Issues:
whether it was a "sale or return" or some other transaction, it is necessary
to determine the intention of the parties.
Ruling:
WE affirm.
Art. 315 (2)(d) as amended by Rep. Act No. 4885, of the Revised Penal
Code, which penalizes any person who shall defraud another "(b)y
postdating a check, or issuing a check in payment of an obligation when the
offender had no funds in the bank, or... his funds deposited therein were not
sufficient to cover the amount of the check."
(t)he failure of the drawer of the check to deposit the amount necessary to
cover his check within three (3) days from receipt of notice from the bank
and/or the payee or holder that said check has been dishonored for lack or
insufficiency of... funds" is deemed prima facie evidence of deceit
constituting false pretense or fraudulent act.
there was a meeting of the minds between the parties as to the object of...
the contract and the consideration therefore only in December 1968, the
same time that the check was issued. The delivery made on November 20,
1968 was only for the purpose of enabling Vallarta to select what jewelry
she wanted.
transaction entered into by Cruz and Vallarta was not a "sale or return."
Rather, it was a "sale on approval" (also called "sale on acceptance," "sale
on trial," or "sale on satisfaction"... sale or return," the ownership... passes
to the buyer on delivery... no meeting of the minds on November 20, 1968,
then, as of that date, there was yet no contract of sale which could be the
basis of delivery or tradition.
the delivery made on November 20, 1968 was not a delivery for purposes of
transferring ownership the... prestation incumbent on the vendor. If
ownership over the jewelry was not transmitted on that date, then it could
have been transmitted only in December 1968, the date when the check was
issued. In which case, it was a "sale on approval" since ownership passed to
the buyer,... Vallarta, only when she signified her approval or acceptance to
the seller, Cruz, and the price was agreed upon.
No error in the assailed decision of the Court of Appeals
45. Aznar v Yapdiangco, G.R. No. L-18536

Facts:
Teodoro Santos advertised in two metropolitan papers the sale of his FORD
FAIRLANE 500.
a certain L. De Dios, claiming to be a nephew of VicenteMarella, went to the
Santos residence to answer the ad. However, Teodoro Santos was out during
this call and only the latter's son, Irineo Santos received and talked with De
Dios.The latter told the young Santos that he had... come in behalf °f his
uncle, Vicente Marella, who was interested to buy he advertised car.
On being informedTeodoro Santos instructed his son to see the said Vicente
Marella .the following day at his given address. And so,...Irineo Santos went
to the above address.
Marella agreed to buy the car for P14,700.00 on the understanding that the
price would be paid only after the car had been registered in his name.Irineo
Santos then fetched his father who, together with L. De Dios, went to the
office of a certain Atty. Jose Padolina where the deed of sale for the car was
executed in Marella's favor. Registration of the car in Marella's name was
effected.
Up to this stage of the transaction, the purchase price had not been paid.
Teodoro Santos returned to his house. He gave the registration papers and a
copy of the deed of sale to his son, Irineo, and instructed him not to part
with them until Marella shall have given the full... payment for the car.
Irineo Santos... demanded for the payment from Vicente Marella.
Marella said that the... amount he had on hand then was short by some
P2,000.00... and begged off to be allowed to secure the shortage from a
sister supposedly living somewhere in Azcarraga Street also in Manila.
Thereafter, he ordered L. De Dios to go to the said sister and... suggested
that Irineo Santos to go with him. At the same time, he requested for the
registration papers and the deed of sale from Ireneo Santos on the pretext
that he would like to show them to h lawyers. Trusting the good faith of
Marella,... Irene" handed over the same to the latter... lrineo Santos and L.
De Dios alighted from the car and entered a house, while their unidentified
companion remained in the car. Once inside, L. De Dios asked lrineo Santos
to wait at the... sala while he went inside a room. That was the last that
Ireneo saw of him. For, after a considerable length of time waiting in vain for
De Dios to return, Ireneo went down to discover that neither the car nor
their... unidentified companion was there anymore.
That very .same day,... Vicente Marella was able to sell the car in question
to the plaintiff... herein, Jose B. Aznar, for P15,000.00.While the car in
question was thus in the possession of Jose B. Aznar and while he was
attending to its registration in his name, agents of the Philippine
Constabulary seized and confiscated the same in consequence of the report
to them by TeodoroSantos that the said car unlawfully taken from him.
Jose B. Aznar filed a complaint for replevin against Captain Rafael
Yapdiangco, the head of the Philippine Constabulary unit which seized the
car in Question. Claiming ownership of the vehicle, he prayed for its delivery
to... him.
At the end of the trial, the lower court... awarding the disputed motor
vehicle toTeodoro Santosit ruled that Teodoro Santos had been unlawfully
deprived of his personal propertybyVicente Marella, from whom the plaintiff-
appellant traces his right.
From this decision, Jose B. Aznar appeals.
Issues:
Jose B. Aznar, who has a better right to the possession of the disputed
automobile?
Ruling:
In this case, the seller had no title at all.
Under Article 712 of the Civil Code, "ownership and other real rights over
property are acquired and transmitted by law, by donation, by testate and
intestate succession, and in consequence of certain contracts, by tradition."
As interpreted by this Court in a host of cases,... by this provision,
ownership is not transferred by contract merely but by tradition or delivery.
Contracts only constitute titles or rights to the transfer or acquisition of
ownership, while delivery or tradition is the mode of accomplishing the
same.
"So long as property is not delivered, the ownership over it is not transferred
by contract -merely but by delivery. Contracts only constitute titles or lights
to the transfer or acquisition of ownership, while delivery or tradition is the
method of accomplishing the same,... the title and the method of acquiring it
being different in our law." (Gonzales vs. Rojas, 16 Phil. 51)
In the case on hand, the car in question was never delivered to the vendee
by the vendor as to complete or consummate the transfer of ownership by
virtue of the contract. It should be recalled that while there was indeed a
contract of sale between Vicente Marella... and Teodoro Santos, the former,
as vendee, took possession of the subject matter thereof by stealing the
same while it was in the custody of the latter's son.
The lower court was correct in applying Article 559 of the Civil Code to the
case at bar. For under it, the rule is to the effect that if the owner has lost
the thing, or if he has been unlawfully deprived of it, he has a right to
recover it, not only from the finder, thief... or robber, but also from the third
person who may have acquired it in good faith from such finder, thief or
robber.
he said article establishes two exceptions to the general rule of
irrevindicability to wit: when the owner (1) has lost the thing, or (2) has
been... unlawfully deprived thereof. In these cases, the possessor cannot
retain the thing as against the owner, who may recover it without paying
any indemnity, except when the possessor acquired it in a public sale.
46. Noel vs CA, G.R. No. 59550

Facts:

Noel vs. CA (240 SCRA 78)Spouses Gregorio and Hilaria Nanaman acquired
a property in IliganCity. Gregorio, however, had a child named Virgilio
Nanaman by another woman. Later on, Gregorio died. Hilaria and
Virgilio then administered theproperty Consequently, Virgilio declared
the property in his name for taxationpurposes. Then, Hilaria and Virgilio,
mortgaged the said property to JoseDeleste (private respondent)Later on,
Hilaria and Virgilio executed a deed of sale over the same tract of land also
in favor of private respondent. When Hilaria died, the daughters filed
intestate estate proceedings concerning the estate of their father, Gregorio.

When Edilberto Noel took over as regular administrator of the estate,he was
not able to take possession of the land in question because it was in the
possession of private respondent and some heirs of Hilaria. The court
thereafter ordered Noel, as regular administrator, to file an action to recover
the land from private respondent.

On December 14, 1973, the trial court rendered a decision, holding that the
action for annulment of the deed of sale. Noel appealed to the Court of
Appeals. In its Decision of February 18,1980, the appellate court ruled that
the transaction between Hilaria and Virgilio on one hand and private
respondent on the other, was indeed a sale. It found that no fraud,
mistake or misrepresentation attended in the execution of the deed of
sale and that no proof was shown that the contract was merely a mortgage.

Issue:

Whether or not Hilaria and Virgilio could dispose of the entire property sold
to private respondent and assuming that they did not have full
ownership thereof.

Held:

Virgilio was not an heir of Gregorio under the Spanish Civil Code of 1889.
Although he was treated as a child by the Nanaman spouses, illegitimate
children who were not natural were disqualified to inherit under the
said Code (Cid v. Burnaman, 24 SCRA 434[1968]).

Article 998 of the Civil Code of the Philippines, which gave an illegitimate
child certain hereditary rights, could not benefit Virgilio because the right of
ownership of the collateral heirs of Gregorio had become vested upon his
death (Civil Code of the Philippines, Art. 2253; Uson v. Del Rosario,92 Phil.
530 [1953]).

Therefore, Virgilio had no right at all to transfer ownership over


which he did not own. In a contract of sale, it is essential that the seller is
the owner of the property he is selling. The principal obligation of a seller is
"to transfer the ownership of" the property sold (Civil Code of the
Philippines, Art. 1458).
This law stemsfrom the principle that nobody can dispose of that which does
not belong tohim (Azcona v. Reyes, 59 Phil. 446 [1934]; Coronel v. Ona, 33
Phil. 456[1916).NEMO DAT QUAD NON HABET. While it cannot be said that
fraud attended the sale to private respondent, clearly there was a mistake
on the part of Hilaria and Virgilio in selling an undivided interest in the
property which belonged to the collateral heirs of Gregorio.
47. Asset Privatization Trust vs TJ Enterprises , G.R. NO. 167195 

 Facts:

Petitioner was a government entity created for the purpose to conserve, to


provisionally manage and to dispose assets of government institutions. It
had acquired assets consisting of machinery and refrigeration equipment
stored at the Golden City compound which was leased to and in the physical
possession of Creative Lines, Inc., (Creative Lines).

These assets were being sold on an as-is-where-is basis.Petitioner and


respondent entered into an absolute deed of sale over certain machinery and
refrigeration equipment wherein respondent paid the full amount as
evidenced by petitioner’s receipt. After two (2) days, respondent demanded
the delivery of the machinery it had purchased. Petitioner issued a Gate Pass
to respondent to enable them to pull out from the compound the properties
designated; however, during the hauling of Lot No. 2 consisting of sixteen
(16) items, only nine (9) items were pulled out by respondent.

Respondent filed a complaint for specific performance and damages against


petitioner and Creative Lines. Upon inspection of the remaining items, they
found the machinery and equipment damaged and had missing parts.
Petitioner claimed that there was already a constructive delivery of the
machinery and equipment upon the execution of the deed of sale it had
complied with its obligation to deliver the object of the sale since there was
no stipulation to the contrary and it was the duty of respondent to take
possession of the property.

The RTC ruled that petitioner is liable for breach of contract and should pay
for the actual damages suffered by respondent. It found that at the time of
the sale, petitioner did not have control over the machinery and equipment
and, thus, could not have transferred ownership by constructive delivery.
The Court of Appeals affirmed the judgment; hence, this petition.

Issue:

Whether or not the petitioner had complied with its obligations to make
delivery of the properties and failure to make actual delivery of the
properties was not attributable was beyond the control of petitioner?

Held:

 No. There was no constructive delivery of the machinery and equipment


upon the execution of the deed of absolute sale or upon the issuance of the
gate pass since it was not the petitioner but Creative Lines which had actual
possession of the property. The presumption of constructive delivery is not
applicable as it has to yield to the reality that the purchaser was not placed
in possession and control of the property.
48. ROSENDO HERNAEZ y ESPINOSA, plaintiff-appellant, vs. MATEO
HERNAEZ y ESPINOSA, ET AL., defendants-appellants, 32 Phil. 214
(1915)
Facts:
The spouses, Pedro Hernaez and Juana Espinosa, died, leaving several
legitimate descendants. Neither of their estates had been divided up to the
date of the institution of this action, but were both under administration.
Their son, Domingo Hernaez y Espinosa, sold all his interest in both his
father's and mother's estate to his son, Vicente Hernaez y Tuason.
Notwithstanding the fact that Domingo Hernaez y Espinosa had thus parted
with all his interest, he executed a document of sale in favor of Alejandro
Montelibano y Ramos, in which he purported to convey all his undivided
interest in his mother's estate. On the same day he executed another
document of sale to Jose MontelibanoUy-Cana. Both of these sales were
made with the connivance of his son. Vicente Hernaez y Tuason, the
undoubted owner thereof, is effectually estopped from asserting his title as
against either of the vendees mentioned.
Jose MontelibanoUy-Cana sold his interest in the estate to Alejandro
Montelibano y Ramos. By this transfer, the latter stood owner of all the
interest of Domingo Hernaez y Espinosa. RosendoHernaez y Espinosa,
another son of the deceased spouses administrator of the estates, was
notified of Montelibano's purchases, when he received notice of
Montelibano's motion, entered in the administration proceedings, asking that
he (Montelibano) be substituted as assignee of the interests of various heirs
of the estate which he had acquired by purchase. Notwithstanding this
knowledge, RosendoHernaez y Espinosa entered into a contract of sale with
Vicente Hernaez y Tuason, whereby the latter purported to convey all the
interest, which he had acquired from his father, in the estate of the
deceased spouses. It will be remembered that he purchased his father's
share of the estate and that he is estopped from asserting title to any
interest in his grandfather's estate and in five-eighteenths of his
grandmother's estate.Rosendo Hernandez y Espinosa purchased with full
knowledge of these facts, therefore, acquired thirteen-eighteenths of the
interest of Domingo Hernaez y Espinosa in the estate of the latter's mother
nothing more.The plaintiff instituted this action seeking to subrogate himself
in the rights acquired by Montelibano in the estate.
The interest which Jose MontelibanoUy-Cana purchased from Domingo
Hernaez y Espinosa, for the sum of P4,500, he afterwards transferred to
Alejandro Montelibano y Ramos for the sum of P10,000.
Issue:
WON the price which an heir is obliged to pay to the purchaser of the
interest of a coheir in order to exercise the right of subrogation is the actual
price at which the coheir parted with the interest.
Ruling:
The sales whereby Domingo Hernaez y Espinosa parted with that portion of
his interest in the estate which is now held by Alejandro Montelibano, as well
as to those sales made by other heirs to Montelibano, the trial court found
that the plaintiff, RosendoHernaez y Espinosa, was not chargeable with
notice. As a consequence, the plaintiff, RosendoHernaez y Espinosa, is
entitled to exercise his right of subrogation.
Article 1067 of the Civil Code provides that the coheir may exercise this right
of subrogation upon the payment to the purchaser of another heir’s interest,
"el precio de la compra" (the purchase price). Obviously, if the interest had
not been resold, the plaintiff, RosendoHernaez y Espinosa, would have had
to pay only the price for which Uy-Cana acquired it. The purpose of the
article cannot be evaded by a reconveyance of the interest to a third person
at a higher price. Subsequent purchasers of the interest acquire it burdened
with the right of subrogation of coheirs at the price for which the heir who
sold it parted with it.
It is urged that the prices in some of the deeds of sale by which Alejandro
Montelibano y Ramos purchased the interests of various heirs in the estates
are fictitious. This is a question of fact upon which both parties adduced
evidence, and we concur in the opinion of the trial court that there is no
basis to the charge.  For the foregoing reasons, the judgment of the court is
modified by substituting, as the price of subrogation of the interest originally
purchased by Jose MontelibanoUy-Cana, the sum of P4,500 and for the sum
of P10,000. As modified, the judgment appealed from is affirmed.
49. VICENTA JALBUENA, plaintiff-appellant, vs. SALVADOR
LIZARRAGA, et al., defendants-appellees, 33 Phil. 77 (1915)
Facts:
Salvador Lizarraga, as judgment creditor, caused the sheriff of the Province
of Iloilo to levy upon an old sugar-mill as the property of Ildefonso Doronila,
the judgment debtor and husband of the plaintiff, Vicenta Jalbuena. At the
time of the levy Doronila stated to the sheriff that the mill belonged to him.
The property was sold in a public auction to Lopez. The plaintiff filed an
action for the purpose of recovering the mill or its value upon the ground
that the same was her exclusive property and that her husband had no
interest therein.The plaintiff knew that the old sugar-mill had been levied;
and that it would be sold as the property of her husband. However, she
stood by and permitted the sale to go forward without making the slightest
protest until ownership had already passed into the hands of Lopez.
Issue:
WONplaintiff can recover the property or value of the property sold at a
judicial sale as the property of the judgment debtor.
Ruling:
The court held that “an execution is an order to the sheriff to attach and sell
the property of the judgment debtor. If he sells the property of another
person, he exceeds his authority and the true owner may sue in trespass for
damages or for the recovery of the property, provided he has not lost his
right to do so by his own conduct.
Bigelow on Estoppel says: "... it is now a well-established principle that
where the true owner of property, for however short a time, holds out
another, or, with knowledge of his own right, allows another to appear, as
the owner of or as having full power of disposition over the property, the
same being in the latter's actual possession, and innocent third parties are
thus led into dealing with some [such] apparent owner, they will be
protected." (Quoted with approval in the case of Hernaez vs. Hernaez, 32
Phil. Rep., 214.) The foregoing quotations from Cyc. and Bigelow are in
harmony with No. 1 of section 333 of the Code of Civil Procedure, wherein it
is provided that — "Whenever a party has, by his own declaration, act, or
omission, intentionally and deliberately led another to believe a particular
thing true, and to act upon such belief, he cannot, in any litigation arising
out of such declaration, act, or omission, be permitted to falsify it."The
plaintiff had full knowledge of the fact that the property was going to be sold
to pay the debts of her husband. She did not communicate her claim to the
purchaser, and it is now too late to assert such a claim.The judgment
appealed from is affirmed.
50. LUCENA MASICLAT, ET AL., petitioner, VS. NATALIA CENTENO,
respondent, G.R. No. L-8420 (1956)
Facts:
Centeno sold sacks of rice at a store on a street near the public market. A
person offered to buy 15 sacks of rice, to be paid as soon as he sold his
adobe stone, which was being unloaded, from a truck on the opposite side of
the street. Centeno then order the sacks of rice to be loaded onto the truck.
Masiclat, loaded the rice while Centeno watched. However the buyer did not
come back. Upon orders of Centeno to unload the rice, Masiclat objected.
Masiclat claimed to have bought the rice.
Issue:
WON the contract of sale was consummated between respondent Centeno
and unknown purchaser (alleged unknown seller to plaintiff)
WON petitioners have a better title to the rice in question
Ruling:
NO. Although a contract of sale is perfected upon the parties having agreed
as to the thing which is the subject matter of the contract and the price,
ownership is not considered transmitted until the property is actually
delivered and the purchaser has taken possession and paid the price agreed
upon. The evidence does not clearly show the identity of the person who
tried to buy the rice in question from the respondent, and neither does it
show that the same person was the one who sold the commodity to Ramon
Masiclat. The sale between the respondent Centeno and the unknown
purchaser was not consummated because although the former allowed the
rice in question to be loaded in the truck, she did not intend to transfer its
ownership until she was paid the stipulated price; and this is very evident
from the fact that respondent continually watched her rice and demanded its
unloading as soon as the unknown purchaser was missing. Respondent thus
has not lost ownership and legal possession thereof.
NO. The general principle of law as enunciated in A1505 CC that where one
of 2 persons must suffer the fraud of a third, the loss should fall upon him
who has enabled the third person to do the wrong, does not apply for the
following reasons: 1) there was no definite finding that the unknown
purchaser was same person who sold the rice to Masiclat, 2) Centeno could
not have been so negligent as to allow the unknown purchaser to run away
with said rice and enable him to sell it to Masiclat, it evident that in fact
Centeno kept an eye on the rice in question.
Decision of CA is affirmed.
51. CONSUELO S. DE GARCIA and ANASTACIO GARCIA, petitioners,
vs. HON. COURT OF APPEALS, ANGELINA D. GUEVARA and JUAN B.
GUEVARA, respondents, 37 SCRA 129, (1971)
Facts:
Private respondent Angelina D. Guevara, while talking to Consuelo S. de
Garcia, owner of a restaurant, she recognized her ring in the finger of Mrs.
Garcia and inquired where she bought it, which the she answered from her
comadre. Mrs. Guevara explained that that ring was stolen from her house.
Mrs. De Garcia handed the ring to Mrs. Guevara and it fitted her finger. Two
or three days later, at the request of plaintiff, her husband Lt. Col. Juan
Guevara, defendant and her attorney proceeded to the store of Mr. Rebullida
to whom they showed the ring in question. Mr. Rebullida examined the ring
and after consulting the stock card thereon, concluded that it was the very
ring that plaintiff bought from him. The ring was returned to Mrs. De Garcia
who despite a written request therefor failed to deliver the ring to Mrs.
Guevara. Later on when the sheriff tried to serve the writ of seizure
(replevin), defendant refused to deliver the ring which had been examined
by Mr. Rebullida, claiming it was lost.
According to De Garcia, she bought the ring from her kumare who got it
from another Miss who in turn got it from the owner, a certain AlingPetring.
AlingPetring however, was nowhere to be found. She boarded three months
at the first buyer’s house but left a week after her landlady bought the ring.
The first buyer did not even know AlingPetring’s last name nor her
forwarding address. De Garcia claims to be a holder in good faith and for
value. She says her possession is equivalent to title.
Issue:
WON the buyer of a lost or stolen property acquires a better right to the
property.
Ruling:
The facts as duly found by respondent Court, which we are not at liberty to
disregard, and the governing legal provisions, there is no basis for reversal.
The decision of respondent Court of Appeals is affirmed.It correctly applied
the law to the facts as found.
Respondent Angelina D. Guevara, having been unlawfully deprived of the
diamond ring in question, was entitled to recover it from petitioner Consuelo
S. de Garcia who was found in possession of the same.The controlling
provision is Article 559 of the Civil Code.  It reads thus:  "The possession of
movable property acquired in good faith is equivalent to a
title.  Nevertheless, one who has lost any movable or has been unlawfully
deprived thereof may recover it from the person in possession of the
same.  If the possessor of a lost movable of which the owner has been
unlawfully deprived, has acquired it in good faith at a public sale, the owner
cannot obtain its return without reimbursing the price paid therefore. "The
only exception the law allows is when there is acquisition in good faith of
the possessor at a public sale, in which case the owner cannot obtain its
return without reimbursing the price.The common law principle that where
one of two innocent persons must suffer by a fraud perpetrated by another,
the law imposes the loss upon the party who, by his misplaced confidence,
has enabled the fraud to be committed, cannot be applied in a case which is
covered by an express provision of the new Civil Code, specifically Article
559.  Between a common law principle and a statutory provision, the latter
must prevail in this jurisdiction."There is no merit in the contention that De
Garcia’s possession is in good faith, equivalent to title, sufficed to defeat the
owner’s claim.
52. EDCA PUBLISHING v. SANTOS

FACTS:
EDCA Publishing sold 406 books to a certain Professor Jose Cruz who
ordered these by telephone, which was agreed to be payable on delivery.
The books were subsequently delivered to him with the corresponding
invoice, and he paid with a personal check.
Cruz then sold the 120 of the books to Leonor Santos who asked for
verification, and was then showed the invoice for the books.

EDCA became suspicious when Cruz ordered another set of books even
before his check cleared. Upon investigation, EDCA found that he wasn’t the
person he claimed to be (Dean in DLSU). EDCA had the police capture Cruz,
as well as seize the books from Santos. Santos demanded the return of the
books.
RTC granted the writ of preliminary attachment.
Subsequent dishonor of a check, which did not render the contract of sale
void does not amount to unlawful deprivation of property. (There was a
perfected contract of sale so the proper remedy is specific performance)

ISSUE:
Whether or not the owner was unlawfully deprived of the property?

HELD:
No. Santos was a good faith buyer after taking steps to verify the identity of
the seller. When she was showed the invoice, she reasonably believed that
he was a legitimate seller.With regard to unlawful deprivation, EDCA was not
unlawfully deprived of the property by mere failure of consideration. There
was already a perfected contract of sale. Proof was even substantiated when
EDCA gave the invoice as proof of payment upon delivery of the books. This
did not amount to unlawful taking, because by the delivery of EDCA to Cruz,
ownership of the books already transferred to him.
53. Smith Bell vs. Sotelo Matti (44 Phil. 874)

Facts:
Plaintiff corporation undertook to sell and deliver equipment for Mr. Sotelo
but no definite dates were fixed for the delivery. The periods were couched
in ambiguous terms such as “within 3 or 4 months”, “in the month of
September or as soon as possible”, and “approximate delivery with 90 days-
This is not guaranteed.” When the goods arrived, Mr. Sotelo refused to
receive them and to pay the prices. Mr. Sotelo then sued for damages
because of the delay suffered.

Issue:
Whether Smith Bell incurred delay in the delivery of goods to Sotelo.

Held:
No, it did not incur delay.
From the record it appears that these contracts were executed at the time of
the world war when there existed connection with the tanks and "Priority
Certificate, subject to the United -States Government requirements," with
respect to the motors. At the time of the execution of the contracts, the
parties were not unmindful of the contingency of the United States
Government not allowing the export of the goods, nor of the fact that the
other foreseen circumstances therein stated might prevent it.
Considering these contracts in the light of the civil law, we cannot but
conclude that the term which the parties attempted to fix is so uncertain
that one cannot tell just whether, as a matter of fact, those articles could be
brought to Manila or not. If that is the case, as we think it is, the obligation
must be regarded as conditional.
When the delivery was subject to a condition the fulfillment of which
depended not only upon the effort of the herein plaintiff, but upon the will of
third persons who could in no way be compelled to fulfill .the condition. In
cases like this, which are not expressly provided for, but impliedly covered,
by the Civil Code, the obligor will be deemed to have sufficiently performed
his part of the obligation, if he has done all that was in his power, even if the
condition has not been fulfilled in reality.
In connection with this obligation to deliver, occurring in a contract of sale
like those in question, the rule in North America is that when the time of
delivery is not fixed in the contract, time is regarded unessential.
When the contract provides for delivery 'as soon as possible' the seller is
entitled to a reasonable time, in view of all the circumstances, such as the
necessities of manufacture, or of putting the goods in condition for delivery.
The term does not mean immediately or that the seller must stop all his
other work and devote himself to that particular order. But the seller must
nevertheless act with all reasonable diligence or without unreasonable delay.
It has been held that a requirement that the shipment of goods should be
the 'earliest possible' must be construed as meaning that the goods should
be sent as soon as the seller could possibly send them, and that it signified
rather more than that the goods should be sent within a reasonable time.
"The question as to what is a reasonable time for the delivery of the goods
by the seller is to be determined by the circumstances attending the
particular transaction, such as the character of the goods, and the purpose
for which they are intended, the ability of the seller to produce the goods if
they are to be manufactured, the facilities available for transportation, and
the distance the goods must be carried, and the usual course of business in
the particular trade." (35 Cyc., 181-184.)
The record shows, as we have stated, that the plaintiff did all within its
power to have the machinery arrive at Manila as soon as possible, and
immediately upon its arrival it notified the purchaser of the fact and offered
to deliver it to him. Taking these circumstances into account, we hold that
the said machinery was brought to Manila by the plaintiff within a reasonable
time.
Therefore, the plaintiff has not been guilty of any delay in the fulfillment of
its obligation, and, consequently, it could not have incurred any of the
liabilities mentioned by the intervenor in its counterclaim or set-off.
54. Soler v. Chelsey, 43 Phil 529 (1922)

Facts:
The plaintiff had agreed with Wm. H. Anderson and Co., for the purchase of
certain machinery, as evidenced by the document Exhibit A.
On April 25, 1919, the defendant's attorney-in-fact, Fred A. Leas, through
Attorney Francisco A. Delgado, wrote the plaintiff the letter, Exhibit 2,
advising him that he contract above referred to was rescinded, it appearing
that the parts of the machinery, which the plaintiff asserted in said contract
were on the way, were not at the time and it was only several days later
that they were shipped for Manila. In this letter the parts received were
placed at the plaintiff's disposal upon the repayment of the sums advanced
by the defendant to Messrs. Anderson and Co.
On the 14th of October, 1919, the plaintiff commenced this action in which,
basing himself on the contract Exhibit B and on the facts set forth in his
complaint, he prayed that the defendant be sentenced to pay him the sum of
P30,546.03 with interest thereon, which sum was the difference between the
P100,000, the consideration of the contract, Exhibit B, and the price of the
aforesaid machinery which had been paid by the defendant, plus the
incidental expenses, as stipulated in the said contract.
The defendant answered, denying generally and specifically the allegations
of the complaint and setting up a special defense and a counterclaim. In his
special defense, he alleges that he had accepted and signed the contract
Exhibit B on the assertion therein contained that of the machinery, which
was the subject matter of the said contract, a part was already in Manila,
and the other part on the way, and also on the promises, assertions, and
contemporary and previous acts of the plaintiff to the same effect, by means
of which the latter succeeded in inducing the defendant to make and sign
the aforesaid contract; that the parts of the machinery which, on the date of
the contract, were said to be on the way, were not in fact in, and did not
arrive at, Manila but long thereafter; that if he signed the contract, it was
because he was desirous of having the machinery, and the defendant
assured him that it would be delivered to him, immediately or within a short
time; that otherwise he would not have signed the contract; that the
prepared in a shed the necessary compartments to install the machinery on
or before the 1st day of January, 1919; that on April 25, 1919, he advised
the plaintiff that he regarded the contract as rescinded; that he had
complied with his part of the contract, having paid Messrs. Anderson and Co.
the sum of P69,453.97; that he suffered damages in the sum of P120,000.

Issue:
Whether or not that time is an essential element of the contract Exhibit B.

Held:
Yes. It appears sufficiently established in the record that if the plaintiff gave
his consent to this contract, it was because he expected that said machinery
would arrive within a short time, — the time reasonably necessary for such
machinery to reach Manila from America, — as the plaintiff asserted in the
document itself that said machinery was then on the way. The act of the
defendant in insisting that this guaranty as to the arrival of the machinery
be stated in the contract, his repeated complaints and protests when he
afterwards made payments as the parts arrived, and his letter of April 25,
1919, leave no room for doubt that the arrival of said machinery within a
reasonably short time was one of the determining elements of his consent.
These acts of the defendant disclose the fact that he intended the arrival of
the machinery to be an essential element of the contract (art. 1282, Civil
Code). We hold that in the case at bar the arrival of the machinery within a
reasonable time was an essential element of the contract, such time to be
determined by taking into account the fact that is was then on the way to
Manila.
55. Republic v. Litton, 94 Phil 52 (1953)

Facts:
Defendants are the partnership Litton &Co, and George Litton. They sell
office supplies from the United States. The defendants were contracted to
supply (1) Padlocks and (2) office supplies for the upcoming April 23,
1946elections.They were tasked to deliver all of the above by April 8. Their
contract with the Republic included a stipulation that “The stipulated delivery
period shall not be exceeded.The items were insured by Central Surety Co.
with bonds.The defendants notified the Republic through mail that it is
understood that the government will give a lettercertifying that the padlocks
are urgently needed and that the export license can be secured without
delay.The defendants did not deliver everything on time.The defendants
asked for an extension which was ignored. The Republic was forced to buy
the remaining equipment from another supplier at a higher cost because
they still lacked some supplies for the elections. It is now filing a case of
damages against the defendants for their failure to deliver the goods on time
for the elections. The defendants claim they are not liable because it was
necessary for the republic to furnish the “export license” inorder to quicken
the delivery of the items to meet the date.

Issue: 
Whether or not it is a condition precedent for the Republic to first provide th
e “export license” before the defendants could meet the strict deadline they 
had agreed upon?

Held: 
No, the intent of the contract, was to furnish the demanded supplies before t
he elections, regardless if there was an export license or not (which was stric
tly provided for in the contract’s stipulation
The contract has a section called “Important Conditions,” which carries the
stipulation that “The stipulated delivery period shall not be exceeded.” This
makes defendants liable in all eventualities.The letter wrote by the
defendants only shows that they merely expected the Republic to give a
certification. Itwas never considered a condition precedent.
56. FELICIANO ESGUERRA, et al. v. VIRGINIA TRINIDAD, et al.

518 SCRA 186 (2007)

What really defines a piece of ground is not the area, calculated with more
or less certainty, mentioned in its description, but the boundaries therein
laid down, as enclosing the land and indicating its limits.

Facts:

Felipe Esguerra and Praxedes de Vera (Esguerra spouses) owned several


parcels of land half of which they sold to their grandchildren Feliciano,
Canuto, Justa, Angel, Fidela, Clara and Pedro, all surnamed Esguerra. The
spouses sold half the remaining land were sold their other grandchildren, the
brothers Eulalio and Julian Trinidad..Subsequentlly, the Esguerra spouses
executed the necessary Deeds of Sale before a notary public. They also
executed a deed of partitioning of the lots , all were about 5,000 square
meteres each.

Eulalio Trinidad (Trinidad) later sold his share of the land to his daughters.
During a cadastral survey conducted in the late 1960s, it was discovered
that the 5,000-square meter portion of Esguerra‘s parcel of land sold to
Trinidad actually measured 6,268 square meters.

Feliciano Esguerra (Feliciano), who inhabits the lot bordering Trinidad,


subsequently filed a motion for nullification of sale between the Esguerra
spouses and Trinidad on the ground that they were procured
through fraud or misrepresentation. Feliciano contended that the stipulations
in the deed of sale was that Trinidad was sold a 5,000 square meter lot. The
boundaries stipulated in the contract of sale which extend the lot‘s area

Both cases were consolidated and tried before the RTC which, after trial,
dismissed the cases. On appeal, the appellate court also dismissed the
cases; and subsequently, the motion for reconsideration was also denied.

ISSUES:

Whether or not the Appellate Court erred in holding that the description and
boundaries of the lot override the stated area of the lot in the deed of sale

HELD:

Where both the area and the boundaries of the immovable are declared, the
area covered within the boundaries of the immovable prevails over the
stated area. In cases of conflict between areas and boundaries, it is
the latter which should prevail.

What really defines a piece of ground is not the area, calculated with more
or less certainty, mentioned in its description, but the boundaries therein
laid down, as enclosing the land and indicating its limits. In a contract of sale
of land in a mass, it is well established that the specific boundaries stated in
the contract must control over any statement with respect to the
area contained within its boundaries. It is not of vital consequence that a
deed or contract of sale of land should disclose the area with mathematical
accuracy. It is sufficient if its extent is objectively indicated with sufficient
precision to enable one to identify it. An error as to the superficial area is
immaterial. Thus, the obligation of the vendor is to deliver everything within
the boundaries, inasmuch as it is the entirety thereof that distinguishes the
determinate object.
Under the Torrens System, an OCT enjoys a presumption of validity, which
correlatively carries a strong presumption that the provisions of the law
governing the registration of land which led to its issuance have been duly
followed. Fraud being a serious charge, it must be supported by clear and
convincing proof. Petitioners failed to discharge the burden of proof,
however.

The same rule shall be applied when two or more immovables are sold for a
single price; but if, besides mentioning the boundaries, which is
indispensable in every conveyance of real estate, its area or number should
be designated in the contract, the vendor shall be bound to deliver all that is
included within said boundaries, even when it exceeds the area or number
specified in the contract; and, should he not be able to do so, he shall suffer
a reduction in the price, in proportion to what is lacking in the area or
number, unless the contract is rescinded because the vendee does not
accede to the failure to deliver what has been stipulated.

In fine, under Article 1542, what is controlling is the entire land included
within the boundaries, regardless of whether the real area should be greater
or smaller than that recited in the deed. This is particularly true since the
area of the land in OCT No. 0-6498 was described in the deed as
“humigitkumulang,” that is, more or less.

A caveat is in order, however. The use of “more or less” or similar words in


designating quantity covers only a reasonable excess or deficiency. A vendee
of land sold in gross or with the description “more or less” with reference to
its area does not thereby ipso facto take all risk of quantity in the land.
57. ALFREDO VS. BORRAS,
G.R. No. 144225, June 17, 2003, 404 SCRA 145.

Facts:
A parcel of land measuring 81,524 square meters ("Subject Land") in
BarrioCulis, Mabiga,Hermosa, Bataan is the subject of controversy in this
case.The spouses Godofredo Alfredo ("Godofredo")and Carmen Limon
Alfredo("Carmen") were the registered owners and they subsequently sold
thesameto the spouses Armando Borras ("Armando") and Adelia Lobaton
Borras ("Adelia") where some of themoney was used to pay the mortgage
loan of spouses Alfredo with the Development Bank of the
Philippines("DBP"). Thereafter, the spouses Borras took possession of the
land but discovered that the spouses Alfredore-sold the land to other
persons.Spouses Borras filed an adverse claim with the Register of Deeds of
Bataan. They discovered thatGodofredo and Carmen had secured an owner's
duplicate copy of OCT No. 284 after filing a petition incourt for the issuance
of a new copy. Godofredo and Carmen claimed in their petition that they lost
theirowner's duplicate copy. Armando and Adelia wrote Godofredo and
Carmen complaining about their acts,but the latter did not reply. Thus,
Armando and Adelia filed a complaint for specific performance.Armando and
Adelia amended their complaint to include the following persons as
additional defendants:the spouses Arnulfo Savellano and Editha B.
Savellano, Danton D. Matawaran, the spouses Delfin F.Espiritu, Jr. and
Estela S. Espiritu, and Elizabeth Tuazon ("Subsequent Buyers"). The
Subsequent Buyers,who are also petitioners in this case, purchased from
Godofredo and Carmen the subdivided portions of theSubject Land. The
Register of Deeds of Bataan issued to the Subsequent Buyers transfer
certificates of titleto the lots they purchased.In their answer, Godofredo and
Carmen and the Subsequent Buyers (collectively "petitioners") argued
thatthe action is unenforceable under the Statute of Frauds. Petitioners
pointed out that there is no writteninstrument evidencing the alleged
contract of sale over the Subject Land in favor of spouses Borras.Petitioners
asserted that the Subsequent Buyers were buyers in good faith and for
value.The Court ruled in favor of the spouses Borras. Petitioners appealed to
the Court of Appeals which thereafterissued its Decision affirming the
decision of the trial court. It denied petitioners' motion for reconsideration.
The Court of Appeals found the factual findings of the trial court well
supported by the evidence. Based onthese findings, the Court of Appeals
also concluded that there was a perfected contract of sale and
theSubsequent Buyers were not innocent purchasers.Petitioners by petition
for review oncertiorari under Rule 45 involve the absence of approval of the
saleby the Secretary of Agriculture and Natural Resources to nullify the sale.
Issue/s:

Whether the alleged sale of the Subject Land in favor of Armando and
Adelia is valid and enforceable, where (1) it was orally entered into
and not in writing; (2) Carmen did not obtain the consent and
authority of her husband, Godofredo, who was the sole owner of the
Subject Land in whose name the title thereto (OCT No. 284) was
issued; and (3) it was entered into during the 25-year prohibitive
period for alienating the Subject Land without the approval of the
Secretary of Agriculture and Natural Resources.

II

Whether the action to enforce the alleged oral contract of sale


brought after 24 years from its alleged perfection had been barred by
prescription and by laches.

III

Whether the deeds of absolute sale and the transfer certificates of


title over the portions of the Subject Land issued to the Subsequent
Buyers, innocent purchasers in good faith and for value whose
individual titles to their respective lots are absolute and indefeasible,
are valid.

IV

Whether petitioners are liable to pay Armando and


Adelia P20,0000.00 as attorneys fees and litigation expenses and the
treble costs, where the claim of Armando and Adelia is clearly
unfounded and baseless.

Whether petitioners are entitled to the counterclaim for attorneys


fees and litigation expenses, where they have sustained such
expenses by reason of institution of a clearly malicious and unfounded
action by Armando and Adelia.[8

Held:
Yes, but only as to errors of law. In a petition for review oncertiorariunder
Rule 45, this Courtreviews only errors of law and not errors of facts. The
factual findings of the appellate court are generallybinding on this Court.
This applies with greater force when both the trial court and the Court of
Appealsare in complete agreement on their factual findings. In this case,
there is no reason to deviate from thefindings of the lower courts. The facts
relied upon by the trial and appellate courts are borne out by therecord. We
agree with the conclusions drawn by the lower courts from these facts.

Petitioners contend that Godofredo and Carmen did not deliver the title of
the Subject Land to Armandoand Adelia as shown by this portion of Adelia's
testimony on cross-examination.Petitioners raise this factual issue for the
first time. The Court of Appeals could have passed upon this issuehad
petitioners raised this earlier. At any rate, the cited testimony of Adelia does
not convincingly provethat Godofredo and Carmen did not deliver the
Subject Land to Armando and Adelia. Adelia's citedtestimony must be
examined in context not only with her entire testimony but also with the
othercircumstances.Petitioners involve the absence of approval of the sale
by the Secretary of Agriculture and NaturalResources to nullify the sale.
Petitioners never raised this issue before the trial court or the Court of
Appeals.Litigants cannot raise an issue for the first time on appeal, as this
would contravene the basic rules of fairplay, justice and due process.
However, we will address this new issue to finally put an end to this case.A
grantee or homesteader is prohibited from alienating to a private individual a
land grant within five yearsfrom the time that the patent or grant is issued.
A violation of this prohibition renders a sale void. Thisprohibition, however,
expires on the fifth year. From then on until the next 20 years the land grant
may bealienated provided the Secretary of Agriculture and Natural
Resources approves the alienation. TheSecretary is required to approve the
alienation unless there are "constitutional and legal grounds" to denythe
approval. In this case, there are no apparent constitutional or legal grounds
for the Secretary todisapprove the sale of the Subject Land.The failure to
secure the approval of the Secretary does notipso factomake a sale void.
The absence ofapproval by the Secretary does not nullify a sale made after
the expiration of the 5-year period, for in suchevent the requirement of
Section 118 of the Public Land Act becomes merely directory or a formality.
Theapproval may be secured later, producing the effect of ratifying and
adopting the transaction as if the salehad been previously authorized.

The contract of sale between the spouses Godofredo and Carmen and the
spouses Armando and Adelia was a perfected contract. A contract is
perfected once there is consent of the contracting parties on the object
certain and on the cause of the obligation.[12 In the instant case, the object
of the sale is the Subject Land, and the price certain is P15,000.00. The trial
and appellate courts found that there was a meeting of the minds on the
sale of the Subject Land and on the purchase price of P15,000.00. This is a
finding of fact that is binding on this Court. We find no reason to disturb this
finding since it is supported by substantial evidence.

The contract of sale of the Subject Land has also been consummated
because the sellers and buyers have performed their respective obligations
under the contract. In a contract of sale, the seller obligates himself to
transfer the ownership of the determinate thing sold, and to deliver the
same, to the buyer who obligates himself to pay a price certain to the seller.
[13 In the instant case, Godofredo and Carmen delivered the Subject Land
to Armando and Adelia, placing the latter in actual physical possession of the
Subject Land. This physical delivery of the Subject Land also constituted a
transfer of ownership of the Subject Land to Armando and Adelia.
[14 Ownership of the thing sold is transferred to the vendee upon its actual
or constructive delivery.[15 Godofredo and Carmen also turned over to
Armando and Adelia the documents of ownership to the Subject Land,
namely the owners duplicate copy of OCT No. 284, the tax declaration and
the receipts of realty tax payments.

On the other hand, Armando and Adelia paid the full purchase price as
evidenced by the receipt dated 11 March 1970 issued by Carmen. Armando
and Adelia fulfilled their obligation to provide the P7,000.00 to pay the D

The trial and appellate courts correctly refused to apply the Statute of
Frauds to this case. The Statute of Frauds[16 provides that a contract for the
sale of real property shall be unenforceable unless the contract or some note
or memorandum of the sale is in writing and subscribed by the party
charged or his agent. The existence of the receipt dated 11 March 1970,
which is a memorandum of the sale, removes the transaction from the
provisions of the Statute of Frauds.

The Statute of Frauds applies only to executory contracts and not to


contracts either partially or totally performed.[17 Thus, where one party has
performed ones obligation, oral evidence will be admitted to prove the
agreement.[18 In the instant case, the parties have consummated the sale
of the Subject Land, with both sellers and buyers performing their respective
obligations under the contract of sale. In addition, a contract that violates
the Statute of Frauds is ratified by the acceptance of benefits under the
contract.[19 Godofredo and Carmen benefited from the contract because
they paid their DBP loan and secured the cancellation of their mortgage
using the money given by Armando and Adelia. Godofredo and Carmen also
accepted payment of the balance of the purchase price.

Godofredo and Carmen cannot invoke the Statute of Frauds to deny the
existence of the verbal contract of sale because they have performed their
obligations, and have accepted benefits, under the verbal
contract. 20 Armando and Adelia have also performed their obligations under
the verbal contract. Clearly, both the sellers and the buyers have
consummated the verbal contract of sale of the Subject Land. The Statute of
Frauds was enacted to prevent fraud.[21 This law cannot be used to advance
the very evil the law seeks to prevent.

Godofredo and Carmen also claim that the sale of the Subject Land to
Armando and Adelia is void on two grounds. First, Carmen sold the Subject
Land without the marital consent of Godofredo. Second, the sale was made
during the 25-year period that the law prohibits the alienation of land grants
without the approval of the Secretary of Agriculture and Natural Resources.

The Subsequent Buyers individual titles to their respective lots are not
absolutely indefeasible. The defense of indefeasibility of the Torrens Title
does not extend to a transferee who takes the certificate of title with notice
of a flaw in his title.[63 The principle of indefeasibility of title does not apply
where fraud attended the issuance of the titles as in this case.[64
58. ZENAIDA M. SANTOS, petitioner, vs. CALIXTO SANTOS, ALBERTO
SANTOS, ROSA SANTOS-CARREON and ANTONIO SANTOS,
respondents.
[G.R. No. 133895. October 2, 2001]

FACTS:

Petitioner Zenaida M. Santos is the widow of Salvador Santos, a brother of


private respondents Calixto, Alberto, Antonio, all surnamed Santos and Rosa
Santos-Carreon.
The spouses Jesus and Rosalia Santos owned a parcel of land. On it was a
four-door apartment administered by Rosalia who rented them out. The
spouses had five children, Salvador, Calixto, Alberto, Antonio and Rosa.
On January 19, 1959, Jesus and Rosalia executed a deed of sale of the
properties in favor of their children Salvador and Rosa. Rosa in turn sold her
share to Salvador on November 20, 1973. Despite the transfer of the
property to Salvador, Rosalia continued to lease and receive rentals from the
apartment units.
On November 1, 1979, Jesus died. Six years after or on January 9, 1985,
Salvador died, followed by Rosalia who died the following month. Shortly
after, petitioner Zenaida, claiming to be Salvador’s heir, demanded the rent
from Antonio Hombrebueno, a tenant of Rosalia. When the latter refused to
pay, Zenaida filed an ejectment suit against him with the Metropolitan Trial
Court of Manila, Branch 24, which eventually decided in Zenaida’s favor.
On January 5, 1989, private respondents instituted an action for
reconveyance of property with preliminary injunction against petitioner in
the Regional Trial Court of Manila, where they alleged that the two deeds of
sale executed on January 19, 1959 and November 20, 1973 were simulated
for lack of consideration. They were executed to accommodate Salvador in
generating funds for his business ventures and providing him with greater
business flexibility.
In her Answer, Zenaida denied the material allegations in the complaint and
as special and affirmative defenses, argued that Salvador was the registered
owner of the property, which could only be subjected to encumbrances or
liens annotated on the title; that the respondents’ right to reconveyance was
already barred by prescription and laches; and that the complaint stated no
cause of action.
On March 17, 1993, the trial court decided in private respondents’ favor. The
trial court reasoned that notwithstanding the deeds of sale transferring the
property to Salvador, the spouses Rosalia and Jesus continued to possess
the property and to exercise rights of ownership not only by receiving the
monthly rentals, but also by paying the realty taxes. Also, Rosalia kept the
owner’s duplicate copy of the title even after it was already in the name of
Salvador. Further, the spouses had no compelling reason in 1959 to sell the
property and Salvador was not financially capable to purchase it. The deeds
of sale were therefore fictitious. Hence, the action to assail the same does
not prescribe.
Upon appeal, the Court of Appeals affirmed the trial court’s decision. It held
that in order for the execution of a public instrument to effect tradition, as
provided in Article 1498 of the Civil Code, [5] the vendor shall have had
control over the thing sold, at the moment of sale. It was not enough to
confer upon the purchaser the ownership and the right of possession. The
thing sold must be placed in his control. The subject deeds of sale did not
confer upon Salvador the ownership over the subject property, because even
after the sale, the original vendors remained in dominion, control, and
possession thereof. The appellate court further said that if the reason for
Salvador’s failure to control and possess the property was due to his
acquiescence to his mother, in deference to Filipino custom, petitioner, at
least, should have shown evidence to prove that her husband declared the
property for tax purposes in his name or paid the land taxes, acts which
strongly indicate control and possession.

ISSUE:

1. Whether the payments of realty taxes and retention of possession are


indications of continued ownership by the original owners
2. Whether a sale through a public instrument is tantamount to delivery of
the thing sold
3. Whether the respondents’ cause of action has prescribed
4. Whether the petitioner can invoke the “Dead Man’s Statute

HELD:

1. It is true that neither tax receipts nor declarations of ownership for


taxation purposes constitute sufficient proof of ownership. They must be
supported by other effective proofs. These requisite proofs we find present in
this case. As admitted by petitioner, despite the sale, Jesus and Rosalia
continued to possess and administer the property and enjoy its fruits by
leasing it to third persons. Both Rosa and Salvador did not exercise any right
of ownership over it. Before the second deed of sale to transfer her 1/2
share over the property was executed by Rosa, Salvador still sought the
permission of his mother. Further, after Salvador registered the property in
his name, he surrendered the title to his mother. These are clear indications
that ownership still remained with the original owners. In Serrano vs. CA,
139 SCRA 179, 189 (1985), we held that the continued collection of rentals
from the tenants by the seller of realty after execution of alleged deed of
sale is contrary to the notion of ownership.
Petitioner argues that Salvador, in allowing her mother to use the property
even after the sale, did so out of respect for her and out of generosity, a
factual matter beyond the province of this Court. Significantly, in Alcos vs.
IAC, 162 SCRA 823, 837 (1988), we noted that the buyer’s immediate
possession and occupation of the property corroborated the truthfulness and
authenticity of the deed of sale. Conversely, the vendor’s continued
possession of the property makes dubious the contract of sale between the
parties.

2. Nowhere in the Civil Code, however, does it provide that execution of a


deed of sale is a conclusive presumption of delivery of possession. The Code
merely said that the execution shall be equivalent to delivery. The
presumption can be rebutted by clear and convincing evidence. Presumptive
delivery can be negated by the failure of the vendee to take actual
possession of the land sold.
In Danguilan vs. IAC, 168 SCRA 22, 32 (1988), we held that for the
execution of a public instrument to effect tradition, the purchaser must be
placed in control of the thing sold. When there is no impediment to prevent
the thing sold from converting to tenancy of the purchaser by the sole will of
the vendor, symbolic delivery through the execution of a public instrument is
sufficient. But if, notwithstanding the execution of the instrument, the
purchaser cannot have the enjoyment and material tenancy nor make use of
it himself or through another in his name, then delivery has not been
effected.
As found by both the trial and appellate courts and amply supported by the
evidence on record, Salvador was never placed in control of the property.
The original sellers retained their control and possession. Therefore, there
was no real transfer of ownership.
Moreover, in Norkis Distributors, Inc. vs. CA, 193 SCRA 694, 698-699
(1991), citing the land case of Abuan vs. Garcia, 14 SCRA 759 (1965), we
held that the critical factor in the different modes of effecting delivery, which
gives legal effect to the act is the actual intention of the vendor to deliver,
and its acceptance by the vendee. Without that intention, there is no
tradition. In the instant case, although the spouses Jesus and Rosalia
executed a deed of sale, they did not deliver the possession and ownership
of the property to Salvador and Rosa. They agreed to execute a deed of
sale merely to accommodate Salvador to enable him to generate funds for
his business venture.

3. In Lacsamana vs. CA, 288 SCRA 287, 292 (1998), we held that the right
to file an action for reconveyance on the ground that the certificate of title
was obtained by means of a fictitious deed of sale is virtually an action for
the declaration of its nullity, which does not prescribe. This applies squarely
to the present case. The complaint filed by respondents in the court a quo
was for the reconveyance of the subject property to the estate of Rosalia
since the deeds of sale were simulated and fictitious. The complaint
amounts to a declaration of nullity of a void contract, which is
imprescriptible. Hence, respondents’ cause of action has not prescribed.

4. Lastly, petitioner in her memorandum seeks to expunge the testimony of


Rosa Santos-Carreon before the trial court in view of Sec. 23, Rule 130 of
the Revised Rules of Court, otherwise known as the “Dead Man’s Statute.”
It is too late for petitioner, however, to invoke said rule. The trial court in
its order dated February 5, 1990, denied petitioner’s motion to disqualify
respondent Rosa as a witness. Petitioner did not appeal therefrom. Trial
ensued and Rosa testified as a witness for respondents and was cross-
examined by petitioner’s counsel. By her failure to appeal from the order
allowing Rosa to testify, she waived her right to invoke the dead man’s
statute. Further, her counsel cross-examined Rosa on matters that occurred
during Salvador’s lifetime. In Goñi vs. CA, 144 SCRA 222, 231 (1986), we
held that protection under the dead man’s statute is effectively waived when
a counsel for a petitioner cross-examines a private respondent on matters
occurring during the deceased’s lifetime. The Court of Appeals cannot be
faulted in ignoring petitioner on Rosa’s disqualification.
59. EXPRESSCREDIT* FINANCING CORPORATION, Petitioner,vs.
SPS. MORTON AND JUANITA VELASCO, Respondents.

G.R. No. 156033 October 20, 2005

FACTS:

The Velasco spouses bought a parcel of land in Diliman, QC from the Garcia
spouses. Since the house on the property was still under construction, the
lot was still covered by the mother title and had no separate title yet. The
sellers promised to give the title after the construction was completed.

The spouses Velasco moved in and had applied for telephone connection and
insurance. However, they still haven’t received the title from the spouses
Garcia despite repeated demands. Thus, they went to the Register of Deeds
in QC and found out that the property has been mortgaged to Express Credit
Financing one year after the Garcia spouses sold the house to them.

The Velascos filed a case for Quieting of Title. A notice of lis pendens was
attached to the title. However, despite this notice and writ of preliminary
injunction, Express Credit still foreclosed the property and bought it in a
public auction as the highest bidder. Thereafter, it secured a certificate of
title in its own name.

RTC: Ruled that Express Credit is a purchaser in good faith. The purchaser is
not required to go beyond the certificate of title. However, Express Credit
should reimburse the spouses Velasco for the amount spent in purchasing
the house.

CA: Reversed the RTC ruling.

Issue:
Whether or not Express Credit has a preferential right over the property for
being a purchaser in good faith and for registering the property first.
Ruling:
NO. This is a case of double sale under Art. 1544. However, the rules
provided in the that article only applies to purchasers in good faith. Express
Credit is not a purchaser in good faith.
Express Credit justifies its acquisition of the property by saying that when it
was mortgaged, the previous sale of the land was not annotated on the title
and so its purchase was in good faith. It is true that a person dealing with
the owner of registered land is not bound to go beyond the certificate of
title. He may rely on the notices of the encumbrances on the property
annotated on the certificate of title or absence of any annotation.
However, we note that the Garcia spouses are unlike other mortgagors.
They are in the business of constructing and selling townhouses and are past
masters in real estate transactions. Further, petitioner is in the business of
extending credit to the public, including real estate loans. In both these
businesses, it devolves upon both, greater charge than ordinary buyers or
encumbrancers for value, who are not in such venture. It is standard in their
business, as a matter of due diligence required of banks and financing
companies, to ascertain whether the property being offered as security for
the debt has already been sold to another to prevent injury to prior innocent
buyers. They also have the resources to ascertain any encumbrances over
the properties they are dealing with.
The caretaker of the house already told the investigators that the house was
already sold to the Velasco spouses. A purchaser cannot close his eyes to
facts which should put a reasonable man upon his guard and claim that he
acted in good faith under the belief that there was no defect in the title of
the vendor.

WHEREFORE, the assailed Decision dated August 20, 2002 and Resolution
dated November 12, 2002 of the Court of Appeals in CA-G.R. CV No. 56491
are AFFIRMED.
SO ORDERED.
60. SPOUSES TANGLAO, vs. SPOUSES PARUNGAO G.R. No. 166913,
05 October 2007

FACTS: 

Spouses Parungao, purchased from Spring Homes 7 Subdivision Lots in


Laguna. Respondents made a down payment, leaving a balance exclusive of
interest. Respondents introduced improvements on the lots. Under the terms
of the Contracts to Sell signed by respondents and Spring Homes, the
balance of was to be paid by them within one year from its execution.
Respondents failed to pay the installments.
Later, Spring Homes executed two separate Deeds of Absolute Sale in favor
of spouses Tanglao, petitioners, wherein the former sold to the latter two
lots. It turned out that the lots sold to them were among the lots previously
sold to the spouses Parungao.In a letter, respondents demanded that Spring
Homes deliver to them the corrected Contracts to Sell, as well as the TCTs
covering the lots they purchased.

Meanwhile, petitioners took possession of the two lots they bought. They
forcibly opened the steel gate as well as the doors of the buildings and
entered the premises.
When informed of these events, respondents demanded an explanation from
Spring Homes; it apologized and promised she would settle the matter with
petitioners. However, the controversy was not settled.

Respondent Spouses Parungao filed with the Housing and Land Use
Regulatory Board (HLURB), a complaint for annulment of deed of sale and/or
return of investment for the seven (7) lots and costs of improvements, plus
interest and damages. Impleaded as respondents were Spring Homes and
petitioners. Despite notice, Spring Homes did not appear during the
hearings.

The HLURB Arbiter rendered a Decision ordering respondent Spring Homes


to pay complainants by way of refund of payments and damages.

Dissatisfied with the ruling, respondents filed a petition for review with the
HLURB Board of Commissioners. The HLURB Board of Commissioners
reversed the Arbiter’s Decision and granted the petition for review.
Petitioners filed a motion for reconsideration, but this was denied by the
HLURB Board of Commissioners. Petitioners then filed an appeal with the
Office of the President, which dismissed their appeal and affirmed the
Decision of the HLURB Board of Commissioners. Petitioners’ MR was also
denied by the said Office.

Eventually, petitioners filed with the CA a petition for review.The CA


rendered its Decision dismissing the petition, hence this petition for Review
on Certiorari

ISSUE: 

Who between the petitioners and respondents have the right of ownership
over the two lots in controversy.

HELD: 

SPOUSES PARUNGAO, the first buyer; PETITION DENIED. The Decision of


the CA is AFFIRMED in toto.
The ownership of immovable property sold to two different persons at
different times is governed by Article 1544 of the Civil Code, 2 which
provides:
Art. 1544. If the same thing should have been sold to different vendees, the
ownership shall be transferred to the person who may have taken
possession thereof in good faith, if it should be movable property.

Should it be immovable property, the ownership shall belong to the person


acquiring it who, in good faith, first recorded it in the Registry of Property.

Should there be no inscription, the ownership shall pertain to the person


who in good faith was first in possession and, in the absence thereof, to the
person who presents the oldest title, provided there is good faith.

In all of these cases, good faith is essential, being the basic premise
of the preferential rights granted to the person claiming ownership
of the immovable.

In Occeña v. Esponilla,5 this Court, speaking through then Associate Justice


(now Chief Justice) Reynato S. Puno, laid down the following rules in the
application of Article 1544:

(1) Knowledge by the first buyer of the second sale cannot defeat the
first buyer’s rights except when the second buyer first registers in good
faith the second sale; and

(2) Knowledge gained by the second buyer of the first sale defeats his
rights even if he is first to register, since such knowledge taints his
registration with bad faith. Differently put, the act of registration by the
second buyer must be coupled with good faith, meaning, the registrant
must have no knowledge of the defect or lack of title of his vendor or
must not have been aware of facts which should put him upon such
inquiry and investigation as might be necessary to acquaint him with the
defects in the title of his vendor.

At the time of the second sale to petitioners by Spring Homes, there were
already occupants and improvements on the two lots in question. These
facts should have put petitioners on their guard. Settled is the rule that a
buyer of real property in possession of persons other than the seller
must be wary and should investigate the rights of those in
possession, for without such inquiry the buyer can hardly be
regarded as a buyer in good faith and cannot have any right over the
property.

As the petitioners cannot be considered buyers in good faith, they cannot


rely upon the indefeasibility of their TCTs in view of the doctrine that the
defense of indefeasibility of a torrens title does not extend to transferees
who take the certificate of title in bad faith.

Considering that respondents who, in good faith, were first in possession of


the subject lots, we rule that the ownership thereof pertains to them

NOTES:
A purchaser in good faith or innocent purchaser for value is one who buys
property and pays a full and fair price for it at the time of the purchase or
before any notice of some other person’s claim on or interest in it. 7 The
burden of proving the status of a purchaser in good faith lies upon him who
asserts that status and it is not sufficient to invoke the ordinary presumption
of good faith, that is, that everyone is presumed to have acted in good faith
61. RICARDO CHENG, petitioner, vs. RAMON B. GENATO and
ERNESTO R. DA JOSE & SOCORRO DA JOSE, respondents., G.R. No.
129760, 29 December 1998

FACTS:
Respondent Genato entered a contract to sell to spouses Da Jose pertaining
to his property in Bulacan. The contract made in public document states that
the spouses shall pay the down payment and 30 days after verifying the
authenticity of the documents, they shall pay the remaining purchase price.

Da Jose spouses was not able to finish verifying the documents and as such
asked for a 30 day extension. Pending the extension and without notice to
the spouses, Genato made a document for the annulment of the contract.

Petitioner Cheng expressed interest over the property and paid 50K check
with the assurance that the contract between Genato and the spouses Da
Jose will be annulled. Da Jose spouses protested with the annulment and
persuaded Genato to continue the contract. Genato returned the check to
Cheng and hence, this petition.

Issue:

Whether or not the contract between the parties is not just a contract to sell
but a conditional contract of sale which give him better rights, thus
precluding the application of Art. 1544.

HELD:
The contract between Genato and spouses Da Jose was a contract to sell
which is subject to a suspensive condition. Thus, there will be no contract to
speak of, if the obligor failed to perform the suspensive condition which
enforces a juridical relation. Obviously, the foregoing jurisprudence cannot
be made to apply to the situation in the instant case because no default can
be ascribed to the Da Jose spouses since the 30-day extension period has
not yet expired.

Even assuming that the spouses defaulted, the contract also cannot be
validly rescinded because no notice was given to them. Thus, Cheng's
contention that the Contract to Sell between Genato and the Da Jose
spouses was rescinded or resolved due to Genato's unilateral rescission finds
no support in this case.

The contract between Genato and Cheng is a contract to sell not a contract
of sale. But But even assuming that it should be treated as a conditional
contract of sale, it did not acquire any obligatory force since it was subject to
a suspensive condition that the earlier contract to sell between Genato and
the Da Jose spouses should first be cancelled or rescinded.

Art.1544 should apply because for not only was the contract between herein
respondents first in time; it was also registered long before petitioner's
intrusion as a second buyer (PRIMUS TEMPORE, PORTIOR JURE). (Spouses
made annotation on the title of Genato). Since Cheng was fully aware, or
could have been if he had chosen to inquire, of the rights of the Da Jose
spouses under the Contract to Sell duly annotated on the transfer
certificates of titles of Genato, it now becomes unnecessary to further
elaborate in detail the fact that he is indeed in bad faith in entering into such
agreement.
62. AGRICULTURAL AND HOME EXTENSION DEVELOPMENT GROUP,
represented by Nicasio D. Sanchez, Sr., substituted by Milagros S.
Bucu, Petitioner, vs
COURT OF APPEALS, and LIBRADO CABAUTAN, Respondents., G.R.
No. 92310. September 3, 1992

SUMMARY
Spouses Andres and Josefa sold the land to Gundran. He did not have it
registered because there was a notice of lis pendens on the title. The
spouses then sold the land to another person, Cabautan who had the land
registered in his name. The RTC, CA and SC ruled that Art. 1544 applies and
Cabautan is not a purchaser in bad faith. The notice of lis pendens is only
notice to third persons that their interest in the land is subject to the result
of the pending suit. Cabautan took the risk, and when the lis pendens was
subsequently cancelled, he acquired the land free from any liens or
encumberances.

FACTS:

On March 29, 1972, the spouses Andres Diaz and Josefa Mia sold to Bruno
Gundran a 19- hectare parcel of land in Las Piñas, Rizal. The owner's
duplicate copy of the title was turned over to Gundran. However, he did not
register the Deed of Absolute Sale because he said he was advised in the
Office of the Register of
Deeds of Pasig of the existence of notices of lis pendens on the title.

Gundran entered into a Joint Venture Agreement with Agricultural and Home
Development Group for the subdivision of the land. This agreement was not
annotated in the title.

On August 30, 1976, the spouses Andres Diaz and Josefa Mia again entered
into another contract of sale of the same property with Librado Cabautan,
the herein private respondent.

On September 3, 1976, by virtue of an order of the Court of First Instance of


Rizal, a new owner's copy of the certificate of title was issued to the Diaz
spouses, who had alleged the loss of their copy. On that same date, the
notices of lis pendens annotated on TCT No. 287416 were canceled and the
Deed of Sale in favor of private respondent Cabautan was recorded.

On March 14, 1977, Gundran instituted an action for reconveyance against


Cabautan and Josefa Mia seeking the cancellation of TCT No. 33850/T-172
and the issuance of a new certificate of title in his name.

On August 31, 1977, the petitioner Agricultural and Home Development


Group, filed a complaint in intervention with substantially the same
allegations and prayers as that in Gundran's complaint.

Gundran and Agricultural & Home claim that Cabautan was a purchaser in
bad faith because he was fully aware of the notices of lis pendens at the
back of TCT No. 287416 and of the earlier sale of the land to Gundran.

RTC: Dismissed Gundran and Agricultural & Home’s complaint for lack of
merit.
CA: Affirmed RTC decision with the modification that Josefa Mia was ordered
to pay Gundran the sum of P90,000.00, with legal interest from September
3, 1976, plus the costs of suit.

ISSUE:

1. W/N Art. 1544 applies in this case


2. W/N Cabautan was a purchaser in bad faith

RULING:

1. YES.
It is not disputed that the first sale to Gundran was not registered while the
second sale to
Cabautan was registered. Following the above-quoted provision, the courts
below were justified in according preferential rights to Cabautan, who had
registered the sale in his favor, as against Gundran, Agricultural & Home's
co-venturer whose right to the same property had not been recorded.

2. NO. Cabautan is not a purchaser in bad faith.An examination of TCT No.


287416 discloses no annotation of any sale, lien, encumbrance or adverse
claim in favor of Gundran or Agricultural & Home.

While it is true that notices of lis pendens in favor of other persons were
earlier inscribed on the title, these did not have the effect of establishing a
lien or encumbrance on the property affected. Their only purpose was to
give notice to third persons and to the whole world that any interest they
might acquire in the property pending litigation would be subject to the
result of the suit.

Cabautan took this risk. Significantly, three days after the execution of the
deed of sale in his favor, the notices of lis pendens were canceled by virtue
of the orders of the Court of First Instance of Rizal, Branch 23, dated April 1,
1974, and April 4, 1974. Cabautan therefore acquired the land free of any
liens or encumbrances and so could claim to be a purchaser in good faith
and for value

FALLO

The language of Article 1544 is clear and unequivocal. In light of its mandate
and of the facts established in this case, we hold that ownership must be
recognized in the private respondent, who bought the property in good faith
and, as an innocent purchaser for value, duly and promptly registered the
sale in his favor.

WHEREFORE, the petition is DENIED and the questioned decision AFFIRMED


in toto, with costs against the petitioner. SO ORDERED.
63. DOLORITA C. BEATINGO, Petitioner, vs.LILIA BU
GASIS, Respondent., G.R. No. 179641, 09 February 2011

FACTS:

1. Petitioner Dolorita Beatingo filed a Complaint for Annulment and


Cancellation of Sale, Reconveyance, Delivery of Title and Damages
against respondent Lilia Bu Gasis.
2. Petitioner alleged that, on May 19, 1998, she bought a piece of land,
denominated as Lot No. 7219 (hereafter referred to as the subject
property), from Flora G. Gasis (Flora). The subject property was
registered in the name of Flora’s predecessor-in-interest. The sale was
evidenced by a notarized Deed of Absolute Sale.
3. On October 18, 1999, petitioner went to the Register of Deeds to have
the sale registered. She, however, failed to obtain registration as she
could not produce the owners duplicate certificate of title. She, thus, filed
a petition for the issuance of the owners duplicate certificate of title but
was opposed by respondent, claiming that she was in possession of the
Original Certificate of Title (OCT) as she purchased the subject property
from Flora on January 27, 1999, as evidenced by a Deed of Sale.
4. This prompted petitioner to file the Complaint, insisting that she is the
rightful owner of the subject property. She also maintained that
respondent had been keeping the OCT despite knowledge that petitioner
is the rightful owner. She further accused respondent of inducing Flora to
violate the contract with her, which caused her damage, prejudice,
mental anguish, and serious anxiety.
5. On the other hand, respondent claimed that she purchased the subject
property from Flora without knowledge of the prior sale of the same
subject property to petitioner, which makes her an innocent purchaser for
value. Respondent denied having induced Flora to violate her contract
with petitioner as she never knew the existence of the alleged first
contract. Lastly, respondent declared that, upon payment of the purchase
price, she immediately occupied the subject property and enjoyed its
produce.
6. The RTC decided in favor of respondent on the basis of the testimonial
and documentary evidence.
7. The RTC considered the controversy as one of double sale and, in
resolving the issues raised by the parties, it applied the rules laid down in
Article 1544 of the Civil Code.
8. As opposed to petitioners admission that she did not pay the purchase
price in full and that she did not acquire possession of the subject
property because of the presence of tenants on it, the court gave more
weight to respondent’s evidence showing that she immediately acquired
possession of the subject property and enjoyed its produce upon full
payment of the purchase price.
9. Since the two sales that of petitioner and that of respondent were not
registered with the Registry of Property, the RTC held that whoever was
in possession had the better right. Hence, it decided in favor of
respondent.

Issue:
Whether or not Gasis has the better right to the property?

RULING:
Respondent Gasis has the better right
 The present controversy is a clear case of double sale, where the seller
sold one property to different buyers, first to petitioner and later to
respondent.
 In determining who has a better right, the guidelines set forth in Article
1544 of the Civil Code apply. Article 1544 states:
 Art. 1544. If the same thing should have been sold to different
vendees, the ownership shall be transferred to the person who may
have first taken possession thereof in good faith, if it should be
movable property.

Should it be immovable property, the ownership shall belong to the


person acquiring it who in good faith first recorded it in the Registry
of Property.

Should there be no inscription, the ownership shall pertain to the


person who in good faith was first in possession; and, in the
absence thereof, to the person who presents the oldest title,
provided there is good faith.
 Admittedly, the two sales were not registered with the Registry of
Property. Since there was no inscription, the next question is who,
between petitioner and respondent, first took possession of the subject
property in good faith. As aptly held by the trial court, it was respondent
who took possession of the subject property and, therefore, has a better
right.
 Petitioner insists that, upon the execution of the public instrument (the
notarized deed of sale), she already acquired possession thereof, and
thus, considering that the execution thereof took place ahead of the
actual possession by respondent of the subject property, she has a better
right.
 We do not agree.
 Indeed, the execution of a public instrument shall be equivalent to the
delivery of the thing that is the object of the contract.
 However, the Court has held that the execution of a public instrument
gives rise only to a prima facie presumption of delivery. It is deemed
negated by the failure of the vendee to take actual possession of the land
sold.
 In this case, though the sale was evidenced by a notarized deed of sale,
petitioner admitted that she refused to make full payment on the
subject property and take actual possession thereof because of
the presence of tenants on the subject property.
 Clearly, petitioner had not taken possession of the subject
property or exercised acts of dominion over it despite her
assertion that she was the lawful owner thereof.
 Respondent, on the other hand, showed that she purchased the subject
property without knowledge that it had been earlier sold by Flora to
petitioner. She had reason to believe that there was no defect in her title
since the owners duplicate copy of the OCT was delivered to her by the
seller upon full payment of the purchase price.
 She then took possession of the subject property and exercised acts of
ownership by collecting rentals from the tenants who were occupying it.
Hence, the RTC is correct in declaring that respondent has a better right to
the subject property.
WHEREFORE, premises considered, the petition is DENIED for lack of merit.
The Court of Appeals Resolutions dated June 27, 2007 and August 13, 2007
in CA-G.R. CEB-CV No. 01624 are AFFIRMED.
SO ORDERED.
64. NORKIS DISTRIBUTORS, INC., petitioner,vs. THE COURT OF
APPEALS & ALBERTO NEPALES, respondents. G.R. No. 91029            
February 7, 1991

FACTS:
Petitioner Norkis Distributors, Inc. is the distributor of Yamaha motorcycles
in Negros Occidental. On September 20, 1979, private respondent Alberto
Nepales bought from the Norkis Bacolod branch a brand new Yamaha
Wonderbike motorcycle Model YL2DX. The price of P7,500.00 was payable
by means of a Letter of Guaranty from the DBP, which Norkis agreed to
accept. Credit was extended to Nepales for the price of the motorcycle
payable by DBP upon release of his motorcycle loan. As security for the loan,
Nepales would execute a chattel mortgage on the motorcycle in favor of
DBP. Petitioner issued a sales invoice which Nepales signed in conformity
with the terms of the sale. In the meantime, however, the motorcycle
remained in Norkis’ possession. On January 22, 1980, the motorcycle was
delivered ¬to a certain Julian Nepales, allegedly the agent of Alberto
Nepales. The motorcycle met an accident on February 3, 1980 at
Binalbagan, Negros Occidental. An investigation conducted by the DBP
revealed that the unit was being driven by a certain Zacarias Payba at the
time of the accident. The unit was a total wreck was returned.

On March 20, 1980, DBP released the proceeds of private respondent’s


motorcycle loan to Norkis in the total sum of P7,500. As the price of the
motorcycle later increased to P7,828 in March, 1980, Nepales paid the
difference of P328 and demanded the delivery of the motorcycle. When
Norkis could not deliver, he filed an action for specific performance with
damages against Norkis in the RTC of Negros Occidental. He alleged that
Norkis failed to deliver the motorcycle which he purchased, thereby causing
him damages. Norkis answered that the motorcycle had already been
delivered to private respondent before the accident, hence, the risk of loss
or damage had to be borne by him as owner of the unit.

ISSUE:
Whether or not there has been a transfer of ownership of the motorcycle to
Alberto Nepales.

HELD:
No.The issuance of a sales invoice does not prove transfer of ownership of
the thing sold to the buyer. An invoice is nothing more than a detailed
statement of the nature, quantity and cost of the thing sold and has been
considered not a bill of sale. In all forms of delivery, it is necessary that the
act of delivery whether constructive or actual, be coupled with the intention
of delivering the thing. The act, without the intention, is insufficient. When
the motorcycle was registered by Norkis in the name of private respondent,
Norkis did not intend yet to transfer the title or ownership to Nepales, but
only to facilitate the execution of a chattel mortgage in favor of the DBP for
the release of the buyer’s motorcycle loan.

Article 1496 of the Civil Code which provides that “in the absence of an
express assumption of risk by the buyer, the things sold remain at seller’s
risk until the ownership thereof is transferred to the buyer,” is applicable to
this case, for there was neither an actual nor constructive delivery of the
thing sold, hence, the risk of loss should be borne by the seller, Norkis,
which was still the owner and possessor of the motorcycle when it was
wrecked. This is in accordance with the well¬ known doctrine of res perit
domino.
65. JAIME GUINHAWA, Petitioners,vs. PEOPLE OF THE
PHILIPPINES, Respondent. G.R. No. 162822 August 25, 2005

FACTS: 
Jaime Guinhawa, a seller of vehicles, sold a van to Spouses Ralph and
Josephine Silo, which he represented as “brand new” when he knew in fact
that it is not since it was already figured in an accident and he just had it
repaired. When the Spouses Ralph and Josephine Silo discovered that the
van was not in fact brand new, they filed a complaint for violation of
paragraph 1, Art. 318 of the RPC, or the crime of Other Deceits.

ISSUE: 
Whether or not Jaime Guinhawa shall be held liable for violation of
paragraph 1, Art. 318 of the RPC, or the crime of Other Deceits.

HELD:
 Yes. The false or fraudulent representation by a seller that what he offers
for sale is brand new is one of those deceitful acts envisaged in paragraph 1,
Art. 318 of the RPC. This provision includes any kind of conceivable deceit
other than those enumerated in Arts. 315 to 317 of the RPC. It is intended
as the catchall provision for that purpose with its broad scope and
intendment. 
66. JAIME D. ANG, Petitioner vs. COURT OF APPEALS AND BRUNO
SOLEDAD, Respondents.

Even under the principle of solutioindebiti, Ang cannot recover from Soledad
the amount he paid BA Finance since Ang settled the mortgage debt on his
own volition and that Soledad did not benefit therein, the latter not being
the one who mortgaged the vehicle.

Facts:

Under a “car-swapping” scheme, Bruno Soledad sold his Mitsubishi GSR


sedan 1982 model to Jaime Ang. For his part, Ang conveyed to Soledad his
Mitsubishi Lancer model 1988. Ang, a buyer and seller of used vehicles, later
offered the Mitsubishi GSR for sale through Far Eastern Motors, a second-
hand auto display center. The vehicle was eventually sold to Paul Bugash.
Before the deed could be registered in Bugash‘s name, however, the vehicle
was seized by virtue of a writ of replevin on account of the alleged failure of
Ronaldo Panes, the owner of the vehicle prior to Soledad, to pay the
mortgage debt constituted thereon.

To secure the release of the vehicle, Ang paid BA Finance. Soledad refused
to reimburse, despite repeated demands, drawing Ang to charge him for
Estafa with abuse of confidence. By Resolution, the City Prosecutor‘s Office
dismissed the complaint for insufficiency of evidence, drawing Ang to file for
consecutive complaints for damages against Soledad before the Regional
Trial Court (RTC) of Cebu City. Subsequently, the RTC rendered judgment in
favor of Ang “for the sake of justice and equity, and in consonance with the
salutary principle of non-enrichment at another‘s expense. The RTC then
ordered Soledad to pay Ang the amount the latter paid to BA Finance.

Soledad then appealed to the Appellate Court, which reverses the decision of
the RTC. The Court of Appeals dismissed Ang‘s petition on the ground that
the filing of said complaint seeking the awarding of damages for breach of
warranty has already prescribed.

Hence, this petition to the High Court.

ISSUE:

1) Whether or not Ang‘s cause of action had not yet prescribed when he filed
the complaint
2) Whether or not Ang can recover from Soledad the amount he paid BA
Finance on account of the mortgage debt

HELD:

First Issue

The resolution of the sole issue of whether the complaint had prescribed
hinges on a determination of what kind of warranty is provided in the Deed
of Absolute Sale subject of the present case.

A warranty is a statement or representation made by the seller of goods,


contemporaneously and as part of the contract of sale, having reference to
the character, quality or title of the goods, and by which he promises or
undertakes to insure that certain facts are or shall be as he then represents
them. Warranties by the seller may be express or implied. Art. 1546 of the
Civil Code defines express warranty – Any affirmation of fact or any promise
by the seller relating to the thing is an express warranty if the natural
tendency of such affirmation or promise is to induce the buyer to purchase
the same, and if the buyer purchases the thing relying thereon. On the other
hand, an implied warranty is that which the law derives by application or
inference from the nature of the transaction or the relative situation or
circumstances of the parties, irrespective of any intention of the seller to
create it.

The ruling in Engineering & Machinery Corporation vs. Court of Appeals


states that “the prescriptive period for instituting actions based on a breach
of express warranty is that specified in the contract, and in the absence of
such period, the general rule on rescission of contract, which is four years
(Article 1389, Civil Code).”

As for actions based on breach of implied warranty, the prescriptive period


is, under Art. 1571 (warranty against hidden defects of or encumbrances
upon the thing sold) and Art. 1548 (warranty against eviction), six months
from the date of delivery of the thing sold.

In declaring that he owned and had clean title to the vehicle at the time the
Deed of Absolute Sale was forged, Soledad gave an implied warranty of title.
In pledging that he “will defend the same from all claims or any claim
whatsoever and will save the vendee from any suit by the government of the
Republic of the Philippines,” Soledad gave a warranty against eviction.

Given Ang‘s business of buying and selling used vehicles, he could not have
merely relied on Soledad‘s affirmation that the car was free from liens and
encumbrances. He was expected to have thoroughly verified the car‘s
registration and related documents.

Since what Soledad, as seller, gave was an implied warranty, the


prescriptive period to file a breach thereof is six months after the delivery of
the vehicle, following Art. 1571. But even if the date of filing of the action is
reckoned from the date petitioner instituted his first complaint for damages
on November 9, 1993, and not on July 15, 1996 when he filed the complaint
subject of the present petition, the action just the same had prescribed, it
having been filed 16 months after July 28, 1992, the date of delivery of the
vehicle.

Second Issue
On the merits of his complaint for damages, even if Ang invokes breach of
warranty against eviction as inferred from the second part of the earlier-
quoted provision of the Deed of Absolute Sale, the following essential
requisites for such breach: (1) The purchaser has been deprived of the
whole or part of the thing sold; (2) This eviction is by a final judgment; (3)
The basis thereof is by virtue of a right prior to the sale made by the vendor;
and (4) The vendor has been summoned and made co-defendant in the suit
for eviction at the instance of the vendee, have not been met. For one, there
is no judgment which deprived Ang of the vehicle. For another, there was no
suit for eviction in which Soledad as seller was impleaded as co-defendant at
the instance of the vendee.

Finally, even under the principle of solutioindebiti which the RTC applied,
Ang cannot recover from Soledad the amount he paid BA Finance. For, as
the appellate court observed, Ang settled the mortgage debt on his own
volition under the supposition that he would resell the car. It turned out that
he did pay BA Finance in order to avoid returning the payment made by the
ultimate buyer Bugash. It need not be stressed that Soledad did not benefit
from Ang‘s paying BA Finance, he not being the one who mortgaged the
vehicle, hence, did not benefit from the proceeds thereof.
67. NUTRIMIX FEEDS CORPORATION, Petitioner, v. COURT OF
APPEALS and SPOUSES EFREN AND MAURA
EVANGELISTA, Respondents. [G.R. NO. 152219 : October 25, 2004]

Facts:
On April 5, 1993, the Spouses Efren and Maura Evangelista, the respondents
herein, started to directly procure various kinds of animal feeds from
petitioner Nutrimix Feeds Corporation.
The petitioner gave the respondents a credit period of thirty to forty-five
days to postdate... checks to be issued in payment for the delivery of the
feeds.
When the above-mentioned checks were deposited at the petitioner's
depository bank, the same were, consequently, dishonored because
respondent Maura Evangelista had already closed her account.
On December 15, 1993, the petitioner filed with the Regional Trial Court of
Malolos, Bulacan, a complaint, docketed as Civil Case No. 1026-M-93,
against the respondents for sum of money and damages with a prayer for
issuance of writ of preliminary attachment.
On January 19, 1994, the respondents also lodged a complaint for damages
against the petitioner, docketed as Civil Case No. 49-M-94, for the untimely
and unforeseen death of their animals supposedly effected by the
adulterated animal feeds the petitioner sold to them.
A joint trial thereafter ensued.
After due consideration of the evidence presented, the trial court ruled in
favor of the petitioner.
Issues:
The threshold issue is whether or not there is sufficient evidence to hold the
petitioner guilty of breach of warranty due to hidden defects.
Ruling:
Art. 1561. The vendor shall be responsible for warranty against hidden
defects which the thing sold may have, should they render it unfit for the
use for which it is intended, or should they diminish its fitness for such use
to such an extent that, had the vendee... been aware thereof, he would not
have acquired it or would have given a lower price for it; but said vendor
shall not be answerable for patent defects or those which may be visible, or
for those which are not visible if the vendee is an expert who, by reason of
his trade or... profession, should have known them.
A hidden defect is one which is unknown or could not have been known to
the vendee.
Under the law, the requisites to recover on account of hidden defects are as
follows:
(a) the defect must be hidden;
(b) the defect must exist at the time the sale was made;
(c) the defect must ordinarily have been excluded from the contract;
(d) the defect, must be important (renders thing UNFIT or considerably
decreases FITNESS);
(e) the action must be instituted within the statute of limitations.
Tracing the defect to the petitioner requires some evidence that there was
no tampering with, or changing of the animal feeds.
The nature of the animal feeds makes it necessarily difficult for... the
respondents to prove that the defect was existing when the product left the
premises of the petitioner.
A review of the facts of the case would reveal that the petitioner delivered
the animal feeds, allegedly containing rat poison, on July 26, 1993; but it is
astonishing that the respondents had the animal feeds examined only on
October 20, 1993, or barely three months after their... broilers and hogs had
died.
In a span of three months, the feeds could have already been contaminated
by outside factors and subjected to many conditions unquestionably beyond
the control of the petitioner.
There is simply no evidence to show that the feeds given to the animals on
July 26 and 27, 1993 were identical to those submitted to the expert
witnesses in October 1993.
Likewise, there was evidence tending to show that the respondents
combined different kinds of animal feeds and that the mixture was given to
the animals.
Even more surprising is the fact that during the meeting with Nutrimix
President Mr. Bartolome, the respondents claimed that their animals were
plagued by disease, and that they needed more time to settle their
obligations with the petitioner.
It was only after a few months that... the respondents changed their
justification for not paying their unsettled accounts, claiming anew that their
animals were poisoned with the animal feeds supplied by the petitioner.
The volte-face of the respondents deserves scant consideration for having
been conjured as... a mere afterthought.
In essence, we hold that the respondents failed to prove that the petitioner
is guilty of breach of warranty due to hidden defects. It is, likewise,
rudimentary that common law places upon the buyer of the product the
burden of proving that the seller of the product breached its... warranty.
It must be stressed, however, that the remedy against violations of warranty
against hidden defects is either to withdraw from the contract
(accionredhibitoria) or to demand a proportionate reduction of the price
(accionquantiminoris), with damages... in either case.
WHEREFORE, in light of all the foregoing, the petition is GRANTED.
Principles:
In essence, we hold that the respondents failed to prove that the petitioner
is guilty of breach of warranty due to hidden defects. It is, likewise,
rudimentary that common law places upon the buyer of the product the
burden of proving that the seller of the product breached its... warranty.
It must be stressed, however, that the remedy against violations of warranty
against hidden defects is either to withdraw from the contract
(accionredhibitoria) or to demand a proportionate reduction of the price
(accionquantiminoris), with damages... in either case.
68. Carlos De Guzman vs. Toyota Cubao, G.R. No. 141480 November
29, 2006

SUBJECT MATTER: Representations & Warranties; Implied warranty


against hidden defects; prescription

FACTS:
De Guzman purchased a Hilux from Toyota Cubao. Almost a year after, the
Hilux’s engine cracked while traversing Marcos Highway during a heavy rain.
De Guzman filed a complaint for damages 17 months after the purchase.
Toyota filed a motion to dismiss on the ground that based on Art. 1571, the
action has prescribed as the case was filed more than 6 months after the
purchase. The SC agreed that the action was already barred by prescription.
DOCTRINES:
Where the cause of action is to hold a seller of a vehicle for a breach of
implied warranty for having sold a vehicle with defective engine, the action
should be filed within 6 months from the delivery of the thing sold.
FACTS:
Petitioner Carlos B. De Guzman
Respondent Toyota Cubao, Inc.

On November 27, 1997, petitioner purchased from respondent a brand new


white Toyota Hi-Lux 2.4 SS double cab motor vehicle, 1996 model in the
amount of P508,000.
Petitioner made a down payment of P152,400, leaving a balance of
P355,600 which was payable in 36 months with 54% interest.
The vehicle was delivered 2 days after.
On October 18, 1998, almost 11 months after the purchase and after 12,000
km of use, petitioner demanded the replacement of the engine of the vehicle
because it developed a crack after traversing Marcos Highway during a
heavy rain.
Petitioner asserted that respondent should replace the engine with a new
one based on an implied warranty.
Respondent countered that the alleged damage on the engine was not
covered by a warranty.
On April 20, 1999, 17 months after the purchase, petitioner filed a complaint
for damages against respondent with the RTC.
Respondent moved to dismiss the case on the ground that under Art. 1571
CC, petitioner’s cause of action had prescribed as the case was filed more
than 6 months from the date the vehicle was sold and/or delivered.
RTC – granted the motion to dismiss. The contract of sale of the subject
pick-up carried an implied warranty that it was free from any hidden faults
or defects, or any charge or encumbrance not declared or known to the
buyer. The prescriptive period thereof is 6 months under Art. 1571 CC.
Petitioner’s motion for reconsideration was likewise denied.

ISSUE/S:
1. WON the petitioner’s action is barred by prescription. (YES)
HOLDING/RATIO:
1. Petitioner’s argument:
The dismissal is erroneous because the applicable provision is Art. 169
of RA 7394 (The Consumer Act of the Philippines) and not Art. 1571
CC.
In seeking enforcement of the contract, i.e. that respondent should
either replace the vehicle or its engine with a new one, he cited Art.
169 RA 7394 to make the suit come within the purview of the 2-year
prescriptive period.
Also, the cause of action based on quasi-delict, prescribes in 4 years,
thus his cause of action has not yet prescribed.

Respondent’s argument:
Cause of action was already barred by the statute of limitations under
Art. 1571 CC for having been filed more than 6 months. Art. 169 RA
7394 does not apply.

Petitioner’s argument is erroneous.


In the absence of an existing express warranty on the part of the
respondent, as in this case, the allegations in the petitioner’s
complaint for damages were clearly anchored on the enforcement of
an implied warranty against hidden defects, i.e., that the engine of the
vehicle which respondent had sold to the petitioner was not defective.
Petitioner wants to hold the respondent responsible for breach of
implied warranty for having sold a vehicle with a defective engine.
However, the petitioner should have exercised the right within 6
months from the delivery of the thing sold.
Since the petitioner filed the complaint on April 20, 1999 or more than
19 months counting from November 29, 1997 (date of purchase), his
cause of action had become time barred.
Even if the complaint is made to fall under RA 7394 Art. 68 (f)(2) in
relation to Art. 169, Art. 68 (e) par. 2 would justify that the same
should still be dismissed since the prescriptive period for implied
warranty thereunder which is one year, had likewise lapsed.

WHEREFORE, the petition is DENIED for being in violation of the hierarchy of


courts (procedural issue not discussed here), and in any event, for lack of
merit.
69. Spouses Mario and Elizabeth Torcuator vs. Spouses Remegio and
Gloria Bernabe and Spouses Diosdado and Lourdes Salvador G.R. No.
134219 June 08, 2005

Thesubjectpropertyofthepresentcaseisaparceloflandwhichwas
purchasedbyrespondentsspousesDiosdadoandLourdesSalvadorfromdeveloper
,
Ayala Corporation. The sale was subject to several restrictions imposed by
Ayala
Corporation. Salvadors later sold the subject property to the spouses
Remigio and
Gloria Bernabe but in view of the the above-stated restrictions imposed by
Ayala
Corporation, the Salvadors executed a special power of attorney authorizing
the
Bernabes to construct a residential house on the lot and to transfer the title
of
the property in their names. The Bernabes, on the other hand, did not make
any
improvementandproceeded tosellthe subjectpropertyto petitioners,Spouses
MarioandElizabethTorcuator.DueagaintotherestrictionsimposedbyAyala
Corporation, the parties agreed to cancel the deed of sale between the
Salvadors
andBernabesandanewonewasexecutedbytheSalvadorsinfavorofthe
Torcuators. A new SPA was executed by the Salvadors for the Torcuators in
order
for the latter to build a house on the land in question, while an irrevocable
SPA
wasexecutedbytheSalvadorstotheBernabesauthorizingthelattertosell,
transfer and convey, with power of substitution, the subject property.
However, no
payment was tendered by the Torcuators to the Bernabes. Subsequently, the
Bernabes
soldthesubjectlandtoathirdperson,LeonardoAngeles.Asaresult,the
Torcuatorscommencedthe instantactionagainst theBernabesand Salvadorsfor
SpecificPerformanceorRescissionwithDamages.Thetrialcourt,however,
dismissed the complaint onthegroundthatnorealdamagewassufferedbythe
Torcuators. The Torcuatorsappealed to the Courtof Appeals, but theappellate
court upheld the trial court’s conclusion and dismissed the appeal. The CA
ruled,
among others, that the sale between the Bernabes and the Torcuators was
tainted
with serious irregularities and bad faith.
Thesubjectpropertyofthepresentcaseisaparceloflandwhichwas
purchasedbyrespondentsspousesDiosdadoandLourdesSalvadorfromdeveloper
,
Ayala Corporation. The sale was subject to several restrictions imposed by
Ayala
Corporation. Salvadors later sold the subject property to the spouses
Remigio and
Gloria Bernabe but in view of the the above-stated restrictions imposed by
Ayala
Corporation, the Salvadors executed a special power of attorney authorizing
the
Bernabes to construct a residential house on the lot and to transfer the title
of
the property in their names. The Bernabes, on the other hand, did not make
any
improvementandproceeded tosellthe subjectpropertyto petitioners,Spouses
MarioandElizabethTorcuator.DueagaintotherestrictionsimposedbyAyala
Corporation, the parties agreed to cancel the deed of sale between the
Salvadors
andBernabesandanewonewasexecutedbytheSalvadorsinfavorofthe
Torcuators. A new SPA was executed by the Salvadors for the Torcuators in
order
for the latter to build a house on the land in question, while an irrevocable
SPA
wasexecutedbytheSalvadorstotheBernabesauthorizingthelattertosell,
transfer and convey, with power of substitution, the subject property.
However, no
payment was tendered by the Torcuators to the Bernabes. Subsequently, the
Bernabes
soldthesubjectlandtoathirdperson,LeonardoAngeles.Asaresult,the
Torcuatorscommencedthe instantactionagainst theBernabesand Salvadorsfor
SpecificPerformanceorRescissionwithDamages.Thetrialcourt,however,
dismissed the complaint onthegroundthatnorealdamagewassufferedbythe
Torcuators. The Torcuatorsappealed to the Courtof Appeals, but theappellate
court upheld the trial court’s conclusion and dismissed the appeal. The CA
ruled,
among others, that the sale between the Bernabes and the Torcuators was
tainted
with serious irregularities and bad faith.
PRICE
FACTS:
Thesubjectpropertyofthepresentcaseisaparceloflandwhichwas
purchasedbyrespondentsspousesDiosdadoandLourdesSalvadorfromdeveloper
,
Ayala Corporation. The sale was subject to several restrictions imposed by
Ayala
Corporation. Salvadors later sold the subject property to the spouses
Remigio and
Gloria Bernabe but in view of the the above-stated restrictions imposed by
Ayala
Corporation, the Salvadors executed a special power of attorney authorizing
the
Bernabes to construct a residential house on the lot and to transfer the title
of
the property in their names. The Bernabes, on the other hand, did not make
any
improvementandproceeded tosellthe subjectpropertyto petitioners,Spouses
MarioandElizabethTorcuator.DueagaintotherestrictionsimposedbyAyala
Corporation, the parties agreed to cancel the deed of sale between the
Salvadors
andBernabesandanewonewasexecutedbytheSalvadorsinfavorofthe
Torcuators. A new SPA was executed by the Salvadors for the Torcuators in
order
for the latter to build a house on the land in question, while an irrevocable
SPA
wasexecutedbytheSalvadorstotheBernabesauthorizingthelattertosell,
transfer and convey, with power of substitution, the subject property.
However, no
payment was tendered by the Torcuators to the Bernabes. Subsequently, the
Bernabes
soldthesubjectlandtoathirdperson,LeonardoAngeles.Asaresult,the
Torcuatorscommencedthe instantactionagainst theBernabesand Salvadorsfor
SpecificPerformanceorRescissionwithDamages.Thetrialcourt,however,
dismissed the complaint onthegroundthatnorealdamagewassufferedbythe
Torcuators. The Torcuatorsappealed to the Courtof Appeals, but theappellate
court upheld the trial court’s conclusion and dismissed the appeal. The CA
ruled,
among others, that the sale between the Bernabes and the Torcuators was
tainted
with serious irregularities and bad faith.
FACTS:
The subject property of the present case is a parcel of land which is
purchased by respondents Spouses Diosdado and Lourdes Salvador from
developer of Ayala Corporation. The sale was subject to several restrictions
imposed by Ayala Corporations. Salvadors later sold the subject property to
the spouses Remigio and Gloria Bernabe but in view of the above-state
restrictions imposed by Ayala Corporation, the Salvadors executed a Special
Power of Attorney authorizing the Bernabes to construct a residential house
on the lot and to transfer the title of the property in their names. The
Bernabes, on the other hand, did not make any improvement and proceeded
to sell the subject property to petitioners, Spouses Mario and Elizabeth
Torcuator.
Due again to the restrictions imposed by Corporation, the parties agreed to
cancel the deed of sale between the Salvadors and Bernabes and a new one
was executed by the Salvadors in favour of Torcuators. A new SPA was
executed by the Salvadors for the Torcuators in order for the latter to build a
house on the land in question, while an irrevocable SPA was executed by the
Salvadors to the Bernabes authorizing the latter to transfer and convey, with
power of substitution, the subject property. However, no payment was
tendered by the Torcuators to the Bernabes.
Subsequently, the Bernabes sold the subject land to a third person,
Leonardo Angeles. As a result, Torcuators commenced the instant action
against the Bernabes and Salvadors for Specific Performance or Rescission
with Damages.
The trial court, however, dismissed the complaint on the ground that no real
damage was suffered by Torcuators. The Torcuators appealed to the CA, but
the appellate Court upheld the trial court’s conclusion and dismissed the
appeal. The CA ruled, among others, that the sale between the Bernabes
and the Torcuators was tainted with serious irregularities and bad faith.

ISSUE:
Whether or not the Torcuators are entitled to rescission of the contract of
sale
RULING:
The Supreme Court says No. Remarkably, the records are bereft of an
indication that petitions ever attempted to tender payment or consign the
purchase price as required by law. The complaint filed by petitioners makes
mention at all of a tender of payment or consignation having been made,
much less that petitioners are willing and ready to pay the purchase price.
Petitioners’ averments to the effect that they have sufficient funds to pay for
the property and have been applied for a telegraphic transfer from their
bank account to the Bernabes’ bank account, uncoupled with actual tender
and consignation, are utterly self-serving.
The trial court correctly noted that petitioners should have consigned the
amount due in court instead of merely sending respondents a letter
expressing interest to push through with the transaction. Mere sending of a
letter by the vendee expressing the intention to pay without the
accompanying payment is not considered a valid tender of payment.
Consignation of the amount due in court is essential in order to extinguish
the obligation to pay and oblige the vendor to convey title.
On this score, even assuming that the agreement was a contract of sale,
respondents may not be compelled to deliver the property and execute the
deed of absolute sale. In cases such as the one before us, which involve the
performance of an obligation and not merely the exercise of a privilege or
right, payment may be effected not by mere tender alone but both by tender
and consignation. The rule is different in cases which involve an exercise of a
right or privilege, such as in an option contract, legal redemption or sale
with right to repurchase wherein mere tender of payment would be sufficient
to preserve the right of privilege. Hence, absent a valid tender of payment
and consignation, petitioners are deemed to have failed to discharge their
obligation to pay.
WHEREFORE, the instant petition is DENIED. Costs against petitioners.
70. Vicente Bareng vs. Honourable Court of Appeals, Patrocinio
Alegria and Agustin Ruiz G.R. No. L-12973 April 25, 1960

FACTS:

Vicente Bareng purchased from respondent Alegria the cinematographic


equipment installed at the Pioneer Theater in Laoag, Ilocos Norte, for the
sum of P15,000. P10,000 of which was paid, and Bareng signed 4
promissory notes for the balance. The first promissory note amounting to
P1,000 was duly paid by Bareng. On February 15, 1952, shortly before the
second note fell due, the other respondent Agustin Ruiz informed Bareng
that he was a co-owner of the equipment in question, and several days later,
Ruiz sent Bareng a telegram instructing him to suspend payments to Alegria
for thebalance of the price as he was not agreeable to the sale. When Alegria
sought to collect the second note on the same day, Bareng only paid P400
and refused to make any more payments on account of Ruiz’s claims.

On March 31, 1952, Ruiz filed suit against Alegria and Bareng for his share
in the price of the cinema equipment. Thereafter in May of the same year,
Alegria and Ruiz reached a compromise wherein the former recognized the
latter as co-owner of the equipment sold to Bareng and promised to pay 2/3
of whatever amount he could recover from the latter. Alegria then sued
Bareng for the amount of P13,500, allegedly the unpaid balance of the price.
But Bareng answered that only P3,600 had not been paid, and prayed for
the rescission of the sale for the supposed violation of Alegria of certain
express warranties as to the quality of the equipment, and asked for
payment of damages for alleged violation of Alegria’s warranty of title.
Bareng added that he is not liable to pay interests to Alegria because he was
justified in suspending payment of the balance of the price of the equipment
from the time he learned of Ruiz’s adverse claims over said equipment,
pursuant to Art. 1590 of the Civil Code.

ISSUE:

Whether or not Bareng is liable to pay interest of the unpaid balance of the
price of the equipment.

HELD:

Bareng is liable to pay interest of the unpaid balance of the price of the
equipment in question. Art. 1590 of the Civil Code provides that: “Should
the vendee be disturbed in the possession or ownership of the thing
acquired, or should he have reasonable grounds to fear such disturbance, by
a vindicatory action or a foreclosure of mortgage, he may suspend the
payment of the price until the vendor has caused the disturbance or danger
to cease, unless the latter gives security for the return of the price in a
proper case, or it has been stipulated that, notwithstanding any such
contingency, the vendee shall be bound to make the payment. A mere act of
trespass shall not authorize the suspension of the payment of the price.”

It is undisputed that petitioner had the right to suspend payment of the


balance of the price of the cinema equipment in question to his vendor from
the time he was informed by Ruiz of the latter’s claims of co-ownership
thereof, especially upon his receipt of Ruiz’s telegram wherein the latter
asserted that he was not agreeable to the sale. However, said right of
Barend ended as soon as “the vendor has caused the disturbance or danger
to cease,” which, in this case, was when Alegria reached a compromise with
Ruiz whereby Ruiz expressed his conformity to the sale to Bareng, subject to
the payment of his share in the price by Alegria. From the time Alegria and
Ruiz reached this settlement, there was no longer any danger of threat to
Bareng’s ownership and full enjoyment of the equipment he bought from
Alegria, by virtue of which Alegria sued petitioner for the unpaid balance.
Bareng admitted his indebtedness in the amount of P3,600, yet he did not
tender payment of said amount nor did he deposit the same in court, but
instead sought for rescission of the sale. It is clear that Bareng was in
default on the unpaid balance of the price of the equipment from the date of
filing of the complaint by Alegria, and under Art. 2209 of the Civil Code, he
must pay legal interests thereon from said date.
71. Spouses Mahusay v. B.E. San Diego Inc., GR# 179675
Facts:
Petitioner spouses Juanito and Francisca Mahusay purchased several lotsin
Aurora Subdivision, Malabon, Metro Manila, owned by respondent B.E. San
Diego, Inc. The transactions were covered by two (2) contracts: Contract to
Sell No.831, executed on May 14, 1973, for the total price of P33,000.00;
and Contract to Sell No. 874 dated August 1, 1975, for the price of
P197,040.00, plus interestof 12% per annum, payable in monthly
installments.
Due to petitionersnonpayment of the monthly amortizations since October
1978, respondent was constrained tofile a case for cancellation of contracts.
The case was dismissed by the trial court for lack of jurisdiction. Thereafter,
a Compromise Agreement was entered into by the parties on October 13,
1989, whereby petitioners agreed to pay respondent the remaining balance
of the purchase price of all the lots in the manner and under the terms
agreed upon by the parties. Petitioners failed to comply withthe terms
embodied in the Compromise Agreement; thus, on April 18, 1990,
respondent filed a Complaint for Specific Performance with the Regional Trial
Court (RTC), Branch 73, Malabon, docketed as Civil Case No. 1433-
MN.Petitioners appealed the decision to the CA on two grounds: (1) it was
the Housing and Land Use Regulatory Board and not the RTC which had
jurisdiction over thesubject matter of the action; and (2) the Compromise
Agreement was unenforceable because it was only Francisca Mahusay who
signed the Agreement on October 13,1989, without the consent of her
husband Juanito Mahusay. In its Decision datedDecember 20, 2001, the CA
upheld the jurisdiction of the RTC.
The CA ratiocinated that respondent s action was one for Specific
Performance with Damages, which isin the nature of ordinary money claims
filed by the unpaid seller against the buyer, that should be litigated in the
regular court. Besides, petitioners were estopped from questioning the
court’s jurisdiction since, by the act of filing an answer and other pleadings,
they were deemed to have submitted themselves to thejurisdiction of the
court. The CA, however, saw merit in the contention that theCompromise
Agreement dated October 13, 1989 was not valid considering that it was
entered into by petitioner Francisca Mahusay alone. Since the Agreement
involved the conjugal properties of petitioners, Francisca could not bind her
husband, who never gave his consent to the Agreement.

Issue:
What is the remedy of the seller of unpaid real property?

Held:
The Court further notes that petitioners are in actual/physical possession of
the properties and enjoying the beneficial use thereof, despite the payment
of only P133,872.76, as of January 30, 1979.19 It would be grossly unfair
for respondent to be deprived of the amount it would have received from the
sale of their properties, while petitioners benefited from the use and
continued possession of the properties even if no payments were made by
them since October 1978. It is abasic rule in law that no one shall unjustly
enrich oneself at the expense of another. Indeed, to allow petitioners to
keep the properties without paying for them in full amounts to unjust
enrichment on their part. The fair market value of the land has tremendously
increased over the past years. It is, therefore, just, fair, and equitable that
petitioners be made to pay interest/penalty for thedelay in their
payments.Finally, the Court notes that this case has dragged on for many
years since 1978. In order to writ fines to this protracted litigation between
the parties, we resolve the case in accordance with jurisprudence on the
matter. Undeniably, theinstant case is a sale of real property where the
purchase price is not paid infull. The unpaid seller s remedy is either an
action to collect the balance or torescind the contract within the time allowed
by law. Since rescission is no longer an option considering that petitioners
have been in possession of the properties for a considerable period of time,
substantial justice dictates that respondent be entitled to receive the unpaid
balance of the purchase price, plus legalinterest thereon.21 In line with our
ruling in Eastern Shipping Lines, Inc. v. Court of Appeals,22 the legal
interest to be paid on the amount shall be 12% perannum, which shall
commence from April 18, 1990, when respondent filed the Complaint for
Specific Performance with the RTC, Branch 73, Malabon, in Civil Case No.
1433-MN, which shall be considered as judicial demand, until the finality of
this Decision. Another 12% interest per annum shall be paid on the amount
due andowing as of and from the date of finality of the Decision until full
payment.
72. ArtemioKatigbakv. Court Of AppealsJan 31, 1962 GR No. L-16480

Facts:

Artemio Katigbak upon reading an advertisement for the sale of the Double
Drum Carco Tractor Winch placed by V. K. Lundberg, owner and operator of
the International Tractor and Equipment Co., Ltd., went to see Lundberg and
inspected the equipment. It was agreed that Katigbak was to purchase the
winch for P12,000.00, payable at P5,000.00 upon delivery and the balance
of P7,000.00 within 60 days. The condition of the sale was that the winch
would be delivered in good condition. Katigbak was apprised that the winch
needed some repairs, which could be done in the shop of Lundberg. It was
then stipulated that the amount necessary for the repairs will be advanced
by Katigbak but deductible from the initial payment of P5,000.00. The
repairs were undertaken and the total of P2,029.85 for spare parts was
advanced by Katigbak for the purpose. For one reason or another, the sale
was not consummated and Katigbak sued Evangelista, Lundberg and the
latter’s company, for the refund of such amount. Lundberg and Evangelista
filed separate Answers to the complaint, the former alleging non-liability for
the amount since the same (obligation for refund) was purely a personal
account between defendant Evangelista and plaintiff Katigbak. Evangelista,
on his part, claimed that while there was an agreement between him and
Katigbak for the purchase and sale of the winch and that Katigbak advanced
the payment for the spare parts, he (Katigbak) refused to comply with his
contract to purchase the same; that as a result of such refusal he
(Evangelista) was forced to sell the same to a third person for only
P10,000.00. The lower court ruled in favor of Katigbak, but the same was
reversed by the CA.

Issue:

Is the lower court correct in ruling in favor of Katigbak?

Held:
No. It was Katigbak who committed a breach of contract. Hence, it follows
that the present action was unjustified and he must be held liable to
appellant Evangelista for attorney’s fees in the sum of P700.00. Also,
pursuant to Hanlon case, when a purchaser of goods upon an executory
contract fails to take delivery and pay the purchase price. The vendor in
such case is entitled to resell the goods. If he is obliged to sell for less than
the contract price, he holds the buyer for the difference. But it has never
been held that there is any need of an action of rescission to authorize the
vendor, who is still in possession, to dispose of the property where the buyer
fails to pay the price and take delivery/ The facts of the case under
consideration are identical to those of the Hanlon case. The herein petitioner
failed to take delivery of the winch, subject matter of the contract and such
failure or breach was, according to the Court of Appeals, attributable to him,
a fact which we are bound to accept under existing jurisprudence. The right
to resell the equipment, therefore, cannot be disputed. It was also found by
theCourt of Appeals that in the subsequent sale of the winch to a third party,
the vendor thereof lost P2,000.00, the sale having been only for P10,000.00,
instead of P12,000.00 as agreed upon, said difference to be borne by the
supposed vendee who failed to take delivery and/or to pay the price.
Of course, petitioner tried to draw a distinction between the Hanlon case and
his case. The slight differences in the facts noted by petitioner are not,
however, to our mode of thinking, sufficient to take away the case at bar
from the application of the doctrine enunciated in the Hanlon case.
73. Borbon II vs. ServicewideG.R. No. 106418 July 11, 1996

FACTS:
The Borbons signed a promissory note where they jointly and severally
promised to pay Pangasinan Auto Mart, Inc. the sum of P122,856.00, to be
payable without need of notice or demand, in installments of P10,238.00
monthly for 12 months. To secure the Promissory Note, the defendants
executed a Chattel Mortgage on 1 brand new 1984 Isuzu, KCD 20 Crew Cab.
The rights of PAMI was later assigned to Filinvest with notice to the Borbons.
Filinvest assigned all its rights over the Promissory Note and the chattel
mortgage to the plaintiff. Because the Borbons did not pay their monthly
installments, Filinvest demanded from the defendants the payment of their
installments due by telegram. The plaintiff attempted to collect by sending a
demand letter to the Borbons which totaled P185,257.80. The Borbonss
claim that what they intended to buy was a jeepney type Isuzu K. C. Cab.
The vehicle that they bought was not delivered. Instead, through
misrepresentation and machination, the PAMI delivered an Isuzu crew cab.
Later the representative of PAMI told the Borbons that their available stock
is an Isuzu Cab but minus the rear body, which the Borbons agreed to
deliver with the understanding that the PAMI will refund the Borbons the
amount of P10,000.00 to have the rear body completed. PAMI was not able
to replace the vehicle until the vehicle delivered was seized by order of this
court. The assignee exercise all the rights of the assignor. The Borbons
further claim that they are not in default of their obligation because the
Pangasinan Auto Mart was first guilty of not fulfilling its obligation in the
contract.

ISSUE:
Whether petitioners could not avoid liability under the promissory note and
the chattel mortgage

HELD:
No. When the seller assigns his credit to another person, the latter is
likewise bound by the same law. Accordingly, when the assignee forecloses
on the mortgage, there can be no further recovery of the deficiency, and the
seller-mortgagee is deemed to have renounced any right thereto. A
contrario, in the event the seller-mortgagee first seeks, instead, the
enforcement of the additional mortgages, guarantees or other security
arrangements, he must then be held to have lost by waiver or non-choice his
lien on the chattel mortgage of the personal property sold by any mortgaged
back to him, although, similar to an action for specific performance, he may
still levy on it.
74. SPOUSES NONATO V. IAC & INVESTOR'S FINANCE CORP 140
SCRA 255 (1985)

FACTS:
In 1976, Spouses RestitutoNonato and Ester Nonato purchased a
volkswagen from the People’s Car Inc on installment basis.
1. To secure their complete payment, Nonato executed a promissory
note and a chattel mortgage in favor of People’s Car Inc.
2. Subsequently, People’s Car Inc assigned its rights and interest over
the note and mortagge in favor of Investor’s Finance Corp (IFC).
3. For failure of the spouses to pay two or more installments, despite
demands, the car was repossessed by IFC.
4. Despite repossession, IFC still demanded from Nonato that they pay
the balance of the price of the car. IFC, then, filed a complaint for the
payment of the price of the car with damages
5. Nonato, in their defense, argued that when the company repossessed
the car, IFC had, by that act, effectively cancelled the sale of the vehicle. As
such, it was barred from exacting the recovery of the unpaid balance of the
purchase price as mandated by Art 1484.
6. The trial court rendered in favor of IFC and ordered the spouses
Nonato pay the balance of the purchase price of the car with interest. CA
affirmed the same.

ISSUE:
WON a vendor or his assignee, who had cancelled the sale of a motor
vehicle for failure of the buyer to pay two or more of the stipulated
installments, may also demand payment of the balance of the purchase price

HELD:
No. The applicable law in the case at bar is Art 1484 which provides that:
In a contract of sale of personal property the price of which is payable in
installments, the vendor may exercise any of the following remedies:
(1) Exact fulfillment of the obligation, should the vendee fail to pay;
(2) Cancel the sale, should the vendee's failure to pay cover two or more
installments;
(3) Foreclose the chattel mortgage on the thing sold, if one has been
constituted, should the vendee's failure to pay cover two or more
installments. In this case, he shall have no further action against the
purchaser to recover any unpaid balance of the price. Any agreement to the
contrary shall be void.
This provision means that should the vendee or the purchaser of a personal
property default in the payment of two or more of the agreed installments,
the vendor or the seller has the option to avail any of these 3 remedies—
either to exact fulfillment by the purchaser of the obligation, or to cancel the
sale, or to foreclose the mortgage on the purchased personal property, if
one was constituted. These remedies have been recognized as an
alternative, not cumulative, that the exercise of one should bar the exercise
of the others.

In the present case, it is not disputed that IFC had taken possession of the
car purchased by the Nonatos after the spouses defaulted in their payments.
The defense of IFC that it the repossession of the vehicle was only for the
purpose of appraising its value and for storage and safekeeping pending full
payment of the spouses is untenable. The receipt issued by IFC to the
spouses when it took possession of the vehicle that the vehicle could be
redeemed within 15 days. This could only mean that should the spouses fail
to redeem the car within the period provided, IFC would retain permanent
possession of the vehicle. IFC even notified the spouses Nonato that the
value of the car was not sufficient to cover the balance of the purchase price
and there was no attempt at all on the part of the company to return the
car.

The acts performed by IFC are consistent with the conclusion that it had
opted to cancel the sale of the vehicle. Therefore, it is barred from exacting
payment from the petitioners of the balance of the price of the vehicle which
it had already repossessed (it cannot have its cake and eat it too)
75. Delta Motor Sales vs. Niu Kim Duan, 213 SCRA 259
Forfeiture clauses in installments sales were valid, so long as they are not
unconscionable. The remedied provided for under Art. 1484 were
alternative, not cumulative.

Facts:

Niu Kim and Chan Fue purchased 3 air conditioners from Delta Motors for
about P19k via a Deed of Conditional Sale. Payable: down payment of P774,
the balance in 24 installments. The appellants-buyer executed a PN to
secure the payment of the balance. They were able to pay 7 monthly
installments, after which, they defaulted in paying at least two installments.
For such, Delta repossessed the air conditioners through the writ of replevin.
The contract between the parties was also declared judicially rescinded.
Despite this, however, Delta still sought to recover the unpaid balance from
the sale of about P7k. The appellants refused to pay.

Thus, Delta brought an action for the collection of said balance.

The appellants raised the issue of the validity of the Deed of Conditional sale
for being contrary to law, morals, good custom, public order or public policy.
Specifically pars.5 and 7 thereof which according to them were iniquitous.
Par. 5, in essence provides: in case of buyer’s failure to comply with the
term and conditions of the sale, the sale will automatically become null and
void; installments paid will be forfeited, i.e. treated as rentals; Delta may
repossess the property without liability for trespass. Whereas, par. 7
provides: obligates the buyers to deliver the property to seller in case of
rescission; to bear costs of suit by the seller in case one be brought; pay
penalty of 25% of the balance.

The trial court ruled in favor of Delta Motors, after declaring the Deed of
Conditional Sale between the parties rescinded. The CA elevated the case to
SC. The SC reversed — Niu Kim Duan not liable.

Issues:

1. Whether or not Delta Motors justified in retaining the


down payment and instalments paid by the appellants,
and treating the same as rental for the use of the
property?
2. Whether or not Delta may still demand exact fulfilment
of the appellant’s obligation to pay the balance under
the contract, notwithstanding its repossession of the
air conditioners and cancellation of the sale?
HELD:

1. YES.

A stipulation in a contract that the installments paid shall not be returned to


the vendee is valid insofar as the same may not be unconscionable under
the circumstances is sanctioned by Article 1486 of the New Civil Code.

Under the circumstances, the treatment of the installment payments as


rentals cannot be said to be unconscionable. The appellants paid for 7
monthly installments while they where able to use the air conditioners for 22
months. Thus, in effect, they were able to use the same for 15 months rent-
free to the prejudice of Delta.

2. NO.

The 3 remedies provided for under Art. 1484, i.e. exact fulfillment,
cancellation of the sale, and foreclosure of the chattel mortgage, are
alternative and NOT cumulative. If the creditor chooses one remedy, he
cannot avail himself of the other two.

Delta Motors was able to repossess the air conditioners by virtue of the writ
of replevin pursuant to pars. 5 and 7 of the Deed of Conditional Sale upon
the appellant’s failure to pay at least two months’ installment. It also
obtained judicial rescission of the said contract. Thus, by electing the second
remedy under Art. 1484, Delta is precluded from exercising the other
alternative remedies thereon. It is barred from exacting payment from
appellants. It cannot have its cake and eat it too.

Judgment set aside.


76. ELISCO TOOL MFG CORP vs. CA
GR # 109966, May31, 1999

FACTS:
 Private respondent Rolando Lantan entered into an agreement with his
employer, herein petitioner, leasing unto the former a Colt lancer fro a
period of 5 years.  The contract also provided that at the end of the 5 year
period, Lantan may exercise the option to purchase price of the car and he
should just pay the remaining balance.  Said option is limited to the
employee.  It also provided that upon Lantan’s failure to pay 3 accumulated
monthly rentals, the petitioner will have the option to lease said vehicle to
another. Lantan also has to return the car in case he resigns or is
dismissed.  He was laid off after petitioner ceased operations in 1981.  At
that time, he has paid P61, 070.94. Petitioner then filed a replevin case
against Lantan and his wife, alleging that they have failed to pay the
monthly rentals despite repeated demands.  

Issue:
Whether or not the Lease Agreement with option to buy may be treated as a
sale on installment.

HELD: 
Although the agreement provides for the payment by Lantan of monthly
rentals”, the 5th paragraph thereof gives them the option to purchase the
motor vehicle at the end of the 5th year or upon payment of the
60th monthly rental when “all monthly rentals shall be applied to the
payment of the full purchase price of the car.” Clearly, the transaction is a
lease in name only.  The so-called monthly rentals are in truth monthly
amortizations on the car’s price.  Being a contract of sale on installment,
A.1484 &1485 apply.  As such, the case should be considered as one for
specific performance pursuant to A.1484 (1). The prayer for a writ of
replevin is only for the purpose of ensuring specific performance by private
respondents.  However, the private respondents could no longer be held
liable for the payment of interest on unpaid monthly rentals since it was
entered into in pursuance of a car plan adopted by petitioner for the benefit
of its deserving employees.  Further, private respondents’ default in
payment was due to the cessation of operations of petitioner’s sister
company. Elizalde Steel Company. That petitioner accepted payments from
Lantan more than 2 years after the latter’s employment have been
terminated constitutes a waiver of petitioner’s right to collect interest upon
delayed payment. What private respondents paid should be considered the
payment in full.
77. PCI Leasing and Finance, Inc. v. Giraffe-X Creative Imaging, Inc.
G.R. No. 142618, 12 July 2007, 527 SCRA 405

FACTS:

Petitioner PCI LEASING and respondent GIRAFFE entered into a Lease


Agreement,1 whereby the former leased out to the latter one (1) set of
Silicon High Impact Graphics and accessories worth ₱3,900,00.00 and one
(1) unit of Oxberry Cinescan 6400-10 worth ₱6,500,000.00. In connection
with this agreement, the parties subsequently signed two (2) separate
documents, each denominated as Lease Schedule.

Likewise forming parts of the basic lease agreement were two (2)
separate documents denominated Disclosure Statements of Loan/Credit
Transaction (Single Payment or Installment Plan) that GIRAFFE also
executed for each of the leased equipment. These disclosure statements
inter alia described GIRAFFE, vis-à-vis the two aforementioned equipment,
as the “borrower” who acknowledged the “net proceeds of the loan,” the
“net amount to be financed,” the “financial charges,” the “total installment
payments” that it must pay monthly for thirty-six (36) months, exclusive of
the 36% per annum “late payment charges.” Thus, for the Silicon High
Impact Graphics, GIRAFFE agreed to pay ₱116,878.21 monthly, and for
Oxberry Cinescan, ₱181.362.00 monthly.

By the terms, too, of the Lease Agreement, GIRAFFE undertook to


remit the amount of ₱3,120,000.00 by way of “guaranty deposit,” a sort of
performance and compliance bond for the two equipment. Furthermore, the
same agreement embodied a standard acceleration clause, operative in the
event GIRAFFE fails to pay any rental and/or other accounts due.

A year into the life of the Lease Agreement, GIRAFFE defaulted in its
monthly rental-payment obligations. And following a three-month default,
PCI LEASING, through one Atty. Florecita R. Gonzales, addressed a formal
pay-or-surrender-equipment type of demand letter4 dated February 24,
1998 to GIRAFFE.

The demand went unheeded. PCI LEASING instituted a case praying


for the issuance of a writ of replevin for the recovery of the leased property.
GIRAFFE filed a Motion to Dismiss arguing that the seizure of the two (2)
leased equipment stripped PCI LEASING of its cause of action. Expounding
on the point, GIRAFFE argues that, pursuant to Article 1484 of the Civil Code
on installment sales of personal property, PCI LEASING is barred from
further pursuing any claim arising from the lease agreement and the
companion contract documents, adding that the agreement between the
parties is in reality a lease of movables with option to buy. The trial court
granted GIRAFFE’s motion to dismiss.

ISSUE:

Whether or not the underlying Lease Agreement, Lease Schedules and


the Disclosure Statements that embody the financial leasing arrangement
between the parties are covered by and subject to the consequences of
Articles 1484 and 1485 of the New Civil Code.

RULING:

The recourse is without merit.


R.A. No. 5980, in its original shape and as amended, partakes of a
supervisory or regulatory legislation, merely providing a regulatory
framework for the organization, registration, and regulation of the
operations of financing companies. As couched, it does not specifically define
the rights and obligations of parties to a financial leasing arrangement. In
fact, it does not go beyond defining commercial or transactional financial
leasing and other financial leasing concepts.

Petitioner foists the argument that the Recto Law, i.e., the Civil Code
provisions on installment sales of movable property, does not apply to a
financial leasing agreement because such agreement, by definition, does not
confer on the lessee the option to buy the property subject of the financial
lease. To the petitioner, the absence of an option-to-buy stipulation in a
financial leasing agreement, as understood under R.A. No. 8556, prevents
the application thereto of Articles 1484 and 1485 of the Civil Code.
We are not persuaded.

A financing arrangement has a purpose which is at once practical and


salutary. R.A. No. 8556 was, in fact, precisely enacted to regulate financing
companies’ operations with the end in view of strengthening their critical role
in providing credit and services to small and medium enterprises and to
curtail acts and practices prejudicial to the public interest, in general, and to
their clienteles, in particular. As a regulated activity, financing arrangements
are not meant to quench only the thirst for profit. They serve a higher
purpose, and R.A. No. 8556 has made that abundantly clear.

We stress, however, that there is nothing in R.A. No. 8556 which


defines the rights and obligations, as between each other, of the financial
lessor and the lessee. In determining the respective responsibilities of the
parties to the agreement, courts, therefore, must train a keen eye on the
attendant FACTS and circumstances of the case in order to ascertain the
intention of the parties, in relation to the law and the written agreement.
Likewise, the public interest and policy involved should be considered. It
may not be amiss to state that, normally, financing contracts come in a
standard prepared form, unilaterally thought up and written by the financing
companies requiring only the personal circumstances and signature of the
borrower or lessee; the rates and other important covenants in these
agreements are still largely imposed unilaterally by the financing companies.
In other words, these agreements are usually one-sided in favor of such
companies. A perusal of the lease agreement in question exposes the many
remedies available to the petitioner, while there are only the standard
contractual prohibitions against the respondent. This is characteristic of
standard printed form contracts.

On the whole, then, we rule, as did the trial court, that the PCI
LEASING- GIRAFFE lease agreement is in reality a lease with an option to
purchase the equipment. This has been made manifest by the actions of the
petitioner itself, foremost of which is the declarations made in its demand
letter to the respondent. There could be no other explanation than that if the
respondent paid the balance, then it could keep the equipment for its own; if
not, then it should return them. This is clearly an option to purchase given to
the respondent. Being so, Article 1485 of the Civil Code should apply.

The present case reflects a situation where the financing company can
withhold and conceal – up to the last moment – its intention to sell the
property subject of the finance lease, in order that the provisions of the
Recto Law may be circumvented. It may be, as petitioner pointed out, that
the basic “lease agreement” does not contain a “purchase option” clause.
The absence, however, does not necessarily argue against the idea that
what the parties are into is not a straight lease, but a lease with option to
purchase. This Court has, to be sure, long been aware of the practice of
vendors of personal property of denominating a contract of sale on
installment as one of lease to prevent the ownership of the object of the sale
from passing to the vendee until and unless the price is fully paid.
78. CASA FILIPINA REALTY CORP. vs. OFFICE OF THE PRESIDENT
and SPOUSES DENNIS and REBECCASEVILLA, 241 SCRA 165,
February 7, 1995

FACTS:

In May or June 1984, spouses Sevilla agreed to buy from Casa Filipina a
parcel of land (264 SQM) in a subdivision in Parañaque. The parties agreed
that the price of P150,480 would be paid on installment basis with P36,115
as DP and P3,560 as monthly installment for 5 years at 28% amortization
interest per year. The agreement was placed in a contract to sell executed
on Nov. 15, 1984.
Spouses Sevilla failed to pay the amortizations on time. The last installments
they paid on September 1985 were for April-July 1985.
Later, Dennis Sevilla requested a refund of all installments made upon
finding out the absence of any improvement in the subdivision and that the
mother title of the subdivision was under lis pendens and mortgage to
ComSavings Bank.
On Nov. 19, 1985, spouses Sevilla sued Casa Filipina with the Human
Settlements Regulatory Commission (HSRC). They prayed for a refund of
P70,431 plus legal interest thereon plus damages. The HSRC found Casa
Filipina to be without license to sell the subdivision involved. The HSRC held
that even if Casa Filipina had a license to sell, it would still be liable for
violation of Sec. 20 of PD 957 as it had failed to develop the subdivision.
Hence, HSRC ordered Casa Filipina to refund the P70,431 with interest and
damages.
The HSRC was affirmed by the Housing and Land Use Regulatory Board
(HLURB) only that it reduced the 28% interest to 6%. The Office of the
President affirmed the HLURB.
As such, Casa Filipina filed this petition. Casa Filipina argues that the
petition should not have been dismissed as it involves the interpretation of
provisions of law as the court has to determine whether it is Sec. 23 or 24 of
PD 957 which should be applied in this case. Casa Filipina argues that since
the spouses Sevilla desisted from paying the installments, they should have
notified Casa Filipina of such desistance in accordance with Sec. 23. More so,
since Spouses Sevilla’s desistance from paying was due to litispendentia and
the mortgage of the mother title of the subdivision, Sec. 24 should have also
been applied.
Casa Filipina underscores the holding of the Office of the President (OP) that
Sec. 23 “does not require that a notice be given first before a demand for
refund can be made” as the notice and demand “can be made in the same
letter of communication” which was exactly what spouses Sevilla did.
While Casa Filipina agrees that the notice and demand for reimbursement
may be made in one communication, it avers that Sec. 23 clearly provides
that there can be NO forfeiture of payments made by a buyer ONLY IF such
buyer has first given notice to the developer that he will not pay the
installments anymore on the ground that the subdivision has not been
developed.
ISSUE:
Whether or not the instalments already paid by spouses Sevilla should be
refunded in pursuance to Sec. 23 of PD 957.

Ruling:
Yes.
Casa Filipina’s contention is premised such that spouses Sevilla’s desistance
from further payment of instalment was based merely on the notice of lis
pendens and the mortgage on the mother title or on “reasons other than
non-development.” This is belied by the letter sent to Casa Filipina
itself.
Under the letter, spouses Sevilla’s refusal to continue paying is based on 2
principal grounds: (1) non-development of the subdivision; and (2) the
encumbrance of the property. As such, the case falls squarely within
the purview of both Secs. 23 and 24 of PD 957.

Considering the peculiar circumstances of this case, the SC agrees with the
OSG that the requirements of Sec. 23 have been complied with by spouses
Sevilla such that Sec. 23 does not require that a notice be given first before
demand of refund can be made. The notice and demand can be made in the
same letter, and this is what spouses Sevilla did.

Casa Filipina would insist that when spouses Sevilla demand a refund,
they were already in default and that their demand had the sound of
belated attempt to cover up their default. This contention is
untenable.
The general rule under Article 1169 is that an obligor incurs in delay or
default only after a demand, judicial or extrajudicial, has been made
from him for the fulfillment of the obligation. Here, there was no such
demand by Casa Filipina. The letters it sent to spouses Sevilla were
the usual reminders that are ordinarily sent by creditors to late-paying
debtors. They are not the demand contemplated by law.
Being in accord with the spirit behind PD 957, OP’s conclusions are hereby
affirmed.

Discussion of PD 957
The decree was issued in the wake of numerous reports that many real
estate subdivision owners, developers, operators and/or sellers "have
reneged on their representations and obligations to provide and maintain
properly subdivision roads, drainage, sewerage, water systems, lighting
systems and other basic requirements" for the health and safety of home
and lot buyer's. It was designed to stem the tide of "fraudulent
manipulations perpetrated by unscrupulous subdivision and condominium
sellers and operators, such as failure to deliver titles to buyers or titles free
from liens and encumbrances."
Should the notice requirement provided for in Sec. 23 be construed as
required to be given before a buyer desists from further paying
amortizations as in this case, the intent of the law to protect subdivision lot
buyers, such as private respondents, will tend to be defeated.
Application in this case

Here, Casa Filipina did not only fail to develop the subdivision it was selling
but had also encumbered it prior to selling the same. The inscription of acts
and transactions relating to the ownership and other rights over
immovables, even as it serves as a constructive notice to the world, is
intended to protect the person in whose favor the entry is made and the
public in general against any possible undue prejudice due to ignorance on
the status of the property.
The rule on constructive notice is not designed to allow a person to escape
from a lawfully incurred liability. Thus, a vendor of a real estate whereon an
adverse claim is validly annotated cannot invoke such registration to avoid
his own obligation to make a full disclosure to the vendee of adverse claims
affecting the property. The registration protects the adverse claimant
because of the rule on constructive notice but not the person making the
conveyance. It behoves such real estate developer and dealers to make
proper arrangements with the financial institutions to allow the release of
titles to buyers upon their full payment of the purchase price.
More so, the HLURB found that Casa Filipina had not secured a license prior
to the sale of the lot which is a requirement under Sec. 5 of PD 957.

Application on the issue of interest payment


On the issue of delinquency interest which Sec. 23 of P.D. No. 957 explicitly
excludes from the amount to be reimbursed to lot buyers, the Solicitor
General avers that since the matter has been belatedly raised, the same
should be deemed waived. However, since rules of procedure are mere tools
designed to facilitate the attainment of justice, their strict and rigid
application which would result in technicalities that tend to frustrate rather
than promote substantial justice, must always be avoided. Technicality
should not be allowed to stand in the way of equitably and completely
resolving the rights and obligations of the parties.
79. THE MANILA BANKING CORPORATION v.
SPOUSES ALFREDO AND CELESTINA RABINA, et al.

574 SCRA 16 (2008)

The Housing and Land Use Regulatory Board (HLURB) has the power to
regulate real estate trade and the same include complaints for annulment f
mortgage.

Facts:

Marenir Development Corporation (MDC), owner/developer of a subdivision


project in Quezon City obtained a loan from the Manila Banking Corporation
(MBC). To secure the payment of such loan, it forged a real estate mortgage
covering real estate properties including the lot which was subject of a
Contract to Sell to Amante Sibuyan (Sibuyan). Sibuyan transferred the lot
via ―Assignment and Transfer of Rights‖ to Celestina Rabina (Rabina), with
the conformity of MDC. The said document mentioned the Contract to Sell
which MDC had executed in favor of Sibuyan.

After Rabina had fully paid the amortization payments for the lot, she asked
MDC for the transfer to her of its title. MDC, however, failed, prompting
Rabina to institute a complaint for non-delivery of titles, annulment of
mortgage and incomplete development of the subdivision project
Reymarville Subdivision, against MDC before the Office of Appeals,
Adjudication and Legal Affairs (OAALA) of the Housing and Land Use
Regulatory Board (HLURB). MBC contended that the HLURB has no
jurisdiction over it by virtue of Section 29 of Republic Act 265, as amended
by Executive Order No. 289.

Housing and Land Use Arbiter Cesar Manuel found in favor of the Rabina and


ordered for the payment of moral damages. Upon MBC‘s appeal, the HLURB
Board of Commissioners affirmed the Arbiter‘s decision. MBC then elevated
the case to the Office of the President (OP) but the same have been
dismissed.

On elevation to the Court of Appeals, the appeal was dismissed and the CA
affirmed the Orders of the OP. Hence, this petition.

ISSUES:

Whether or not the HLURB has jurisdiction over the case at bar under
Presidential Decree 957

HELD:

The act of MDC in mortgaging the lot, without the knowledge and consent of
lot Spouses Rabina and without the approval of the HLURB, as required by
P.D. 957, is not only an unsound real estate business practice but also highly
prejudicial to them.

The jurisdiction of the HLURB to regulate the real estate trade is broad
enough to include jurisdiction over complaints for annulment of mortgage.
To disassociate the issue of nullity of mortgage and lodge it separately with
the liquidation court would only cause inconvenience to the parties and
would not serve the ends of speedy and inexpensive administration of justice
as mandated by the laws vesting quasi-judicial powers in the agency.
80. EMILIANO RILLOvs.COURT OF APPEALS and CORB REALTY
INVESTMENT, CORP.,G.R. No. 125347, June 19, 1997

Facts:

 Rillo signed a Contract to Sell of Condominium Unit with respondent for a


61.5 sqm unit condominium in Mandaluyong for 150K, 75K upon the
execution of the contract and the remainder in 12 equal monthly installments
of 7,092 beginning July 18, 1985. All outstanding balanced would bear 24%
interest/annum and the installment in arrears would be subject to liquidated
penalty of 1.5% for every month of default from due date. Furthermore if
there would be 3 or more installments defaulted, forfeiture proceedings
would be governed by existing laws, particularly the condominium act.
 Rillo failed to pay the initial installment on July and the one for August. On
September he paid his first amortization on September (7092), 2 nd on
October (7092) and third on February of the next year(1986) and only 5000.
On July 20, 1987, 17 months after his last payment CORB Realty informed
him by letter of cancellation of the contract due to his failure to settle his
accounts on time, with the willingness to refund the payments.
 Such cancellation did not occur after receiving 60K from Rillo. But petitioner
defaulted again resulting in CORB again warning him of the rescission and
instructions to accept from their office the payments which was 102,459 after
deducting rentals. Such also did not push through instead they entered into a
compromise.
o Restructure outstanding balance to 50K
o To be paid 2k/mos at 18% interest
o To pay taxes due
o Installments would start on apr 15, 1989
 Rillo paid 2k on April and May but failed to pay the rest.
 On April 3 CORB sent a letter fixing all the arrears at 155,129 and failure to
pay such amount lead to it filing a cmpalint for cancellation of the contract
with RTC Pasig
 Rillo answered stating that as a personal title had still to be delivered to him
even after he had paid 159K in total and that CORB could not claim against
the old agreement which has been novated to pay 50K.
 They stipulated on pre-trial that the outstanding amount is 50K and only 4k
had been paid therefore.
 RTC: found that due to the paid amounts CORB could not rescind as such
had been substantially complied with.
 CA: Reversed. Rescission does not apply as the contract is not an absolute
conveyance of real property and that the Condominium Act has nothing to
say on forfeiture in sales by installment. That what is applicable is PD 957
which states that RA 6552 should be applied which means the contract to sell
is thus declared cancelled and rendered ineffective.

Issues:

 WoN CA erred in not applying rescission under Art 1191 and 1592? No
 WoN the CA erred in applying the Maceda Law (6552)? No

Ruling:

 The contract between the parties is not an absolute conveyance of real


property but a contract to sell. In a contract to sell real property on
installments, the full payment of the purchase price is a positive suspensive
condition, the failure of which is not considered a breach, casual or serious,
but simply an event which prevented the obligation of the vendor to convey
title from acquiring any obligatory force.”
 The transfer of ownership and title would occur after full payment of the
purchase price.
 R.A. No. 6552 recognizes in conditional sales of all kinds of real estate
(industrial, commercial, residential) the right of the seller to cancel the
contract upon non-payment of an installment by the buyer, which is simply
an event that prevents the obligation of the vendor to convey title
 Rights of a buyer in the case of his fault:
o (1) Where he has paid at least two years of installments,
 “(a) To pay, without additional interest, the unpaid installments
due within the total grace period earned by him, which is hereby
fixed at the rate of one month grace period for every one year
of installment payments made: Provided, That this right shall be
exercised by the buyer only once in every five years of the life
of the contract and its extensions, if any.
 (b) If the contract is cancelled, the seller shall refund to the
buyer the cash surrender value of the payments on the property
equivalent to fifty per cent of the total payments made and,
after five years of installments, an additional five per cent every
year but not to exceed ninety per cent of the total payments
made: Provided, That the actual cancellation of the contract
shall take place after cancellation or the demand for rescission
of the contract by a notarial act and upon full payment of the
cash surrender value to the buyer. Down payments, deposits or
options on the contract shall be included in the computation of
the total number of installments made.”
o (2) Where he has paid less than two years in installments,
 “Sec. 4. x xx the seller shall give the buyer a grace period of not
less than sixty days from the date the installment became due.
If the buyer fails to pay the installments due at the expiration of
the grace period, the seller may cancel the contract after thirty
days from receipt by the buyer of the notice of cancellation or
the demand for rescission of the contract by a notarial act.
 Having paid for less than 2 years buyer had a grace period to pay of not less
than 60 days and Seller had a right to cancel after 30 days from the receipt
by rillo of the notice of cancellation.
 No Novation; Parties to a contract must expressly agree that they are
abrogating their old contract in favor of a new one.18 In the absence of an
express agreement, novation takes place only when the old and the new
obligations are incompatible on every point.
 In the case at bar, the parties executed their May 12, 1989 “compromise
agreement” precisely to give life to their “Contract to Sell.” It merely clarified
the total sum owed by petitioner RILLO to private respondent CORB REALTY
with the view that the former would find it easier to comply with his
obligations under the Contract to Sell. In fine, the “compromise agreement”
can stand together with the Contract to Sell.
 Modification: no refund
o Under Republic Act No. 6552, the right of the buyer to a refund
accrues only when he has paid at least two (2) years of installments.
In the case at bar, RILLO has paid less than two (2) years in
installments, hence, he is not entitled to a refund.
 DENIED, CA Affirmed

81. ACTIVE REALTY & DEVELOPMENT CORPORATION vs. NECITA G.


DAROYA, represented by Attorney-In-Fact Shirley Daroya-Quinones,
G.R. No. 141205, May 9, 2002
Facts:

 Active Realty entered into a Contract to Sell with Daroya, whereby the latter
agreed to buy a lot for P224,025.00 in petitioner’s subdivision and that the
respondent shall pay a down payment upon execution of the contract and the
balance in sixty (60) monthly installments which totalled to a figure higher
than that stated as the contract price.
 However, respondent was in default representing three (3) monthly
amortizations. Petitioner moved for the cancellation of their contract to sell.
Petitioner refused the respondent’s offer to pay the remaining amount as it
has sold the lot to another buyer.
 Respondent filed a complaint against petitioner before the Housing and Land
Use Regulatory Board (HLURB) for the execution of a final Deed of Absolute
Sale in respondent’s favor after she pays any balance that may still be due
from her.
 HLURB Arbiter found for the respondent and ruled that the cancellation of the
contract to sell was void as petitioner failed to pay the cash surrender value
to respondent as mandated by law. On appeal, the HLURB Board of
Commissioners set aside the Arbiter’s Decision which did not apply the
remedies provided under the Maceda Law and found both parties were at
fault, i.e., respondent incurred in delay in her installment payments and
respondent failed to send a notarized notice of cancellation. The Board
ordered petitioner to refund to the respondent one half of the total amount
she has paid.
 Respondent appealed to the Office of the President which modified the
Decision of the HLURB. The COS between the parties subsisted and
concluded that respondent was entitled to the lot after payment of her
outstanding balance due to petitioner’s failure to comply with the legal
requisites for a valid cancellation of the contract. However, as the lot was
already sold to another person and that the actual value of the lot as of the
date of the contract was P1,700.00 per square meter, petitioner was ordered
to refund to the respondent the amount of P875,000.00, the true and actual
value of the lot as of the date of the contract, with interest at 12% per
annum computed from August 26, 1991 (date of filing of the complaint) until
fully paid, or to deliver a substitute lot at the choice of respondent.
 CA ruled against petitioner only on the basis of form and substance.

Issue:

 WON the petitioner can be compelled to refund to the respondent the value
of the lot or to deliver a substitute lot at respondent’s option.

Held:

 SC found for the respondent and ruled in the affirmative.


 The contract to sell in the case at bar is governed by the Maceda Law. More
specifically, Section 3 of R.A. No. 6552 provided for the rights of the buyer in
case of default in the payment of succeeding installments, where he has
already paid at least two (2) years of installments.
 The records clearly showed that the petitioner failed to comply with the
mandatory twin requirements for a valid and effective cancellation under the
law---that petitioner
 Failed to send a notarized notice of cancellation and failed to refund the cash
surrender value since it was only during the preliminary hearing of the case
before the HLURB arbiter when petitioner offered to pay the cash surrender
value
 Moreover, there was no formal notice of cancellation or court action to
rescind the contract and therefore SC found it illegal and iniquitous that
petitioner, without complying with the mandatory legal requirements for
cancelling the contract, forfeited both respondent’s land and hard-earned
money after she has paid for, not just the contract price, but more than the
consideration stated in the contract to sell.
 Thus, for failure to cancel the contract in accordance with the procedure
provided by law, SC hold that the contract to sell between the parties
remains valid and subsisting. Following Section 3(a) of R.A. No. 6552,
respondent has the right to offer to pay for the balance of the purchase price,
without interest, which she did in this case.
 Ordinarily, petitioner would have had no other recourse but to accept
payment. However, respondent can no longer exercise this right as the
subject lot was already sold by the petitioner to another buyer which lot was
valued at P1,700.00 per square meter. As respondent lost her chance to pay
for the balance of the P875,000.00 lot, it is only just and equitable that the
petitioner be ordered to refund to respondent the actual value of the lot
resold, i.e., P875,000.00, with 12% interest per annum computed from
August 26, 1991 until fully paid or to deliver a substitute lot at the option of
the respondent.

82. ISAIAS F. FABRIGAS and MARCELINA R. FABRIGAS vs. SAN


FRANCISCO DEL MONTE, INC., G.R. No. 152346 November 25, 2005

Facts:
 Spouses Fabrigas and San Francisco entered into an agreement denominated
as a Contract to Sell, whereby San Francisco agreed to the spouses a parcel
of residential land. The agreement included:
o P30,000 downpayment
o The balance to be paid in monthly installments for 10 years
o Automatic cancellation clause in case of default
 After paying the downpayment, Spouses Fabrigas took possession of the
property but failed to make any installment payments on the balance
 Del Monte sent demand letters on 4 occasions. Thereafter, Del Monte
considered the Contract to Sell cancelled 15 days after it made a final
demand letter. However, Del Monte did not furnish the spouses any notice
regarding its cancellation

 10 months after, Marcelina Fabrigas remitted P13,000, followed by P12,000


in another 3 months.

 The parties entered into another agreement covering the same property but
under restructured terms of payment
 The spouses made irregular payments – prompting Del Monte to send a
demand letter once again, informing the spouses of their overdue account
equivalent to 9 installments
 For failure to pay, Del Monte notified the spouses that their contract has been
cancelled and demanded that they vacate the property

Issue:

 W/N a party to a contract to sell may unilaterally rescind the contract upon
default if it is so stipulated

Ruling:

 NO. SECTION 4 of R.A. 6552 or the Realty Installment Buyer Act provides:

 In case where less than two years of installments were paid, the seller
shall give the buyer a grace period of not less than sixty days from the
date the installment became due.
 If the buyer fails to pay the installments due at the expiration of the
grace period, the seller may cancel the contract after thirty days from
receipt by the buyer of the notice of cancellation or the demand for
rescission of the contract by a notarial act.
 While the Court concedes that Del Monte had allowed petitioners a grace
period longer than the minimum 60-day requirement under Section 4, it did
not comply, however, with the requirement of notice of cancellation or a
demand for rescission. Instead, Del Monte applied the automatic rescission
clause of the contract. Contrary, however, to Del Monte's position which the
appellate court sustained, the automatic cancellation clause is void under
R.A. 6552
 The Court, however, found that although the original contract to sell was not
validly rescinded, it has already been extinguished by novation since the
parties executed a second contract to sell under restructured payment terms.
Thus, the spouses remain liable for breach under the second contract.
 WHEREFORE, the instant Petition for Review is DENIED and the September
28, 2001 Decision of the Court of Appeals in CA-G.R. CV No. 45203 is
AFFIRMED. Costs against petitioners.
 SO ORDERED

83. JESTRA DEVELOPMENT AND MANAGEMENT CORPORATION v.


DANIEL PONCE PACIFICO
513 SCRA 403 (2007)

Cancellation of the contract, under the law, requires that the seller should


extend the buyer a grace period of at least 60 days from the due date of the
installment, and at the end of the grace period, the seller shall furnish the
buyer with a notice of cancellation or demand for rescission.

Facts:

Daniel Ponce Pacifico (Pacifico) signed a Reservation Application with Fil-


Estate Marketing Association for the purchase of a house and lot located at
Paranaque, Metro Manila and paid the reservation fee of 20,000.00. Under
the Reservation Application, upon fulfillment of the 30% down payment by
Pacifico, he will sign a contract to sell with the owner and developer of the
property which is the JESTRA Development and Management Corporation
(Jestra).

Pacifico run out of funds to pay for the property, and he requested to Jestra
to suspend payment. Jestra denied his request. Pacifico filed a complaint
before the Housing and Land Use Regulatory Board (HLURB) against Jestra
claiming that despite his full payment of the down payment, Jestra failed to
deliver to him the property within 90 days as provided in the contract to sell
dated March 6, 1997 and Jestra instead sold the property to another buyer
in October 1998.

ISSUE:

Whether or not the act of Jestra in canceling the contract to sell agreement
with Pacifico is valid

HELD:

R.A. 6552 was enacted to protect buyers of real estate on installment


against onerous and oppressive conditions. In Fabrigas v. San Francisco del
Monte,Inc., the court described the cancellation of the contract under
Section 4 of R.A. 6552 as a two-step process. First, the seller should extend
the buyer a grace period of at least 60 days from the due date of the
installment. Second, at the end of the grace period, the seller shall furnish
the buyer with a notice of cancellation or demand for rescission through a
notarial act, effective 30 days from the buyer‘s receipt thereof.

Pacifico admits that the first installment on the 70% balance of


the purchase price was due on January 5, 1998. He issued checks for it but
was dishonored due to insufficiency of funds. Pacifico was notified of the
dishonor of the checks but he took no action, hence, 60 days grace period
lapsed. Pacifico made no further payments thereafter. Instead, he requested
for suspension of payment.

Also, Pacifico admits that Jestra was justified in canceling the contract to sell
via the notarial Notice of Cancellation which he received on May 13, 1998
which took effect on June 12, 1998. Thus, the cancellation of the contract to
sell of Jestra is valid.

84. Villarica v. Court of Appeals


G.R. No. L-19196, 29 November 1968, 26 SCRA 189
FACTS:

On May 19, 1951, the spouses Angel Villarica and Nieves Palma Gil de Villarica sold
to the spouses Gaudencio Consunji and Juliana Monteverde a lot containing an area
of 1,174 sq. meters, situated in the poblacion of the City of Davao, for the price of
P35,000. The instrument of absolute sale dated May 19, 1951 in the form of a deed
poll, drafted by Counselor Juan B. Espolong who had been appointed by the
Villaricas as their agent to sell the lot, was acknowledged on May 25, 1951, before
the same Juan B. Espolong who was also a Notary Public. The public instrument of
absolute sale and the vendors’ TCT No. 2786 were delivered to the vendees. On the
same day, May 25, 1951, the spouses Consunji executed another public instrument,
“, whereby they granted the spouses Villarica an option to buy the same property
within the period of one year for the price of P37,750. In July, same year, the
spouses Consunji registered the absolute deed of sale, in the names of the spouses
Villarica was cancelled and a new TCT No. 3147 was issued in the names of the
spouses Consunji.

In February, 1953, the spouses Consunji sold the lot to Jovito S. Francisco for the
price of P47,000 by means of a public instrument of sale “. This public instrument of
sale was registered in view of which TCT No. 3147 in the names of the spouse
Consunji was cancelled and a new TCT in the name of Jovito S. Francisco was
issued.

On April 14, 1953, the spouses Villarica brought an action in the Court of First
Instance of Davao against the spouses Consunji and Jovito S. Francisco for the
reformation of the instrument of absolute sale, into an equitable mortgage as a
security for a usurious loan of P28,000 alleging that such was the real intention of
the parties. Defendants answered that the deed of absolute sale expressed the real
intention of the parties and they also alleged a counterclaim for sums of money
borrowed by the plaintiffs from the Consunjis which were then due and
demandable. After trial, the Court of First Instance of Davao rendered its decision
holding that the instrument of absolute sale, was really intended as an equitable
mortgage.

ISSUE:

Whether or not the instrument to be reformed was an equitable mortgage?

RULING:

No. The transaction was not an equitable mortgage.

The vendors did not remain in possession of the land sold as lessees or otherwise.
On their request in order to help them in the expenses of their children in Manila,
the vendors were merely allowed by the vendees to collect the monthly rents of
P300 for five months up to October, 1951, on the understanding that the amounts
so collected would be charged against them. But thereafter the vendees were the
ones who collected the monthly rents from the tenants. It follows that the vendors
did not remain in possession of the land as lessees or otherwise.

(3) In Exh. “D” the Consunjis as new owners of the lot granted the Villaricas an
option to buy the property within the period of one year from May 25, 1951 for the
price of P37,750. Said option to buy is different and distinct from the right of
repurchase which must be reserved by the vendor, by stipulation to that effect, in
the contract of sale. This is clear from Article 1601 of the Civil Code, which
provides:

Conventional redemption shall take place when the vendor reserves the right to
repurchase the thing sold, with the obligation to comply with the provisions of
article 1616 and other stipulation which may have been agreed upon.

The right of repurchase is not a right granted the vendor by the vendee in a
subsequent instrument, but is a right reserved by the vendor in the same
instrument of sale as one of the stipulations of the contract. Once the instrument of
absolute sale is executed, the vendor can no longer reserve the right to repurchase,
and any right thereafter granted the vendor by the vendee in a separate instrument
cannot be a right of repurchase but some other right like the option to buy in the
instant case. Hence, Exhibits “B” and “D” cannot be considered as evidencing a
contract of sale with pacto de retro. Since Exh. “D” did not evidence a right to
repurchase but an option to buy, the extension of the period of one year for the
exercise of the option by one month does not fall under No. 3, of Article 1602 of the
Civil Code, which provides that: When upon or after the expiration of the right to
repurchase another instrument extending the period of redemption or granting a
new period is executed.
85. VICENCIO T. TORRES and SOCORRO S. TORRES, petitioners,
vs.
COURT OF APPEALS, CEFERINO ILLUSCUPIDES, ARACELI
ILLUSCUPIDES and EMILIO OLORES, respondents., G.R. No. 93390, 09
December 1992
86. Arrofo vs. Quiño GR No. 145794, 26 January 2005

FACTS:
Pedro Quiño was the owner of the subject land situated in Mandaue City,
Cebu. In April 1990, he executed a Deed of Sale over the land, but excluding
the house on the lot, in favor of Renato Mencias. In March 1991, Quiño
executed another Deed of Absolute Sale over the same property, this time
including the house, in favor of Mencias. In March 1993, Mencias executed a
Deed of Absolute Sale in favor of Lourdes Arroyo
In July 1994, respondent Quiño filed an action for Reconveyance of Property
with Annulment of Deeds of Sale against Spouses Mencias (Renato and
Myrna) and Arrofo, claiming that his transaction with Mencias was a
mortgage and not an absolute sale. Quiño alleged that he borrowed Php
15,000 from Mencias and the land and the house served as security for the
loan. He further alleged that the Deed of Absolute Sale would only be
registered with the Register of Deeds should he fail to pay his Php 15,000
loan within 5 years from April 1990.
The trial court the declared the signed Deed of Absolute Sale as valid.
CA reversed the trial court's decision. It found Arrofo not a buyer in good
faith.
ISSUES:
1. Whether or not the contract is one of sale or one of equitable
mortgage

2. Whether or not Arrofo is a buyer in good faith

RULING:
1. The contract is one of equitable mortgage.

The decisive factor in evaluating whether a deed purporting to be an


absolute sale on its face is in reality a loan secured by a mortgage is the
intention of the parties as shown by all the surrounding circumstances.
Under Art. 1602, in relation to Art. 1604 of the CC, a contract purporting to
be an absolute sale is deemed an equitable mortgage in any of the following
cases:
1. When the price of a sale with right to repurchase is unusually
inadequate

2. When the vendor remains in possession as lessee or otherwise

3. When upon or after the expiration of the right to repurchase, another


instrument extending the period of redemption or granting a new
period is executed

4. When the purchaser retains for himself a part of the purchase price

5. When the vendor binds himself to pay the taxes on the thing sold

6. In any other case where it may be fairly inferred that the real
intention of the parties shall secure the payment of a debt or the
performance of any other obligation.
In the present case some of the above-mentioned circumstances are
satisfied as follows:
1) Spouses Mencias, as buyers, did not take possession of the property
after the execution of the first Deed of Sale. Moralde, a lessee of Quifio
since 1989, testified that he has been paying the rentals to the latter.
He only learned of the sale in April 1993 when Mencias instructed him
to pay the rentals to the latter. Prior to this, Moralde was never
informed that there was already a new owner nor was he asked to
remit the rental payments to the new owner. Since Moralde continued
making his payments to Quiño, Quiño must have retained his
possession of the property.
2) Neither the first nor the second Deed of Absolute Sale reflects the
real intention of the parties. Mabanto, a signatory to the first Deed of
Sale, prepared the original draft of the document. He testified that the
parties were aware that the proper document that should be executed
was that of real estate mortgage and not a deed of sale but Spouses
Mencias were not willing to provide Quiño with the money without the
execution of the deed of sale. The CA also noted that Quiño is an
illiterate. He cannot read and write and can only write his signature.
When Quiño testified before the trial court, his lack of education was
highlighted.
(3) The consideration paid to Quiño is unusually inadequate. The Deeds
of Absolute Sale executed by Mencias to Arrofo state that the
consideration for the sale is Php 50,000. As claimed by Mencias, Quiño
sold the land to him for Php 160,000. If this is the case, the Court
found it unbelievable that Mencias would resell the same property for
less than half the amount he paid for it. That Mencias paid Quino Php
15,000 for the property is a logical explanation. Considering that
Mencias was able to sell the property to Arrofo for Php 50,000, the
amount paid to Quiño is inadequate.
The Court also held that Myrna Mencias' claim that the first Deed of
Absolute Sale is fabricated is false Myrna claimed that she did not know the
person who notarized the said Deed. But the original title of the land showed
the annotation pertaining to the first Deed of Absolute Sale, which means
that the basis of cancellation of the original title is the first Deed. The Court
said that Myrna's testimony is full of contradiction but what is clear is that
the first Deed is not fabricated.
2. NO. Arrolfo failed to act as a prudent buyer.

The law generally does not require a person dealing with registered
and to inquire further than what the Torrens title on its face indicates but
there are exceptions. A purchaser or mortgagee cannot close his eyes to
facts which should put a reasonable man on his guard. Arrofo admitted that
she saw a house constructed on the property but she did not bother to
inquire as to its occupants. Moreover, Myrna was occupying a room in
Arrofo's house as her lessee so the latter knew all along that someone else
must have been occupying the house.
87. Adiarte vs. Tumaneng GR No. L-3031, 15 March 1951

FACTS:
Amanda Madamba Vda de Adiarte owned two parcels of land in Ilocos
Norte. On February 25, 1929, she sold the properties to Sps. Tumaneng for
P1,100 with a right to repurchase within 10 years. The spouses had the
properties registered in February 1944.
On April 6, 1944, the vendor called the vendee offering to purchase
the land. The husband (Clio Gudang) was reluctant to sell at first, but on the
same day, agreed to sell the land back to Adiarte on the condition that he
would have them in his possession for the following two years. To show good
faith, he wrote a document embodying the promise
The husband died on or after April 6, 1946. Vendor offered to purchase
the land but the widow refused to receive P1,100 tendered by the plaintiff.
The widow denied knowledge of the agreement between Agudong and
Adiarte.
ISSUE:
Whether or not the promise to sell signed by the late Cirilo Agudong in his
lifetime is lawful and valid
RULING:
YES.
After the lapse of the ten-year period agreed upon in the deed of sale with
the right to repurchase executed by the appellee, as vendor, the appellant
and her husband, as vendees, became the absolute owners of the two
parcels of land sold to them by the appellee. The promise to sell and convey
the two parcels of land made by Cirilo Agudong, after he and his wife had
become absolute owners thereof, cannot be regarded as a promise to resell
the parcels of land by virtue of the right to repurchase reserved by the
vendor, because that right was lost to the latter after the expiration of ten
years agreed upon without making the repurchase of the two parcels of land.
Hence there is no room for the application of the provisions of article 1508 of
the Civil Code which prohibit an agreement or stipulation for redemption of
the property sold beyond ten years from the date of the contract.
In this case, the original contract of sale with the right to repurchase
reserved by the vendor no longer existed at the time the promise to sell was
made by the purchaser - who had become the absolute owner after the
lapse of the period of time for repurchase to the seller - who had lost all her
right to the property sold, because her failure to repurchase it within the
time agreed upon. The promise is not a promise to sell by virtue of pacto de
retro but an entirely new agreement since the original contact no finger
exists. The term "repurchase" was used for lack of better term known to
Agudong daw.
3. Promise to sell binds the estate of Agudong, because of the absence of
proof that one-half of the purchase price was paraphernal, the presumption
is that it was conjugal.
DISSENT
Pablo, J. It is not a promise of sale. In a promise of sale, there is no prior
sale: a new contract is not preceded by any other. In this case, the same
expresses Agudong allowed to purchase the land, not by simple generosity,
but in the belief that he was obliged to do because the purchase had
obtained in retro pact. Because both parties acted under that belief, sale
should not be allowed in compliance with the provisions of the document,
which is prohibited by Article 1508 of the Civil Code. The right to repurchase
in some sense a suspension the right of ownership of the land or property in
question.
Montemayor, J. The option to repurchase had long expired. It is in violation
of article 1508 of the Civil Code which prohibits any agreement to
repurchase beyond ten years.
88. Heirs of Arches vs. Diaz G.R. No. L-27136, 30 April 1973

FACTS:
The heirs of Jose A. Arches filed a complaint against Maria B. Vda. de Diaz in
the court a quo, alleging inter alia that on January 21, 1954 the defendant
executed in favor of the late Jose A. Arches a deed of sale with pacto de
retro over a parcel of land known as Lot No. 2706 of the Cadastral Survey of
Capiz for and in consideration of P 1250000 that Jose A. Arches during his
lifetime filed a petition on November 20, 1958 in Cadastral Case No. 6. LR.C.
Record No. 338 of the Court of First Instance of Capiz, to consolidate
ownership over the lot. The defendant opposed the petition alleging among
other things that the said deed of sale with pacto de retro did not express
the true intention of the parties, which was merely to constitute a mortgage
on the proper security for a loan.
The Trial Court ruled that when the late Jose A. Arches, father and
predecessor in interest of plaintiffs herein, petitioned this Court on
November 20. 1958, to consolidate in his name ownership and title over Lot
2706, Capiz Cadastre, by virtue of the alleged sale a retro executed by
defendant herein in his favor on January 21, 1954, with reservation of
vendor's right to repurchase in one year, said Jose A. Arches, had two
remedies, inconsistent though they certainly were - (a) to consolidate title
and ownership, and (b) to foreclose in the event the deed of sale a retro be
declared one of equitable mortgage. That said Jose A. Arches elected to
consolidate without alternatively opting to foreclose. When he opted to
consolidate and prosecuted his option to a final determination, he was
thereby barred from pursuing the other alternative and inconsistent remedy
of foreclosure of mortgage or collection of debt
ISSUE:
Whether or not the decision of the cadastral court, holding in effect that the
sale with pacto de retro was an equitable mortgage and consequently
dismissing the petition to consolidate ownership, constitute an adjudication
of the right to foreclose the mortgage or to collect the indebtedness
RULING:
No, in the case of Correa vs. Mateo and Icasiano, wherein an unrecorded
pacto de retro sale was construed as an equitable mortgage, it was ruled
that the plaintiff had the right within sixty days after final judgment for a
failure to pay the amount due and owing him to foreclose his mortgage in a
proper proceeding and sell all or any part of the ten parcels of land to satisfy
his debt" In effect this Court recognize the right of the plaintiff to enforce his
right lien in a separate proceeding notwithstanding the fact that he had
failed to obtain judgment declaring him the sole and absolute owner of the
parcels of land in question.
The law abhors injustice. It would be unjust in this case to allow the
defendant to escape payment of his debt and worse still, to rationalize such
a result by his very claim that he is a debtor and not, as the plaintiff says,
vendor of property in favor of the latter. Strictly speaking, where the petition
of the vendee in a pacto de retro sale is for a judicial order pursuant to
Article 1607 of the Civil Code, so that consolidation of ownership by virtue of
the failure of the vendor to redeem may be recorded in the Registry of
Property the right of action to foreclose the mortgage or to collect the
indebtedness arises from the judgment of the court declaring the contract as
equitable mortgage. Although an alternative prayer to this effect may be
made in the petition, the same cannot but be conditional, is, only in the
event such a declaration made. contrary to the plaintiff's claim and the
principal relief he seeks. His failure to make that alternative prayer, and the
failure of the court to grant it in the judgment dismissing the petition should
not be considered as a bar to collecting the indebtedness in a proper action
for that purpose.
89. Gargollo v. Duero, 1 SCRA 1311 (1961)

Facts:

Gargollo (seller) sold to Duero & Espejo (buyers) with pacto de retro a parcel


of land in Iloilo. This was in May 1953. The Price is P400.
Later, the agreed price increased to P750.

According to the sale with pacto de retro, sellers could redeem the land on or


before 1962. In Sept 1958, seller notified the buyers that she is redeeming
the land the following month. When that month came (October), she gave the
buyers written notice to accept the redemption amount of P750. When buyers
refused the payment, seller deposited the payment to the Clerk of Court
then advised the buyers to withdraw the amount but the buyers say that because of
the promise of seller to definitely sell the land to them for P1k, they
made improvements on the land (banana & fruit trees, rice paddies, corn.)

In Pre-trial, buyers agreed to return the property to seller upon payment of P750


plus P25 as reimbursement for real estate tax plus value of improvements but the
parties failed to agree on who should assess the value of the improvements and
seller says that she did not want to exercise the option to refund the buyer’s
expenses or pay the increase in value of the land as provided in NCC 546(2), but
she is claiming the right given her by NCC Art. 547. And ART. 546 states that
“Necessary expenses shall be responded to every possessor; but only the possessor
in good faith may retain the thing until he has been reimbursed therefor. Useful
expenses shall refunded only to the possessor in good faith with the same right of
retention, the person who has defeated him in a possession having the option of
refunding the amount of the expenses or of paying the increase in value which the
thing may have acquired by reason thereof.” While ART. 547 states that “If the
useful improvements can be removed without damage to the principal thing, the
possessor in good faith may remove them, unless the person who recovers the
possession exercise the option under paragraph 2 of the proceeding article. The
seller now prays that judgment be rendered declaring the land as already redeemed
in view of her deposit of P750 to the Clerk of Court;
to order the buyers to remove all improvements and to vacate; and
to order the buyers to pay the costs of the suit. Then the buyer answers that NCC
Art.1616 states that the seller cannot avail of the right of repurchase
without returning to the buyer the price of the sale and in addition: (1) the
expenses of the contract, and any other legitimate payments made by reason of the
sale; (2) the necessary and useful expenses made on the thing sold.

The lower court decided in favor of seller.

The seller did not want to exercise the option given by NCC 546 (2).

According to NCC 547, as possessors in good faith, buyers are not entitled to


retain the land but only to remove theimprovements if it can be done without
damage.

Issue:

Can buyer be compelled to return the land without seller reimbursing him for the
improvement?

Held:

SC says no. The article applicable is NCC 1616 not NCC 547. NCC 1616 says the
right of repurchase cannot be availed without theseller returning the price of the
land plus necessary and useful expenses.
90. Butte v. Manuel Uy, 4 SCRA 526

FACTS:

Jose V. Ramirez was a co-owner of a house and lot located at Sta Cruz, Manila.
Upon the death of Jose V. Ramirez, all his property including the 1/6 undivided
share was bequeathed to his children and grandchildren and 1/3 of the free portion
to Mrs. Angela M. Butte.

Mrs. Marie Garnier Vda de Ramirez sold the property to Manuel Uy and Sons, Inc.
including the undivided 1/6 share property in Sta Cruz, Manila. On the same day, a
copy of letter regarding the above-mentioned sell was sent to Bank of the Philippine
Islands, as administrator of the property of Jose V. Ramirez.

Mrs. Angela M. Butte filed a case against Manuel Uy and Sons, Inc for legal
redemption when the latter refused Mrs. Butte to redeem the said sold property.

ISSUE:

Whether or not Mrs. Angela M. Butte has the right of succession to exercise legal
redemption over the share sold by Mrs. Marie Garnier Vda de Ramirez. Whether or
not the her right for legal redemption was exercised in due time.

HELD:

Yes, Mrs. Angela M. Butte has the right of succession to exercise legal redemption
over the share sold by Mrs. Marie Garnier Vda de Ramirez for being one of the co-
owners of the heirs of the 1/6 undivided property of Jose V. Ramirez.

According to Article 1620 of the Civil Code of the Philippines, a co-owner of a thing
may exercise the right of redemption in case the shares of all the other co-owners
or of any of them, are sold to a third person. If the price of the alienation is gross
expensive, the redemptioner shall pay only a reasonable one.

Should two or more co-owners desire to exercise the right to redemption, they may
only do so in proportion to the share that may respectively have in the thing owned
in common.

Yes, The notice which became operative is that given by Mrs. Chambers, in her
capacity as attorney-in-fact of the vendor Marie Garnier Vda. de Ramirez. Under
date of December 11, 1958, she wrote the Administrator Bank of the Philippine
Islands that her principal's one-sixth (1/6) share in the Sta. Cruz property had been
sold to Manuel Uy& Sons, Inc. for P500,000.00. The Bank received this notice on
December 15, 1958, and on the same day endorsed it to Mrs. Butte, care of
Delgado, Flores and Macapagal (her attorneys), who received the same on
December 16, 1958. Mrs. Butte tendered redemption and upon the vendee's
refusal, judicially consigned the price of P500,000.00 on January 15, 1959. The
latter date was the last one of the thirty days allowed by the Code for the
redemption, counted by excluding December 16, 1958 and including January 15,
1959, pursuant to Article 13 of the Civil Code. Therefore, the redemption was made
in due time.
91. Conejero v. CA, 16 SCRA 775 (1966)

FACTS:

Paz Torres and Enrique Torres were co-owners pro indiviso of a lot and building in
Cebu City that both had inherited from their deceased parents.

Enrique Torres sold his half interest to the Spouses Raffiñan with right to
repurchase within 1 year.

6 months after the expiration of the right to repurchase, said Enrique executed a
deed of absolute sale of the same half interest in the property in favor of the
Raffiñans. This deed of absolute sale had not been brought to the attention of
Enrique's sister and co-owner, Paz Torres de Conejero, nor of her husband, until
August 19, 1952, when Enrique Torres showed his brother-in-law, a copy of the
deed of absolute sale of his share of the property in favor of the Raffiñans.

Conejero forthwith went to the buyers, offering to redeem his brother-in-law's


share.

Conejeros filed a complaint seeking to be declared entitled to redeem the half


interest of Enrique Torres; to which the Raffiñans made answer, claiming absolute
title to the property in dispute and pleading that plaintiffs lost their right
of redemption because they failed to exercise it within the statutory period.

ISSUES:

Whether there was a valid redemption? NO

HELD:

In legal preemption or redemption under the Civil Code of the Philippines, written
notice of the sale to all possible redemptioner is indispensable.

Mere knowledge of the sale, acquired in some other manner by the redemptioner, is
not sufficient. Article 1623 of the Civil Code does not prescribe any particular form
of notice, or distinctive method for notifying the redemptioner. So long, therefore,
as the latter is informed in writing of the sale and the particulars thereof, the 30-
day period for redemption starts running. In the case at bar, the redemptioner
admit that their co-owner vendor gave them a copy of the deed of sale of his
undivided share in favor of respondent spouses. The furnishing ofthis copy was
equivalent to the giving of written notice required by law.

As a necessary consequence, the 30-day period for the legal redemption byco-
owner Paz Torres began to run from August 19, 1952, ending on September18, of
the same year.

Conejeros failed to make a valid tender of the price of the sale paid by theRaffiñans
within the period fixed by law. Conejero merely offered a check forP10,000, which
was not even legal tender and which the Raffiñans rejected, inlieu of the price
of P28,000 recited by the deed of sale. Nor were the vendeesobligated to accept
Conejero's promise to pay the balance by means of aloan to be obtained in future
from a bank. Bona fide redemption necessarilyimports a seasonable and valid
tender of the entire repurchase price, and thiswas not done. There is no cogent
reason for requiring the vendee to acceptpayment by installments from a
redemptioner, as it would ultimately result in anindefinite extension of the 30-day
redemption period, when the purpose of thelaw in fixing a short and definite term is
clearly to avoid prolonged and anti-economic uncertainty as to ownership of the
thing sold.
92. Doromal v. CA, 66 SCRA 575 (1975)

FACTS:
A parcel of land in Iloilo were co-owned by 7 siblings all surnamed Horilleno.
5 of the siblings gave a SPA to their niece Mary Jimenez, who succeeded her
father as a co-owner, for the sale of the land to father and son Doromal. One
of the co-owner, herein petitioner, Filomena Javellana however did not gave
her consent to the sale even though her siblings executed a SPA for her
signature. The co-owners went on with the sale of 6/7 part of the land and a
new title for the Doromals were issued.

Respondent offered to repurchase the land for 30K as stated in the deed of
sale but petitioners declined invoking lapse in time for the right of
repurchase. Petitioner also contend that the 30K price was only placed in the
deed of sale to minimize payment of fees and taxes and as such, respondent
should pay the real price paid which was P115, 250.

ISSUE:
WON the period to repurchase of petitioner has already lapsed.
WON the respondent should pay the price stipulated in the contract or the
actual price paid.

HELD:
Period of repurchase has not yet lapsed because the respondent was not
notified of the sale. The 30-day period for the right of repurchase starts only
after actual notice not only of a perfected sale but of actual execution and
delivery of the deed of sale.

The letter sent to the respondent by the other co-owners cannot be


considered as actual notice because the letter was only to inform her of the
intention to sell the property but not its actual sale. As such, the 30-day
period has not yet commenced and the respondent can still exercise his right
to repurchase.

The respondent should pay only the 30K stipulated in the deed of sale
because a redemptioner’s right is to be subrogated by the same terms and
conditions stipulated in the contract.
93. Francisco v. Boiser, 332 SCRA 305 (2000)

FACTS:

Petitioner Adalia B. Francisco and three of her sisters, Ester, Elizabeth and
Adeluisa, were co-owners of four parcels of registered lands. On August 6,
1979, they sold 1/5 of their undivided share in the subject parcels of land to
their mother, Adela Blas, for P10,000.00, thus making the latter a co-owner
of said real property to the extent of the share sold.

On August 8, 1986, without the knowledge of the other co-owners, Adela


Blas sold her 1/5 share for P10,000.00 to respondent Zenaida Boiser who is
another sister of petitioner.

On August 5, 1992, petitioner received summons, with a copy of the


complaint in Civil Case No. 15510, filed by respondent demanding her share
in the rentals being collected by petitioner from the tenants of the building.
Petitioner then informed respondent that she was exercising her right of
redemption as a co-owner of the subject property. On August 12, 1992, she
deposited the amount of P10,000.00 as redemption price with the Clerk of
Court.

She alleged that the 30-day period for redemption under Art. 1623 of the
Civil Code had not begun to run against her since the vendor, Adela Blas,
never informed her and the other owners about the sale to respondent. She
learned about the sale only on August 5, 1992, after she received the
summons in Civil Case No. 15510, together with the complaint.

Respondent, on the other hand, contended that petitioner knew about the
sale as early as May 30, 1992, because, on that date, she wrote petitioner a
letter informing the latter about the sale, with a demand that the rentals
corresponding to her 1/5 share of the subject property be remitted to her.
On the same date, letters were likewise sent by respondent to the tenants of
the building, namely, Seiko Service Center and Glitters Corporation,
informing them of the sale and requesting that, thenceforth, they pay 1/5 of
the monthly rentals to respondent. That petitioner received these letters is
proved by the fact that on June 8, 1992, she wrote the building’s tenants
advising them to disregard respondent’s request and continue paying full
rentals directly to her.

ISSUE:

whether or not a notice sent by the vendee may be given in lieu of that
required to be given by the vendor or prospective vendor.

RULING:
No. The right of legal pre-emption or redemption shall not be exercised
except within thirty days from the notice in writing by the prospective
vendor, or by the vendor, as the case maybe. The test of Article 1623 clearly
and expressly prescribes that the thirty days for making the redemption are
to be counted from notice in writing by the vendor. Under the old law (Civil
Code of 1889, Art. 1524), it was immaterial who gave the notice; so long as
the redeeming co-owner learned of the alienation in favor of the stranger,
the redemption period began to run. It is thus apparent that the Philippine
legislature in Article 1623 deliberately selected a particular method of giving
notice, and that method must be deemed exclusive
94. Misterio v. Cebu State College of Science and Technology, 461
SCRA 122 (2005)

FACTS:

Asuncion sold to Sudlon Agricultural High School(SAHS) a parcel of land,


reserving the right to repurchase the same in case (1) the school ceases to
exist, or (2) the school transfers location. She had her right annotated. She
died. By virtue of BP 412, SAHS was merged with the Cebu State College,
effective June1983. In 1990, the heirs of Asuncion sought to exercise their
right to redeem, claiming that school has ceased to exist.

ISSUE:
W/N the heirs of Asuncion may still exercise their right to redeem the
property

HELD:
NO. Their right has already prescribed. Considering that no period for
redemption was agreed upon, the law imposes a 4-year limitation. This
means that from the time the school was merged to Cebu State College,
they had 4 years, or until June 1987 to redeem the property. However, they
failed to do so within the period. Failure to redeem automatically
consolidates ownership in favor of the vendee. The fact that the right to
redeem was annotated does not make it imprescriptible, it only serves to
notify third persons.

You might also like